[obm-l] Re: [obm-l] Re: [obm-l] cadeira de 3 pés

2023-01-23 Por tôpico Claudio Buffara
Obrigado, Wagner e Ponce:

Eu tinha pensado em algo na linha do que o Ponce escreveu, ainda que em
certos casos patológicos (pelo menos de piso...) o terceiro pé pode não
encontrar apoio: imagine um piso com um pico fino em algum lugar (p.ex. a
superfície gerada pela revolução de z = 1/(1+(x^2+y^2)^20) em torno do eixo
z). Neste caso, precisaríamos de uma cadeira bem pequena, ou pelo menos com
as pontas dos pés bem próximas umas das outras.

Mas, pelo que o Wagner escreveu, acho que ainda tem um teorema mais
profundo aí.

[]s,
Claudio.


On Mon, Jan 23, 2023 at 11:54 AM Rogerio Ponce  wrote:

> Ola' Claudio!
> Eu diria que as duas explicações estão erradas, pois não se depende de
> ter apenas um plano definido pelas pontas dos pés, visto que uma
> cadeira de 4 pés pode, perfeitamente, ter as pontas dos 4 pés em
> apenas um plano, e, ainda assim, ela não é necessariamente estável.
>
> Um explicação menos ruim é que, numa cadeira de 3 pés, sempre podemos
> apoiar quaisquer 2 pés num piso (mesmo irregular), e, em torno do eixo
> definido pelos 2 pés já apoiados, podemos girar a cadeira até que o
> terceiro pé encontre o piso, de modo que a cadeira fique totalmente
> apoiada.
> Já numa cadeira de 4 pés, é comum que um dos pés fique sem contato com
> o chão, permitindo que a cadeira oscile em torno do eixo definido
> pelos 2 pés vizinhos ao pé sem contato.
>
> []'s
> Rogerio Ponce
>
> On Sun, Jan 22, 2023 at 11:23 PM Claudio Buffara
>  wrote:
> >
> > Achei na internet duas explicações distintas para a estabilidade de uma
> cadeira (ou mesa ou banco) de 3 pés.
> > Aqui estão:
> > https://www.somatematica.com.br/curiosidades/c98.php
> >
> http://colegiofarroupilha.com.br/site/qual-cadeira-e-mais-firme-a-que-tem-tres-ou-quatro-pes/
> >
> > Qual das duas é a explicação correta?
> > Ou nenhuma das duas? E, nesse caso, qual a explicação?
> >
> > []s,
> > Claudio.
> >
> > --
> > Esta mensagem foi verificada pelo sistema de antivírus e
> > acredita-se estar livre de perigo.
>
> --
> Esta mensagem foi verificada pelo sistema de antivírus e
>  acredita-se estar livre de perigo.
>
>
> =
> Instru�ões para entrar na lista, sair da lista e usar a lista em
> http://www.mat.puc-rio.br/~obmlistas/obm-l.html
> =
>

-- 
Esta mensagem foi verificada pelo sistema de antiv�rus e
 acredita-se estar livre de perigo.



[obm-l] Re: [obm-l] Re: [obm-l] cadeira de 3 pés

2023-01-23 Por tôpico Anderson Torres
Em seg, 23 de jan de 2023 11:54, Rogerio Ponce 
escreveu:

> Ola' Claudio!
> Eu diria que as duas explicações estão erradas, pois não se depende de
> ter apenas um plano definido pelas pontas dos pés, visto que uma
> cadeira de 4 pés pode, perfeitamente, ter as pontas dos 4 pés em
> apenas um plano, e, ainda assim, ela não é necessariamente estável.
>

Como não? A ideia de botar um calço é precisamente estabilizar o pé que não
encosta na superfície. E, exceto em feiras de ciências e circos, o chão
costuma ser plano.


> Um explicação menos ruim é que, numa cadeira de 3 pés, sempre podemos
> apoiar quaisquer 2 pés num piso (mesmo irregular), e, em torno do eixo
> definido pelos 2 pés já apoiados, podemos girar a cadeira até que o
> terceiro pé encontre o piso, de modo que a cadeira fique totalmente
> apoiada.
> Já numa cadeira de 4 pés, é comum que um dos pés fique sem contato com
> o chão, permitindo que a cadeira oscile em torno do eixo definido
> pelos 2 pés vizinhos ao pé sem contato.
>
> []'s
> Rogerio Ponce
>
> On Sun, Jan 22, 2023 at 11:23 PM Claudio Buffara
>  wrote:
> >
> > Achei na internet duas explicações distintas para a estabilidade de uma
> cadeira (ou mesa ou banco) de 3 pés.
> > Aqui estão:
> > https://www.somatematica.com.br/curiosidades/c98.php
> >
> http://colegiofarroupilha.com.br/site/qual-cadeira-e-mais-firme-a-que-tem-tres-ou-quatro-pes/
> >
> > Qual das duas é a explicação correta?
> > Ou nenhuma das duas? E, nesse caso, qual a explicação?
> >
> > []s,
> > Claudio.
> >
> > --
> > Esta mensagem foi verificada pelo sistema de antivírus e
> > acredita-se estar livre de perigo.
>
> --
> Esta mensagem foi verificada pelo sistema de antivírus e
>  acredita-se estar livre de perigo.
>
>
> =
> Instru�ões para entrar na lista, sair da lista e usar a lista em
> http://www.mat.puc-rio.br/~obmlistas/obm-l.html
> =
>

-- 
Esta mensagem foi verificada pelo sistema de antiv�rus e
 acredita-se estar livre de perigo.



[obm-l] Re: [obm-l] Re: [obm-l] Re: [obm-l] cadeira de 3 pés

2023-01-23 Por tôpico Anderson Torres
Em seg, 23 de jan de 2023 11:15, Claudio Buffara 
escreveu:

> Será que o argumento usando apenas o plano é suficiente?  Pois um banco de
> 3 pés também fica estável num piso irregular.
>

Mas nem toda cadeira de quatro pés fica estável em qualquer piso irregular.

A ideia subjacente ainda é a de "planidade". Os pés da cadeira estão na
intersecção entre o "plano dos pés" e o piso.
Mas o mesmo não se aplica às cadeiras quadrúpedes, pois nem sempre existe
um "plano dos pés".

Se bem que neste último caso é possível que os pés da mesa sejam coplanares
mas a superfície não o seja. Ou pior ainda, forçando um pouco na
continuidade, pode ser que qualquer cadeira de quatro pés se encaixe em
qualquer superfície não-plana.

Ou não. Se pegarmos por exemplo o mapa topográfico de uma planície com um
poço escavado e a distância entre dois pés da mesa sempre for maior que o
diâmetro do poço, não tem como encaixar uma mesa torta de quatro pés.

Estranho...

O resultado mais geral em que pensei foi o seguinte: dada qualquer
> superfície bi-dimensional contínua (por exemplo, que seja o gráfico de uma
> função contínua de RxR em R - uma suposição razoável se estamos tentando
> modelar um piso), você sempre consegue encostar nela as pontas dos 3 pés do
> banco, de modo que o banco fique "firme" ou sem folgas.
> Ou, mais formalmente, dado um triângulo ABC no espaço, existe uma
> isometria (do espaço) tal que as imagens de A, B e C por esta isometria
> estão em S.
>

Pensei algo do gênero, mas de maneira bem menos elaborada: dados um
triângulo ABC e dois pontos A',B' em uma superfície contínua que distam AB,
existe um ponto C´ tal que ABC = A'B'C'


> Enfim, esse talvez seja um problema mais de topologia do que de geometria.
> Pois, no fim das contas, "3 pontos não colineares determinam um único
> plano", assim como "2 pontos determinam uma única reta" são afirmações que
> têm um certo ar topológico, pelo menos pra mim.
>

> []s,
> Claudio.
>
> On Mon, Jan 23, 2023 at 7:02 AM Anderson Torres <
> torres.anderson...@gmail.com> wrote:
>
>>
>>
>> Em dom, 22 de jan de 2023 23:23, Claudio Buffara <
>> claudio.buff...@gmail.com> escreveu:
>>
>>> Achei na internet duas explicações distintas para a estabilidade de uma
>>> cadeira (ou mesa ou banco) de 3 pés.
>>> Aqui estão:
>>> https://www.somatematica.com.br/curiosidades/c98.php
>>>
>>> http://colegiofarroupilha.com.br/site/qual-cadeira-e-mais-firme-a-que-tem-tres-ou-quatro-pes/
>>>
>>
>> Nesse caso específico, a primeira me parece mais correta. Ou melhor, a
>> segunda tem falhas.
>>
>> A rigidez dos triângulos (TCC caso LLL de igualdade de triângulos) é
>> irrelevante para a questão da cadeira bamba. Em qualquer cadeira física e
>> palpável, as pernas são rígidas - portanto o polígono formado pelas
>> extremidades dessas pernas é rígido também.
>>
>> Mas um polígono rígido não é necessariamente um polígono bidimensional -
>> o que a cadeira bamba de 4 pernas exemplifica perfeitamente.
>>
>> Contate o webmaster da segunda página sugerindo correções!
>>
>>
>>> Qual das duas é a explicação correta?
>>> Ou nenhuma das duas? E, nesse caso, qual a explicação?
>>>
>>> []s,
>>> Claudio.
>>>
>>> --
>>> Esta mensagem foi verificada pelo sistema de antivírus e
>>> acredita-se estar livre de perigo.
>>
>>
>> --
>> Esta mensagem foi verificada pelo sistema de antivírus e
>> acredita-se estar livre de perigo.
>
>
> --
> Esta mensagem foi verificada pelo sistema de antivírus e
> acredita-se estar livre de perigo.

-- 
Esta mensagem foi verificada pelo sistema de antiv�rus e
 acredita-se estar livre de perigo.



[obm-l] Re: [obm-l] cadeira de 3 pés

2023-01-23 Por tôpico Rogerio Ponce
Ola' Claudio!
Eu diria que as duas explicações estão erradas, pois não se depende de
ter apenas um plano definido pelas pontas dos pés, visto que uma
cadeira de 4 pés pode, perfeitamente, ter as pontas dos 4 pés em
apenas um plano, e, ainda assim, ela não é necessariamente estável.

Um explicação menos ruim é que, numa cadeira de 3 pés, sempre podemos
apoiar quaisquer 2 pés num piso (mesmo irregular), e, em torno do eixo
definido pelos 2 pés já apoiados, podemos girar a cadeira até que o
terceiro pé encontre o piso, de modo que a cadeira fique totalmente
apoiada.
Já numa cadeira de 4 pés, é comum que um dos pés fique sem contato com
o chão, permitindo que a cadeira oscile em torno do eixo definido
pelos 2 pés vizinhos ao pé sem contato.

[]'s
Rogerio Ponce

On Sun, Jan 22, 2023 at 11:23 PM Claudio Buffara
 wrote:
>
> Achei na internet duas explicações distintas para a estabilidade de uma 
> cadeira (ou mesa ou banco) de 3 pés.
> Aqui estão:
> https://www.somatematica.com.br/curiosidades/c98.php
> http://colegiofarroupilha.com.br/site/qual-cadeira-e-mais-firme-a-que-tem-tres-ou-quatro-pes/
>
> Qual das duas é a explicação correta?
> Ou nenhuma das duas? E, nesse caso, qual a explicação?
>
> []s,
> Claudio.
>
> --
> Esta mensagem foi verificada pelo sistema de antivírus e
> acredita-se estar livre de perigo.

-- 
Esta mensagem foi verificada pelo sistema de antiv�rus e
 acredita-se estar livre de perigo.


=
Instru��es para entrar na lista, sair da lista e usar a lista em
http://www.mat.puc-rio.br/~obmlistas/obm-l.html
=


[obm-l] Re: [obm-l] cadeira de 3 pés

2023-01-23 Por tôpico Eduardo Wagner
O banco de 3 pernas não balança porque nosso mundo é tridimensional.
Não tem nada a ver com plano ou triângulo. Um banco de 3 pernas não balança
se for colocado
no teto de um carro.
No mundo 2D um banco de 2 pernas não balança, mas um de 3 pernas pode
balançar.
Em um mundo 4D uma cadeira de 4 pernas não balança.
Wbs
Wagner


Em dom., 22 de jan. de 2023 às 23:24, Claudio Buffara <
claudio.buff...@gmail.com> escreveu:

> Achei na internet duas explicações distintas para a estabilidade de uma
> cadeira (ou mesa ou banco) de 3 pés.
> Aqui estão:
> https://www.somatematica.com.br/curiosidades/c98.php
>
> http://colegiofarroupilha.com.br/site/qual-cadeira-e-mais-firme-a-que-tem-tres-ou-quatro-pes/
>
> Qual das duas é a explicação correta?
> Ou nenhuma das duas? E, nesse caso, qual a explicação?
>
> []s,
> Claudio.
>
> --
> Esta mensagem foi verificada pelo sistema de antivírus e
> acredita-se estar livre de perigo.

-- 
Esta mensagem foi verificada pelo sistema de antiv�rus e
 acredita-se estar livre de perigo.



[obm-l] Re: [obm-l] Re: [obm-l] cadeira de 3 pés

2023-01-23 Por tôpico Claudio Buffara
Será que o argumento usando apenas o plano é suficiente?  Pois um banco de
3 pés também fica estável num piso irregular.
O resultado mais geral em que pensei foi o seguinte: dada qualquer
superfície bi-dimensional contínua (por exemplo, que seja o gráfico de uma
função contínua de RxR em R - uma suposição razoável se estamos tentando
modelar um piso), você sempre consegue encostar nela as pontas dos 3 pés do
banco, de modo que o banco fique "firme" ou sem folgas.
Ou, mais formalmente, dado um triângulo ABC no espaço, existe uma isometria
(do espaço) tal que as imagens de A, B e C por esta isometria estão em S.

Enfim, esse talvez seja um problema mais de topologia do que de geometria.
Pois, no fim das contas, "3 pontos não colineares determinam um único
plano", assim como "2 pontos determinam uma única reta" são afirmações que
têm um certo ar topológico, pelo menos pra mim.

[]s,
Claudio.

On Mon, Jan 23, 2023 at 7:02 AM Anderson Torres <
torres.anderson...@gmail.com> wrote:

>
>
> Em dom, 22 de jan de 2023 23:23, Claudio Buffara <
> claudio.buff...@gmail.com> escreveu:
>
>> Achei na internet duas explicações distintas para a estabilidade de uma
>> cadeira (ou mesa ou banco) de 3 pés.
>> Aqui estão:
>> https://www.somatematica.com.br/curiosidades/c98.php
>>
>> http://colegiofarroupilha.com.br/site/qual-cadeira-e-mais-firme-a-que-tem-tres-ou-quatro-pes/
>>
>
> Nesse caso específico, a primeira me parece mais correta. Ou melhor, a
> segunda tem falhas.
>
> A rigidez dos triângulos (TCC caso LLL de igualdade de triângulos) é
> irrelevante para a questão da cadeira bamba. Em qualquer cadeira física e
> palpável, as pernas são rígidas - portanto o polígono formado pelas
> extremidades dessas pernas é rígido também.
>
> Mas um polígono rígido não é necessariamente um polígono bidimensional - o
> que a cadeira bamba de 4 pernas exemplifica perfeitamente.
>
> Contate o webmaster da segunda página sugerindo correções!
>
>
>> Qual das duas é a explicação correta?
>> Ou nenhuma das duas? E, nesse caso, qual a explicação?
>>
>> []s,
>> Claudio.
>>
>> --
>> Esta mensagem foi verificada pelo sistema de antivírus e
>> acredita-se estar livre de perigo.
>
>
> --
> Esta mensagem foi verificada pelo sistema de antivírus e
> acredita-se estar livre de perigo.

-- 
Esta mensagem foi verificada pelo sistema de antiv�rus e
 acredita-se estar livre de perigo.



[obm-l] Re: [obm-l] cadeira de 3 pés

2023-01-23 Por tôpico Anderson Torres
Em dom, 22 de jan de 2023 23:23, Claudio Buffara 
escreveu:

> Achei na internet duas explicações distintas para a estabilidade de uma
> cadeira (ou mesa ou banco) de 3 pés.
> Aqui estão:
> https://www.somatematica.com.br/curiosidades/c98.php
>
> http://colegiofarroupilha.com.br/site/qual-cadeira-e-mais-firme-a-que-tem-tres-ou-quatro-pes/
>

Nesse caso específico, a primeira me parece mais correta. Ou melhor, a
segunda tem falhas.

A rigidez dos triângulos (TCC caso LLL de igualdade de triângulos) é
irrelevante para a questão da cadeira bamba. Em qualquer cadeira física e
palpável, as pernas são rígidas - portanto o polígono formado pelas
extremidades dessas pernas é rígido também.

Mas um polígono rígido não é necessariamente um polígono bidimensional - o
que a cadeira bamba de 4 pernas exemplifica perfeitamente.

Contate o webmaster da segunda página sugerindo correções!


> Qual das duas é a explicação correta?
> Ou nenhuma das duas? E, nesse caso, qual a explicação?
>
> []s,
> Claudio.
>
> --
> Esta mensagem foi verificada pelo sistema de antivírus e
> acredita-se estar livre de perigo.

-- 
Esta mensagem foi verificada pelo sistema de antiv�rus e
 acredita-se estar livre de perigo.



Re: [obm-l] Irracionalidade de Pi

2023-01-22 Por tôpico Claudio Buffara
Valeu!
E os links extras contém uma boa discussão, às vezes meio acalorada, sobre
motivação pra certas demonstrações.
Eu particularmente me interesso bastante por este tema.
Pois acho que demonstrações "mágicas", baseadas em ideias "vindas do além",
são problemáticas do ponto de vista pedagógico, pois acho que podem
desmotivar estudantes de matemática, que passam a achar que o assunto é só
pra gênios.
Estas não devem ser confundidas com demonstrações/soluções brilhantes mas
que são "óbvias a posteriori", ou seja, que dependem de uma sacada que o
leitor poderia ter tido se tivesse prestado mais atenção ou feito um
desenho mais preciso ou pensado um pouquinho mais no problema.  Não me
parece ser o caso dessa demonstração do Niven da irracionalidade de Pi.

[]s,
Claudio.




On Sun, Jan 22, 2023 at 9:53 AM Anderson Torres <
torres.anderson...@gmail.com> wrote:

> Em sáb., 21 de jan. de 2023 às 13:27, Claudio Buffara
>  escreveu:
> >
> > A demonstração tradicional da irracionalidade de Pi começa estabelecendo
> algumas propriedades da função:
> > x |--> x^n * (1-x)^n / n!
> > no intervalo (0,1).
> >
> > Essa função me parece tirada da cartola, sem qualquer motivação prévia.
> > Alguém sabe o que levou o autor da demonstração a usar esta função?
>
> Bem, eu fiz uma rápida busca no Google por "motivated demonstration
> irrationality pi" e encontrei isso:
>
> "Discovering and Proving that π Is Irrational" por Timothy W. Jones
>
> The American Mathematical Monthly
> Vol. 117, No. 6 (June-July 2010), pp. 553-557 (5 pages)
> https://doi.org/10.4169/000298910x492853
>
> E também uns links extras:
>
>
> https://mattbaker.blog/2015/03/15/a-motivated-and-simple-proof-that-pi-is-irrational/
>
> https://math.stackexchange.com/questions/4051354/what-is-the-motivation-behind-the-steps-in-this-simple-proof-that-pi-is-irr
> https://page.math.tu-berlin.de/~mdmv/archive/19/mdmv-19-zhou.pdf
>
> Divirta-se :)
>
> >
> > []s,
> > Claudio.
> >
> > --
> > Esta mensagem foi verificada pelo sistema de antivírus e
> > acredita-se estar livre de perigo.
>
> --
> Esta mensagem foi verificada pelo sistema de antivírus e
>  acredita-se estar livre de perigo.
>
>
> =
> Instru�ões para entrar na lista, sair da lista e usar a lista em
> http://www.mat.puc-rio.br/~obmlistas/obm-l.html
> =
>

-- 
Esta mensagem foi verificada pelo sistema de antiv�rus e
 acredita-se estar livre de perigo.



Re: [obm-l] Irracionalidade de Pi

2023-01-22 Por tôpico Anderson Torres
Em sáb., 21 de jan. de 2023 às 13:27, Claudio Buffara
 escreveu:
>
> A demonstração tradicional da irracionalidade de Pi começa estabelecendo 
> algumas propriedades da função:
> x |--> x^n * (1-x)^n / n!
> no intervalo (0,1).
>
> Essa função me parece tirada da cartola, sem qualquer motivação prévia.
> Alguém sabe o que levou o autor da demonstração a usar esta função?

Bem, eu fiz uma rápida busca no Google por "motivated demonstration
irrationality pi" e encontrei isso:

"Discovering and Proving that π Is Irrational" por Timothy W. Jones

The American Mathematical Monthly
Vol. 117, No. 6 (June-July 2010), pp. 553-557 (5 pages)
https://doi.org/10.4169/000298910x492853

E também uns links extras:

https://mattbaker.blog/2015/03/15/a-motivated-and-simple-proof-that-pi-is-irrational/
https://math.stackexchange.com/questions/4051354/what-is-the-motivation-behind-the-steps-in-this-simple-proof-that-pi-is-irr
https://page.math.tu-berlin.de/~mdmv/archive/19/mdmv-19-zhou.pdf

Divirta-se :)

>
> []s,
> Claudio.
>
> --
> Esta mensagem foi verificada pelo sistema de antivírus e
> acredita-se estar livre de perigo.

-- 
Esta mensagem foi verificada pelo sistema de antiv�rus e
 acredita-se estar livre de perigo.


=
Instru��es para entrar na lista, sair da lista e usar a lista em
http://www.mat.puc-rio.br/~obmlistas/obm-l.html
=


[obm-l] Re: [obm-l] Mostrar que [n!]/e é sempre par

2022-12-15 Por tôpico Anderson Torres
Em sex, 16 de dez de 2022 00:53, Artur Costa Steiner <
artur.costa.stei...@gmail.com> escreveu:

> Problema interessante: Mostre que, para todo inteiro n >= 0,  [n!]/e é
> sempre par, sendo [x] o piso de x.
>

você quis dizer [n!/e] onde e é a base do log natural?

Bem, 1/e=e^(-1)=
(1/0!-1/1!)+(1/2!-1/3!)+(1/4!-1/5!)+(1/6!-1/7!)+... =
2/3! + 4/5! + 6/7! + 8/9! + ...

Dessa forma, [n!/e]=

[2n!/3! + 4n!/5! + 6n!/7! + 8n!/9! + ...]

Que, obviamente, redunda numa sema de pares após truncado (denominador
>n!),


> Abraços
>
> Artur
>
> --
> Esta mensagem foi verificada pelo sistema de antivírus e
> acredita-se estar livre de perigo.

-- 
Esta mensagem foi verificada pelo sistema de antiv�rus e
 acredita-se estar livre de perigo.



[obm-l] Re: [obm-l] Teoria dos números, trigonometria e racionalidade

2022-12-11 Por tôpico Anderson Torres
Em dom., 11 de dez. de 2022 às 10:32, Anderson Torres
 escreveu:
>
> Em sáb., 10 de dez. de 2022 às 22:08, marcone augusto araújo borges
>  escreveu:
> >
> > Seja p um número primo tal que p = = 3 (mod4) e @ um ângulo tal que tan@ é 
> > racional. Prove que tan((p+1)@) também é racional com numerador múltiplo de 
> > p
> > Desde já agradeço por algum esclarecimento ou solução.
>
> Bem, o que eu consigo pensar é em algo desse tipo.
>
> Sabemos que tan(m+n) = (tan(m) + tan(n))/(1-tan(m)* tan(n))
>
> Escrevamos tan(nX)=p(n)/q(n), onde p e q são polinômios em t=tan(X).
> Temos então a seguinte recorrência:
>
> p(1)=t; p(n+1)=p(n)+tq(n)
> q(1)=1; q(n+1)=-tp(n)+q(n)
>
> Jogando aqui e ali, temos
>
> p(1)=t; p(2)=2t; p(n+2)=2p(n+1)-(t^2+1)p(n)
> q(1)=1; q(2)=1-t^2; q(n+2)=2q(n+1)-(t^2+1)q(n)
>
> De cara, se nota que p sempre será múltiplo de p, e que q sempre deixa
> resto 1 módulo t, o que já dá uma pista do que procurar...
> Decerto, vai aparecer alguma coisa do tipo x^2+1, e com isso se usa o
> fato de p ser primo da forma 4k-1...
>

Acho que dá para melhorar. Suponha tan(nX)=A(n)/B(n). Assim,

A(n+1) =  B*A(n) + A*B(n)
B(n+1) = -A*A(n) + B*B(n)

E portanto

A(n+2) = 2B*A(n+1) - (A^2+B^2)*A(n), A(1)=A, A(2)=2AB
B(n+2) = 2B*B(n+1) - (A^2+B^2)*B(n), B(1)=B, B(2)=B^2-A^2

A ideia então seria demonstrar que A(p+1) é múltiplo de p para p primo
da forma 4k-1, e B(p+1) não é múltiplo de p para p primo da forma
4k-1.

Dessa forma, ao menos em princípio seria possível verificar a segunda
premissa, pois a primeira é óbvia.

> >
> > --
> > Esta mensagem foi verificada pelo sistema de antivírus e
> > acredita-se estar livre de perigo.

-- 
Esta mensagem foi verificada pelo sistema de antiv�rus e
 acredita-se estar livre de perigo.


=
Instru��es para entrar na lista, sair da lista e usar a lista em
http://www.mat.puc-rio.br/~obmlistas/obm-l.html
=


[obm-l] Re: [obm-l] OBM e Olímpiadas internacionais

2022-12-11 Por tôpico Anderson Torres
Em qua., 7 de dez. de 2022 às 03:39, Obindinachukwu Desire Yema
 escreveu:
>
> Bom dia a todos,
> Nesse ano eu despertei um interesse em matemática pura, pensando um pouco 
> decidi que iria tentar no próximo ano fazer a OBM nivel universitário. 
> Pesquisando no site da OBM, eu não achei nada relacionado com o conteúdo que 
> cai na prova.
> Eu queria perguntar para vocês como que me preparo para a prova, no sentido 
> de: conteúdo que devo saber.
> Desde já agradeço a atenção.

De fato tem pouca coisa além das provas. Então, te sugiro pegar
pesado: estude a PUTNAM e a IMC. Com isso você vai ter mais material.

>
> --
> Esta mensagem foi verificada pelo sistema de antivírus e
> acredita-se estar livre de perigo.

-- 
Esta mensagem foi verificada pelo sistema de antiv�rus e
 acredita-se estar livre de perigo.


=
Instru��es para entrar na lista, sair da lista e usar a lista em
http://www.mat.puc-rio.br/~obmlistas/obm-l.html
=


[obm-l] Re: [obm-l] Teoria dos números, trigonometria e racionalidade

2022-12-11 Por tôpico Anderson Torres
Em sáb., 10 de dez. de 2022 às 22:08, marcone augusto araújo borges
 escreveu:
>
> Seja p um número primo tal que p = = 3 (mod4) e @ um ângulo tal que tan@ é 
> racional. Prove que tan((p+1)@) também é racional com numerador múltiplo de p
> Desde já agradeço por algum esclarecimento ou solução.

Bem, o que eu consigo pensar é em algo desse tipo.

Sabemos que tan(m+n) = (tan(m) + tan(n))/(1-tan(m)* tan(n))

Escrevamos tan(nX)=p(n)/q(n), onde p e q são polinômios em t=tan(X).
Temos então a seguinte recorrência:

p(1)=t; p(n+1)=p(n)+tq(n)
q(1)=1; q(n+1)=-tp(n)+q(n)

Jogando aqui e ali, temos

p(1)=t; p(2)=2t; p(n+2)=2p(n+1)-(t^2+1)p(n)
q(1)=1; q(2)=1-t^2; q(n+2)=2q(n+1)-(t^2+1)q(n)

De cara, se nota que p sempre será múltiplo de p, e que q sempre deixa
resto 1 módulo t, o que já dá uma pista do que procurar...
Decerto, vai aparecer alguma coisa do tipo x^2+1, e com isso se usa o
fato de p ser primo da forma 4k-1...

>
> --
> Esta mensagem foi verificada pelo sistema de antivírus e
> acredita-se estar livre de perigo.

-- 
Esta mensagem foi verificada pelo sistema de antiv�rus e
 acredita-se estar livre de perigo.


=
Instru��es para entrar na lista, sair da lista e usar a lista em
http://www.mat.puc-rio.br/~obmlistas/obm-l.html
=


[obm-l] Re: [obm-l] OBM e Olímpiadas internacionais

2022-12-07 Por tôpico Claudio Buffara
Eu começaria olhando as provas de anos anteriores, por exemplo aqui:
https://www.obm.org.br/como-se-preparar/provas-e-gabaritos/

On Wed, Dec 7, 2022 at 3:39 AM Obindinachukwu Desire Yema <
obindinachukwu.y...@usp.br> wrote:

> Bom dia a todos,
> Nesse ano eu despertei um interesse em matemática pura, pensando um pouco
> decidi que iria tentar no próximo ano fazer a OBM nivel universitário.
> Pesquisando no site da OBM, eu não achei nada relacionado com o conteúdo
> que cai na prova.
> Eu queria perguntar para vocês como que me preparo para a prova, no
> sentido de: conteúdo que devo saber.
> Desde já agradeço a atenção.
>
> --
> Esta mensagem foi verificada pelo sistema de antivírus e
> acredita-se estar livre de perigo.

-- 
Esta mensagem foi verificada pelo sistema de antiv�rus e
 acredita-se estar livre de perigo.



[obm-l] Re: [obm-l] Re: [obm-l] Re: [obm-l] Caracterização de Inteiros

2022-11-17 Por tôpico Claudio Buffara
Só completando...

Apesar de números irracionais serem conhecidos desde a época de Pitágoras
(vide a famosa historinha do pitagórico Hipaso, que supostamente foi
afogado por ter "vazado" o segredo da existência dos irracionais), me
parece que eles só começaram a realmente fazer falta no século 19, quando
os matemáticos estavam preocupados em formalizar a análise. Foi naquele
ponto que o Dedekind se deu conta da importância da completeza (essa
palavra tá certa?...) dos reais e da necessidade dela ser garantida por um
axioma. Se não me engano, o teorema que ele queria provar é o de que uma
sequência monótona e limitada de números reais sempre converge. E descobriu
que não conseguiria sem fazer uma hipótese adicional sobre o corpo
dos reais, capaz de diferenciá-lo dos racionais (também um corpo ordenado),
mas dentro do qual nem toda sequência converge (por exemplo, x(n+1) =
(1/2)*(x(n) + 2/x(n)), com x(0) = 1, a qual converge pra raiz(2): um
irracional). A princípio, bastaria ele incluir o axioma de completeza dos
reais e provar que este axioma não contradiz os demais axiomas. Mas ele foi
mais longe e acabou inventando uma construção dos reais a partir dos
racionais (via os chamados "cortes de Dedekind"). Talvez (estou
conjecturando aqui) ele só tenha feito isso por uma necessidade psicológica
de provar que algo é possível (no caso a extensão dos racionais aos reais)
exibindo uma construção explícita. Isso talvez seja um reflexo da ojeriza
que matemáticos da época tinham (e alguns poucos têm até hoje) de
argumentos não construtivos pra provar a existência de fatos ou objetos
matemáticos. Por exemplo, demonstrações usando o princípio das casas de
pombos são desse tipo.

Sobre "entender sem compreender", lembrei de um ditado do John von
Neumann: "Em matemática, você nunca entende as coisas. Apenas se acostuma
com elas."

[]s,
Claudio.

On Wed, Nov 16, 2022 at 6:52 PM Claudio Buffara 
wrote:

> Não entendi como uma homotetia poderia reduzir um par ordenado a um único
> número... enfim...
>
> O que se faz, no caso da relação de equivalência que descrevi, é
> representar o par (a,b) pela notação a-b.
> Daí, (a,b) e (c,d) são equivalentes sss a-b = c-d.
> E a novidade são os números negativos: as classes de equivalência de pares
> (a,b) com a < b, representadas, por exemplo, pelo par (0,c), onde c = b-a.
> Ou, na notação usual, -c.
>
> Mas não acho que se deva perder muito tempo com a construção de sistemas
> numéricos via classes de equivalência, estendendo naturais para inteiros
> para racionais para reais e para complexos.  Até porque é extremamente
> sacal, a cada etapa, checar que as operações usuais (+ e *), quando
> aplicadas aos novos números, têm todas as propriedades que conhecemos da
> escola.
> Essas construções foram a maneira que os matemáticos acharam pra
> formalizar os sistemas numéricos, a partir de conceitos mais básicos (no
> caso, pares ordenados e relações de equivalência) - é o programa do Hilbert
> (ou de Russell e Whitehead), de reduzir toda a matemática à teoria dos
> conjuntos.
> Mas, no fundo, esta é uma construção artificial, ex post.  Pois
> matemáticos já usavam todos os números muito antes dessa formalização ser
> inventada.
> E não acho que ela renda muitos frutos, nem pedagógicos (a menos que seu
> objetivo seja "entender sem compreender") e nem pra ampliação da fronteira
> do conhecimento, exceto colocar os sistemas numéricos numa base axiomática
> sólida.
> Em particular, no que diz respeito aos números reais, a única coisa que
> interessa é que eles são um corpo ordenado completo. Tanto é que vários
> livros de análise partem deste axioma e não se preocupam em construir os
> reais a partir dos naturais.
>
> []s,
> Claudio.
>
>
>
> On Tue, Nov 15, 2022 at 5:07 PM Pedro José  wrote:
>
>> Obrigado a você e ao Cláudio. Mas não sou criativo para inventar. Mas já
>> vi que terei que fazer uma homotetia, para as classes de equivalência para
>> representar só como um número e não como um par, creio eu.
>>
>> Cordialmente,
>> PJMS
>>
>> Em ter., 15 de nov. de 2022 às 16:00, Anderson Torres <
>> torres.anderson...@gmail.com> escreveu:
>>
>>>
>>>
>>> Em ter, 15 de nov de 2022 14:33, Pedro José 
>>> escreveu:
>>>
 Boa tarde!
 Para os |Naturais, temos os postulados de Peano.

 Para os Inteiros há alguma formalização?

>>>
>>> invente uma!
>>>
>>> Pode ser por exemplo o conjunto de pares (p,q) tais que p-q é constante.
>>>
>>> ou melhor (p1,q1)=(p2,q2) se e só se p1+q2=p2+q1.
>>>
>>>
 Acho pobre dizer que é necessário ter outros números devido ao problema
 de fechamento nos naturais para a subtração que é fato e daí introduzir os
 simétricos que são inteiros e ainda não foram caracterizados.

 No meu antigo ginásio aprendi que os Reais era a união dos conjuntos
 disjuntos irracionais e racionais. Os racionais haviam sido bem definidos.
 Aí questionei e o que são irracionais? resposta: são os Reais que não são
 racionais, os que não podem 

[obm-l] Re: [obm-l] Re: [obm-l] Re: [obm-l] Caracterização de Inteiros

2022-11-16 Por tôpico Claudio Buffara
Não entendi como uma homotetia poderia reduzir um par ordenado a um único
número... enfim...

O que se faz, no caso da relação de equivalência que descrevi, é
representar o par (a,b) pela notação a-b.
Daí, (a,b) e (c,d) são equivalentes sss a-b = c-d.
E a novidade são os números negativos: as classes de equivalência de pares
(a,b) com a < b, representadas, por exemplo, pelo par (0,c), onde c = b-a.
Ou, na notação usual, -c.

Mas não acho que se deva perder muito tempo com a construção de sistemas
numéricos via classes de equivalência, estendendo naturais para inteiros
para racionais para reais e para complexos.  Até porque é extremamente
sacal, a cada etapa, checar que as operações usuais (+ e *), quando
aplicadas aos novos números, têm todas as propriedades que conhecemos da
escola.
Essas construções foram a maneira que os matemáticos acharam pra formalizar
os sistemas numéricos, a partir de conceitos mais básicos (no caso, pares
ordenados e relações de equivalência) - é o programa do Hilbert (ou de
Russell e Whitehead), de reduzir toda a matemática à teoria dos conjuntos.
Mas, no fundo, esta é uma construção artificial, ex post.  Pois matemáticos
já usavam todos os números muito antes dessa formalização ser inventada.
E não acho que ela renda muitos frutos, nem pedagógicos (a menos que seu
objetivo seja "entender sem compreender") e nem pra ampliação da fronteira
do conhecimento, exceto colocar os sistemas numéricos numa base axiomática
sólida.
Em particular, no que diz respeito aos números reais, a única coisa que
interessa é que eles são um corpo ordenado completo. Tanto é que vários
livros de análise partem deste axioma e não se preocupam em construir os
reais a partir dos naturais.

[]s,
Claudio.



On Tue, Nov 15, 2022 at 5:07 PM Pedro José  wrote:

> Obrigado a você e ao Cláudio. Mas não sou criativo para inventar. Mas já
> vi que terei que fazer uma homotetia, para as classes de equivalência para
> representar só como um número e não como um par, creio eu.
>
> Cordialmente,
> PJMS
>
> Em ter., 15 de nov. de 2022 às 16:00, Anderson Torres <
> torres.anderson...@gmail.com> escreveu:
>
>>
>>
>> Em ter, 15 de nov de 2022 14:33, Pedro José 
>> escreveu:
>>
>>> Boa tarde!
>>> Para os |Naturais, temos os postulados de Peano.
>>>
>>> Para os Inteiros há alguma formalização?
>>>
>>
>> invente uma!
>>
>> Pode ser por exemplo o conjunto de pares (p,q) tais que p-q é constante.
>>
>> ou melhor (p1,q1)=(p2,q2) se e só se p1+q2=p2+q1.
>>
>>
>>> Acho pobre dizer que é necessário ter outros números devido ao problema
>>> de fechamento nos naturais para a subtração que é fato e daí introduzir os
>>> simétricos que são inteiros e ainda não foram caracterizados.
>>>
>>> No meu antigo ginásio aprendi que os Reais era a união dos conjuntos
>>> disjuntos irracionais e racionais. Os racionais haviam sido bem definidos.
>>> Aí questionei e o que são irracionais? resposta: são os Reais que não são
>>> racionais, os que não podem ser escritos na forma p/q p e q inteiros e
>>> q<>0. Mas me deram um tombo. Definiram os |Reais com base nos irracionais e
>>> os irracionais com base nos |Reais. 3 +2i também não pode ser inscrito na
>>> forma p/q. Só mais tarde no científico, é que meu professor definiu
>>> irracional como um número que não podia ser escrito na forma p/q e cuja
>>> representação decimal tinha uma infinidade de algarismos, sem haver uma
>>> periodicidade.
>>> Na época foi o maior nó que tive com a matemática. O mestre demonstrou
>>> que os racionais eram densos, mas entre eles ainda cabiam os irracionais.
>>> Não satisfeito mostrou que os racionais eram enumeráveis e por absurdo
>>> mostrou que os |Reais não. Não satisfeito mostrou que a cardinalidade do
>>> intervalo [0,1] era maior que a dos |Naturais. Não conseguia conceber que
>>> havia um infinito maior que outro. Outra coisa que demorei a aceitar,mesmo
>>> vendo a bijeção, era que os inteiros e naturais tinham a mesma
>>> cardinalidade. Na minha cabeça, os inteiros têm todos os naturais ainda
>>> sobram os negativos, como é igual?
>>> Hoje, depois de velho, arrumei uma enteada, que muito me pergunta e
>>> estou enrolado. Para dar um ar de superioridade, questionei se conhecia os
>>> inteiros de Gaus, que 5 não era primo nos inteiros de Gaus. Estrepei-me, a
>>> danada foi pesquisar e me questiona sobre o que não tenho um domínio pleno.
>>> Em suma, como apresentei a ela os postulados de Peano para a
>>> caracterização dos Naturais, ela me cobra por algo semelhante para os
>>> Inteiros, e não sei responder.
>>> HELP! SOCORRO! AU SECOURS! AYUDA! AIUTO! HILFE!
>>> Cordialmente,
>>> PJMS
>>>
>>> --
>>> Esta mensagem foi verificada pelo sistema de antivírus e
>>> acredita-se estar livre de perigo.
>>
>>
>> --
>> Esta mensagem foi verificada pelo sistema de antivírus e
>> acredita-se estar livre de perigo.
>
>
> --
> Esta mensagem foi verificada pelo sistema de antivírus e
> acredita-se estar livre de perigo.

-- 
Esta mensagem foi verificada pelo sistema de 

[obm-l] Re: [obm-l] Re: [obm-l] Re: [obm-l] Caracterização de Inteiros

2022-11-15 Por tôpico Anderson Torres
Em ter, 15 de nov de 2022 17:07, Pedro José  escreveu:

> Obrigado a você e ao Cláudio. Mas não sou criativo para inventar. Mas já
> vi que terei que fazer uma homotetia, para as classes de equivalência para
> representar só como um número e não como um par, creio eu.
>

Eu lembro de quando li o Guidorizzi formalizando os reais. Até hoje sinto
que entendo sem compreender, haha!

Por outro lado, números reais (irracionais, no caso) são bem menos
palpáveis que os outros. Dívidas e frações são fáceis de entender, afinal.


> Cordialmente,
> PJMS
>
> Em ter., 15 de nov. de 2022 às 16:00, Anderson Torres <
> torres.anderson...@gmail.com> escreveu:
>
>>
>>
>> Em ter, 15 de nov de 2022 14:33, Pedro José 
>> escreveu:
>>
>>> Boa tarde!
>>> Para os |Naturais, temos os postulados de Peano.
>>>
>>> Para os Inteiros há alguma formalização?
>>>
>>
>> invente uma!
>>
>> Pode ser por exemplo o conjunto de pares (p,q) tais que p-q é constante.
>>
>> ou melhor (p1,q1)=(p2,q2) se e só se p1+q2=p2+q1.
>>
>>
>>> Acho pobre dizer que é necessário ter outros números devido ao problema
>>> de fechamento nos naturais para a subtração que é fato e daí introduzir os
>>> simétricos que são inteiros e ainda não foram caracterizados.
>>>
>>> No meu antigo ginásio aprendi que os Reais era a união dos conjuntos
>>> disjuntos irracionais e racionais. Os racionais haviam sido bem definidos.
>>> Aí questionei e o que são irracionais? resposta: são os Reais que não são
>>> racionais, os que não podem ser escritos na forma p/q p e q inteiros e
>>> q<>0. Mas me deram um tombo. Definiram os |Reais com base nos irracionais e
>>> os irracionais com base nos |Reais. 3 +2i também não pode ser inscrito na
>>> forma p/q. Só mais tarde no científico, é que meu professor definiu
>>> irracional como um número que não podia ser escrito na forma p/q e cuja
>>> representação decimal tinha uma infinidade de algarismos, sem haver uma
>>> periodicidade.
>>> Na época foi o maior nó que tive com a matemática. O mestre demonstrou
>>> que os racionais eram densos, mas entre eles ainda cabiam os irracionais.
>>> Não satisfeito mostrou que os racionais eram enumeráveis e por absurdo
>>> mostrou que os |Reais não. Não satisfeito mostrou que a cardinalidade do
>>> intervalo [0,1] era maior que a dos |Naturais. Não conseguia conceber que
>>> havia um infinito maior que outro. Outra coisa que demorei a aceitar,mesmo
>>> vendo a bijeção, era que os inteiros e naturais tinham a mesma
>>> cardinalidade. Na minha cabeça, os inteiros têm todos os naturais ainda
>>> sobram os negativos, como é igual?
>>> Hoje, depois de velho, arrumei uma enteada, que muito me pergunta e
>>> estou enrolado. Para dar um ar de superioridade, questionei se conhecia os
>>> inteiros de Gaus, que 5 não era primo nos inteiros de Gaus. Estrepei-me, a
>>> danada foi pesquisar e me questiona sobre o que não tenho um domínio pleno.
>>> Em suma, como apresentei a ela os postulados de Peano para a
>>> caracterização dos Naturais, ela me cobra por algo semelhante para os
>>> Inteiros, e não sei responder.
>>> HELP! SOCORRO! AU SECOURS! AYUDA! AIUTO! HILFE!
>>> Cordialmente,
>>> PJMS
>>>
>>> --
>>> Esta mensagem foi verificada pelo sistema de antivírus e
>>> acredita-se estar livre de perigo.
>>
>>
>> --
>> Esta mensagem foi verificada pelo sistema de antivírus e
>> acredita-se estar livre de perigo.
>
>
> --
> Esta mensagem foi verificada pelo sistema de antivírus e
> acredita-se estar livre de perigo.

-- 
Esta mensagem foi verificada pelo sistema de antiv�rus e
 acredita-se estar livre de perigo.



[obm-l] Re: [obm-l] Re: [obm-l] Caracterização de Inteiros

2022-11-15 Por tôpico Pedro José
Obrigado a você e ao Cláudio. Mas não sou criativo para inventar. Mas já vi
que terei que fazer uma homotetia, para as classes de equivalência para
representar só como um número e não como um par, creio eu.

Cordialmente,
PJMS

Em ter., 15 de nov. de 2022 às 16:00, Anderson Torres <
torres.anderson...@gmail.com> escreveu:

>
>
> Em ter, 15 de nov de 2022 14:33, Pedro José 
> escreveu:
>
>> Boa tarde!
>> Para os |Naturais, temos os postulados de Peano.
>>
>> Para os Inteiros há alguma formalização?
>>
>
> invente uma!
>
> Pode ser por exemplo o conjunto de pares (p,q) tais que p-q é constante.
>
> ou melhor (p1,q1)=(p2,q2) se e só se p1+q2=p2+q1.
>
>
>> Acho pobre dizer que é necessário ter outros números devido ao problema
>> de fechamento nos naturais para a subtração que é fato e daí introduzir os
>> simétricos que são inteiros e ainda não foram caracterizados.
>>
>> No meu antigo ginásio aprendi que os Reais era a união dos conjuntos
>> disjuntos irracionais e racionais. Os racionais haviam sido bem definidos.
>> Aí questionei e o que são irracionais? resposta: são os Reais que não são
>> racionais, os que não podem ser escritos na forma p/q p e q inteiros e
>> q<>0. Mas me deram um tombo. Definiram os |Reais com base nos irracionais e
>> os irracionais com base nos |Reais. 3 +2i também não pode ser inscrito na
>> forma p/q. Só mais tarde no científico, é que meu professor definiu
>> irracional como um número que não podia ser escrito na forma p/q e cuja
>> representação decimal tinha uma infinidade de algarismos, sem haver uma
>> periodicidade.
>> Na época foi o maior nó que tive com a matemática. O mestre demonstrou
>> que os racionais eram densos, mas entre eles ainda cabiam os irracionais.
>> Não satisfeito mostrou que os racionais eram enumeráveis e por absurdo
>> mostrou que os |Reais não. Não satisfeito mostrou que a cardinalidade do
>> intervalo [0,1] era maior que a dos |Naturais. Não conseguia conceber que
>> havia um infinito maior que outro. Outra coisa que demorei a aceitar,mesmo
>> vendo a bijeção, era que os inteiros e naturais tinham a mesma
>> cardinalidade. Na minha cabeça, os inteiros têm todos os naturais ainda
>> sobram os negativos, como é igual?
>> Hoje, depois de velho, arrumei uma enteada, que muito me pergunta e estou
>> enrolado. Para dar um ar de superioridade, questionei se conhecia os
>> inteiros de Gaus, que 5 não era primo nos inteiros de Gaus. Estrepei-me, a
>> danada foi pesquisar e me questiona sobre o que não tenho um domínio pleno.
>> Em suma, como apresentei a ela os postulados de Peano para a
>> caracterização dos Naturais, ela me cobra por algo semelhante para os
>> Inteiros, e não sei responder.
>> HELP! SOCORRO! AU SECOURS! AYUDA! AIUTO! HILFE!
>> Cordialmente,
>> PJMS
>>
>> --
>> Esta mensagem foi verificada pelo sistema de antivírus e
>> acredita-se estar livre de perigo.
>
>
> --
> Esta mensagem foi verificada pelo sistema de antivírus e
> acredita-se estar livre de perigo.

-- 
Esta mensagem foi verificada pelo sistema de antiv�rus e
 acredita-se estar livre de perigo.



[obm-l] Re: [obm-l] Caracterização de Inteiros

2022-11-15 Por tôpico Anderson Torres
Em ter, 15 de nov de 2022 14:33, Pedro José  escreveu:

> Boa tarde!
> Para os |Naturais, temos os postulados de Peano.
>
> Para os Inteiros há alguma formalização?
>

invente uma!

Pode ser por exemplo o conjunto de pares (p,q) tais que p-q é constante.

ou melhor (p1,q1)=(p2,q2) se e só se p1+q2=p2+q1.


> Acho pobre dizer que é necessário ter outros números devido ao problema de
> fechamento nos naturais para a subtração que é fato e daí introduzir os
> simétricos que são inteiros e ainda não foram caracterizados.
>
> No meu antigo ginásio aprendi que os Reais era a união dos conjuntos
> disjuntos irracionais e racionais. Os racionais haviam sido bem definidos.
> Aí questionei e o que são irracionais? resposta: são os Reais que não são
> racionais, os que não podem ser escritos na forma p/q p e q inteiros e
> q<>0. Mas me deram um tombo. Definiram os |Reais com base nos irracionais e
> os irracionais com base nos |Reais. 3 +2i também não pode ser inscrito na
> forma p/q. Só mais tarde no científico, é que meu professor definiu
> irracional como um número que não podia ser escrito na forma p/q e cuja
> representação decimal tinha uma infinidade de algarismos, sem haver uma
> periodicidade.
> Na época foi o maior nó que tive com a matemática. O mestre demonstrou que
> os racionais eram densos, mas entre eles ainda cabiam os irracionais. Não
> satisfeito mostrou que os racionais eram enumeráveis e por absurdo mostrou
> que os |Reais não. Não satisfeito mostrou que a cardinalidade do intervalo
> [0,1] era maior que a dos |Naturais. Não conseguia conceber que havia um
> infinito maior que outro. Outra coisa que demorei a aceitar,mesmo vendo a
> bijeção, era que os inteiros e naturais tinham a mesma cardinalidade. Na
> minha cabeça, os inteiros têm todos os naturais ainda sobram os negativos,
> como é igual?
> Hoje, depois de velho, arrumei uma enteada, que muito me pergunta e estou
> enrolado. Para dar um ar de superioridade, questionei se conhecia os
> inteiros de Gaus, que 5 não era primo nos inteiros de Gaus. Estrepei-me, a
> danada foi pesquisar e me questiona sobre o que não tenho um domínio pleno.
> Em suma, como apresentei a ela os postulados de Peano para a
> caracterização dos Naturais, ela me cobra por algo semelhante para os
> Inteiros, e não sei responder.
> HELP! SOCORRO! AU SECOURS! AYUDA! AIUTO! HILFE!
> Cordialmente,
> PJMS
>
> --
> Esta mensagem foi verificada pelo sistema de antivírus e
> acredita-se estar livre de perigo.

-- 
Esta mensagem foi verificada pelo sistema de antiv�rus e
 acredita-se estar livre de perigo.



Re: [obm-l] Caracterização de Inteiros

2022-11-15 Por tôpico Claudio Buffara
A única que conheço e’ a que define uma relação de equivalência em pares 
ordenados de naturais (união {0}) dada por (a,b) ~ (c,d)  <==> a+d = b+c. Os 
inteiros são as classes de equivalência desta relação.

Enviado do meu iPhone

> Em 15 de nov. de 2022, à(s) 14:33, Pedro José  escreveu:
> 
> 
> Boa tarde!
> Para os |Naturais, temos os postulados de Peano.
> 
> Para os Inteiros há alguma formalização?
> 
> Acho pobre dizer que é necessário ter outros números devido ao problema de 
> fechamento nos naturais para a subtração que é fato e daí introduzir os 
> simétricos que são inteiros e ainda não foram caracterizados. 
> 
> No meu antigo ginásio aprendi que os Reais era a união dos conjuntos 
> disjuntos irracionais e racionais. Os racionais haviam sido bem definidos. 
> Aí questionei e o que são irracionais? resposta: são os Reais que não 
> são racionais, os que não podem ser escritos na forma p/q p e q inteiros e 
> q<>0. Mas me deram um tombo. Definiram os |Reais com base nos irracionais e 
> os irracionais com base nos |Reais. 3 +2i também não pode ser inscrito na 
> forma p/q. Só mais tarde no científico, é que meu professor definiu 
> irracional como um número que não podia ser escrito na forma p/q e cuja 
> representação decimal tinha uma infinidade de algarismos, sem haver uma 
> periodicidade.
> Na época foi o maior nó que tive com a matemática. O mestre demonstrou que 
> os racionais eram densos, mas entre eles ainda cabiam os irracionais. Não 
> satisfeito mostrou que os racionais eram enumeráveis e por absurdo mostrou 
> que os |Reais não. Não satisfeito mostrou que a cardinalidade do intervalo 
> [0,1] era maior que a dos |Naturais. Não conseguia conceber que havia um 
> infinito maior que outro. Outra coisa que demorei a aceitar,mesmo vendo a 
> bijeção, era que os inteiros e naturais tinham a mesma cardinalidade. Na 
> minha cabeça, os inteiros têm todos os naturais ainda sobram os negativos, 
> como é igual?
> Hoje, depois de velho, arrumei uma enteada, que muito me pergunta e estou 
> enrolado. Para dar um ar de superioridade, questionei se conhecia os inteiros 
> de Gaus, que 5 não era primo nos inteiros de Gaus. Estrepei-me, a danada foi 
> pesquisar e me questiona sobre o que não tenho um domínio pleno.
> Em suma, como apresentei a ela os postulados de Peano para a caracterização 
> dos Naturais, ela me cobra por algo semelhante para os Inteiros, e não sei 
> responder.
> HELP! SOCORRO! AU SECOURS! AYUDA! AIUTO! HILFE!
> Cordialmente,
> PJMS
> 
> -- 
> Esta mensagem foi verificada pelo sistema de antivírus e 
> acredita-se estar livre de perigo.

-- 
Esta mensagem foi verificada pelo sistema de antiv�rus e
 acredita-se estar livre de perigo.


=
Instru��es para entrar na lista, sair da lista e usar a lista em
http://www.mat.puc-rio.br/~obmlistas/obm-l.html
=


Re: [obm-l] problema de probabilidade

2022-11-09 Por tôpico Claudio Buffara
Essa também:
https://thedailyviz.com/2016/09/17/how-common-is-your-birthday-dailyviz/


On Wed, Nov 9, 2022 at 12:04 PM Claudio Buffara 
wrote:

> Achei isso aqui interessante: https://www.panix.com/~murphy/bday.html
>
> []s,
> Claudio.
>
> On Tue, Nov 8, 2022 at 9:56 PM Ralph Costa Teixeira 
> wrote:
>
>> Mis ou menos... O que faltou foi a hipótese exata da distribuição de
>> probabilidade dos aniversários.
>>
>> Se a gente supõe que cada mês tem os mesmos 1/12 de chance para cada
>> aluno, e que os meses são independentes entre si, sim,
>> p=12/12^2=1/12~8.3%.
>>
>> Agora, talvez um modelo um pouco mais preciso seria supor que cada DIA do
>> ano tem a mesma probabilidade (e que são independentes entre si). Isto
>> afeta um tiquinho a resposta, porque cada mes têm um número ligeiramente
>> diferente de dias! Ignorando anos bissextos (huh!?!), temos:
>> -- 7 meses com 31 dias;
>> -- 4 meses com 30 dias;
>> -- 1 mes com 28 dias;
>> Portanto, seria um pouco mais "realista" usar:
>> p=(7*31^2+4*30^2+28^2)/(365^2) ~ 8.34003%
>>
>> Eu ponho esse "realista" bem entre aspas; primeiro, porque eu ignorei
>> anos bissextos (fique à vontade para inclui-los e refazer a conta :D :D
>> :D); mas a hipótese de que todos os dias do ano tem a mesma probabilidade
>> não é tão realista quanto parece! Existe uma certa "concentração" de
>> aniversários em determinadas épocas do ano... mas, sem dados exatos sobre
>> como seja a tal concentração, o melhor que podemos fazer seria uma das
>> estimativas acima.
>>
>> Ainda tem um segundo problema sutil: *mesmo que todos os dias tivessem a
>> mesma probabilidade, talvez n*ã*o seja 100% correto supor que os
>> aniversários dos alunos da mesma turma do CMBel sejam independentes*!
>> Por exemplo, existe uma probabilidade maior que zero de ter gêmeos numa
>> mesma turma (comum uma família com gêmeos colocá-los na mesma escola), o
>> que afeta a independência dos dados, e muda um pouquinho aqueles 8.3% (para
>> cima)... sem uma estimativa desta probabilidade de ter gêmeos na mesma
>> turma, não conseguimos calcular a resposta "exata".
>>
>> Isto tudo dito... em quase qualquer problema de probabilidade a gente vai
>> ter que fazer ALGUMA hipótese simplificadora para poder sair do lugar.
>> Assim, eu diria que o problema não está 100% bem posto, mas não acho
>> ridículo fazer uma das hipóteses simplificadoras acima que levam a 8.3%
>> ou 8.34003% (e a diferença me parece tão pequena que eu aceitaria ambas as
>> respostas como corretas, desde que as hipóteses utilizadas em cada caso
>> fossem citadas).
>>
>> Abraço, Ralph.
>>
>> On Tue, Nov 8, 2022 at 3:07 PM Luis Paulo  wrote:
>>
>>> Prezados, o problema abaixo está bem posto?
>>>
>>> Uma turma do CMBel tem 25 alunos. Escolhendo-se aleatoriamente dois
>>> estudantes dessa turma, qual a probabilidade de eles façam aniversário no
>>> mesmo mês?
>>>
>>> A resposta da banca: 1/12.
>>>
>>>
>>>
>>> --
>>> Esta mensagem foi verificada pelo sistema de antivírus e
>>> acredita-se estar livre de perigo.
>>
>>
>> --
>> Esta mensagem foi verificada pelo sistema de antivírus e
>> acredita-se estar livre de perigo.
>
>

-- 
Esta mensagem foi verificada pelo sistema de antiv�rus e
 acredita-se estar livre de perigo.



Re: [obm-l] problema de probabilidade

2022-11-09 Por tôpico Claudio Buffara
Achei isso aqui interessante: https://www.panix.com/~murphy/bday.html

[]s,
Claudio.

On Tue, Nov 8, 2022 at 9:56 PM Ralph Costa Teixeira 
wrote:

> Mis ou menos... O que faltou foi a hipótese exata da distribuição de
> probabilidade dos aniversários.
>
> Se a gente supõe que cada mês tem os mesmos 1/12 de chance para cada
> aluno, e que os meses são independentes entre si, sim,
> p=12/12^2=1/12~8.3%.
>
> Agora, talvez um modelo um pouco mais preciso seria supor que cada DIA do
> ano tem a mesma probabilidade (e que são independentes entre si). Isto
> afeta um tiquinho a resposta, porque cada mes têm um número ligeiramente
> diferente de dias! Ignorando anos bissextos (huh!?!), temos:
> -- 7 meses com 31 dias;
> -- 4 meses com 30 dias;
> -- 1 mes com 28 dias;
> Portanto, seria um pouco mais "realista" usar:
> p=(7*31^2+4*30^2+28^2)/(365^2) ~ 8.34003%
>
> Eu ponho esse "realista" bem entre aspas; primeiro, porque eu ignorei
> anos bissextos (fique à vontade para inclui-los e refazer a conta :D :D
> :D); mas a hipótese de que todos os dias do ano tem a mesma probabilidade
> não é tão realista quanto parece! Existe uma certa "concentração" de
> aniversários em determinadas épocas do ano... mas, sem dados exatos sobre
> como seja a tal concentração, o melhor que podemos fazer seria uma das
> estimativas acima.
>
> Ainda tem um segundo problema sutil: *mesmo que todos os dias tivessem a
> mesma probabilidade, talvez n*ã*o seja 100% correto supor que os
> aniversários dos alunos da mesma turma do CMBel sejam independentes*! Por
> exemplo, existe uma probabilidade maior que zero de ter gêmeos numa mesma
> turma (comum uma família com gêmeos colocá-los na mesma escola), o que
> afeta a independência dos dados, e muda um pouquinho aqueles 8.3% (para
> cima)... sem uma estimativa desta probabilidade de ter gêmeos na mesma
> turma, não conseguimos calcular a resposta "exata".
>
> Isto tudo dito... em quase qualquer problema de probabilidade a gente vai
> ter que fazer ALGUMA hipótese simplificadora para poder sair do lugar.
> Assim, eu diria que o problema não está 100% bem posto, mas não acho
> ridículo fazer uma das hipóteses simplificadoras acima que levam a 8.3%
> ou 8.34003% (e a diferença me parece tão pequena que eu aceitaria ambas as
> respostas como corretas, desde que as hipóteses utilizadas em cada caso
> fossem citadas).
>
> Abraço, Ralph.
>
> On Tue, Nov 8, 2022 at 3:07 PM Luis Paulo  wrote:
>
>> Prezados, o problema abaixo está bem posto?
>>
>> Uma turma do CMBel tem 25 alunos. Escolhendo-se aleatoriamente dois
>> estudantes dessa turma, qual a probabilidade de eles façam aniversário no
>> mesmo mês?
>>
>> A resposta da banca: 1/12.
>>
>>
>>
>> --
>> Esta mensagem foi verificada pelo sistema de antivírus e
>> acredita-se estar livre de perigo.
>
>
> --
> Esta mensagem foi verificada pelo sistema de antivírus e
> acredita-se estar livre de perigo.

-- 
Esta mensagem foi verificada pelo sistema de antiv�rus e
 acredita-se estar livre de perigo.



Re: [obm-l] problema de probabilidade

2022-11-09 Por tôpico Anderson Torres
Em ter, 8 de nov de 2022 21:55, Ralph Costa Teixeira 
escreveu:

> Mis ou menos... O que faltou foi a hipótese exata da distribuição de
> probabilidade dos aniversários.
>
> Se a gente supõe que cada mês tem os mesmos 1/12 de chance para cada
> aluno, e que os meses são independentes entre si, sim,
> p=12/12^2=1/12~8.3%.
>
> Agora, talvez um modelo um pouco mais preciso seria supor que cada DIA do
> ano tem a mesma probabilidade (e que são independentes entre si). Isto
> afeta um tiquinho a resposta, porque cada mes têm um número ligeiramente
> diferente de dias! Ignorando anos bissextos (huh!?!), temos:
> -- 7 meses com 31 dias;
> -- 4 meses com 30 dias;
> -- 1 mes com 28 dias;
> Portanto, seria um pouco mais "realista" usar:
> p=(7*31^2+4*30^2+28^2)/(365^2) ~ 8.34003%
>
> Eu ponho esse "realista" bem entre aspas; primeiro, porque eu ignorei
> anos bissextos (fique à vontade para inclui-los e refazer a conta :D :D
> :D); mas a hipótese de que todos os dias do ano tem a mesma probabilidade
> não é tão realista quanto parece! Existe uma certa "concentração" de
> aniversários em determinadas épocas do ano... mas, sem dados exatos sobre
> como seja a tal concentração, o melhor que podemos fazer seria uma das
> estimativas acima.
>

Em uma turma com tão pouca gente, eu acho que considerações como "a
concentração de pessoas concebidas no Carnaval" podem ser ignoradas para um
problema tão simples. E, pelo que se nota, a conta mais limpa dá uma
diferença minúscula, 0,01%. Desconheço aplicação tão precisa na prática.


> Ainda tem um segundo problema sutil: *mesmo que todos os dias tivessem a
> mesma probabilidade, talvez n*ã*o seja 100% correto supor que os
> aniversários dos alunos da mesma turma do CMBel sejam independentes*! Por
> exemplo, existe uma probabilidade maior que zero de ter gêmeos numa mesma
> turma (comum uma família com gêmeos colocá-los na mesma escola), o que
> afeta a independência dos dados, e muda um pouquinho aqueles 8.3% (para
> cima)... sem uma estimativa desta probabilidade de ter gêmeos na mesma
> turma, não conseguimos calcular a resposta "exata".
>
> Isto tudo dito... em quase qualquer problema de probabilidade a gente vai
> ter que fazer ALGUMA hipótese simplificadora para poder sair do lugar.
> Assim, eu diria que o problema não está 100% bem posto, mas não acho
> ridículo fazer uma das hipóteses simplificadoras acima que levam a 8.3%
> ou 8.34003% (e a diferença me parece tão pequena que eu aceitaria ambas as
> respostas como corretas, desde que as hipóteses utilizadas em cada caso
> fossem citadas).
>
> Abraço, Ralph.
>
> On Tue, Nov 8, 2022 at 3:07 PM Luis Paulo  wrote:
>
>> Prezados, o problema abaixo está bem posto?
>>
>> Uma turma do CMBel tem 25 alunos. Escolhendo-se aleatoriamente dois
>> estudantes dessa turma, qual a probabilidade de eles façam aniversário no
>> mesmo mês?
>>
>> A resposta da banca: 1/12.
>>
>>
>>
>> --
>> Esta mensagem foi verificada pelo sistema de antivírus e
>> acredita-se estar livre de perigo.
>
>
> --
> Esta mensagem foi verificada pelo sistema de antivírus e
> acredita-se estar livre de perigo.

-- 
Esta mensagem foi verificada pelo sistema de antiv�rus e
 acredita-se estar livre de perigo.



Re: [obm-l] problema de probabilidade

2022-11-08 Por tôpico Ralph Costa Teixeira
Mis ou menos... O que faltou foi a hipótese exata da distribuição de
probabilidade dos aniversários.

Se a gente supõe que cada mês tem os mesmos 1/12 de chance para cada aluno,
e que os meses são independentes entre si, sim, p=12/12^2=1/12~8.3%.

Agora, talvez um modelo um pouco mais preciso seria supor que cada DIA do
ano tem a mesma probabilidade (e que são independentes entre si). Isto
afeta um tiquinho a resposta, porque cada mes têm um número ligeiramente
diferente de dias! Ignorando anos bissextos (huh!?!), temos:
-- 7 meses com 31 dias;
-- 4 meses com 30 dias;
-- 1 mes com 28 dias;
Portanto, seria um pouco mais "realista" usar:
p=(7*31^2+4*30^2+28^2)/(365^2) ~ 8.34003%

Eu ponho esse "realista" bem entre aspas; primeiro, porque eu ignorei
anos bissextos (fique à vontade para inclui-los e refazer a conta :D :D
:D); mas a hipótese de que todos os dias do ano tem a mesma probabilidade
não é tão realista quanto parece! Existe uma certa "concentração" de
aniversários em determinadas épocas do ano... mas, sem dados exatos sobre
como seja a tal concentração, o melhor que podemos fazer seria uma das
estimativas acima.

Ainda tem um segundo problema sutil: *mesmo que todos os dias tivessem a
mesma probabilidade, talvez n*ã*o seja 100% correto supor que os
aniversários dos alunos da mesma turma do CMBel sejam independentes*! Por
exemplo, existe uma probabilidade maior que zero de ter gêmeos numa mesma
turma (comum uma família com gêmeos colocá-los na mesma escola), o que
afeta a independência dos dados, e muda um pouquinho aqueles 8.3% (para
cima)... sem uma estimativa desta probabilidade de ter gêmeos na mesma
turma, não conseguimos calcular a resposta "exata".

Isto tudo dito... em quase qualquer problema de probabilidade a gente vai
ter que fazer ALGUMA hipótese simplificadora para poder sair do lugar.
Assim, eu diria que o problema não está 100% bem posto, mas não acho
ridículo fazer uma das hipóteses simplificadoras acima que levam a 8.3%
ou 8.34003% (e a diferença me parece tão pequena que eu aceitaria ambas as
respostas como corretas, desde que as hipóteses utilizadas em cada caso
fossem citadas).

Abraço, Ralph.

On Tue, Nov 8, 2022 at 3:07 PM Luis Paulo  wrote:

> Prezados, o problema abaixo está bem posto?
>
> Uma turma do CMBel tem 25 alunos. Escolhendo-se aleatoriamente dois
> estudantes dessa turma, qual a probabilidade de eles façam aniversário no
> mesmo mês?
>
> A resposta da banca: 1/12.
>
>
>
> --
> Esta mensagem foi verificada pelo sistema de antivírus e
> acredita-se estar livre de perigo.

-- 
Esta mensagem foi verificada pelo sistema de antiv�rus e
 acredita-se estar livre de perigo.



Re: [obm-l] integrais

2022-10-05 Por tôpico carlos h Souza
Desculpe -me ao colar o texto do bloco de notas, acabei verificando que
havia uma imagem. Vou refazer a pergunta ..
 Obrigado

Em qua, 5 de out de 2022 16:24, Anderson Torres <
torres.anderson...@gmail.com> escreveu:

> É spam??
>
> Em ter, 4 de out de 2022 15:48, carlos h Souza 
> escreveu:
>
>>
> --
> Esta mensagem foi verificada pelo sistema de antivírus e
> acredita-se estar livre de perigo.

-- 
Esta mensagem foi verificada pelo sistema de antiv�rus e
 acredita-se estar livre de perigo.



Re: [obm-l] integrais

2022-10-05 Por tôpico Anderson Torres
É spam??

Em ter, 4 de out de 2022 15:48, carlos h Souza 
escreveu:

>

-- 
Esta mensagem foi verificada pelo sistema de antiv�rus e
 acredita-se estar livre de perigo.



[obm-l] Re: [obm-l] Sugestões para Iniciação Criança

2022-09-14 Por tôpico Tiago Machado
Não sei como andam as atualizações, mas talvez a revista Eureka seja um dos
bons lugares para começar
https://www.obm.org.br/revista-eureka/

On Wed, Sep 14, 2022 at 10:49 AM Esaú Gomes  wrote:

> Olá pessoal, tenho um filho de 10 anos (quinto ano) e ele curte
> matemática. Gostaria de iniciá-lo em material referente às olimpíadas para
> ver se ele pega gosto. Alguém indica material para fazer sua iniciação?
>
> Desde já, obrigado.
>
> --
> Esta mensagem foi verificada pelo sistema de antivírus e
> acredita-se estar livre de perigo.

-- 
Esta mensagem foi verificada pelo sistema de antiv�rus e
 acredita-se estar livre de perigo.



[obm-l] Re: [obm-l] Perguntas mínimas no Tabuleiro

2022-09-14 Por tôpico Anderson Torres
Em ter, 13 de set de 2022 22:59, Jeferson Almir 
escreveu:

> Os números de 1 a 49 são arbitrariamente dispostos num tabuleiro quadrado
> 7x7 . Podemos escolher qualquer quadrado composto de múltiplas células e
> perguntar quais números estão contidos nele. Ao menos quantas perguntas são
> necessárias para determinarmos a configuração exata dos números?
>
> Alguém tem uma ideia ótima ??
>

tenho não, mas para o caso 3x3 a resposta é no máximo 4. Perguntando quem
são os elementos dos quadradinhos 2x2, é possível obter quem são os 9 caras
usando uniões e intersecções.

com isso já podemos recursivamente cortar o quadrado 7x7 em quadrados 3x3.
Se fosse 9x9, isso daria no máximo 24.

vou pensar mais em casa.


>
> --
> Esta mensagem foi verificada pelo sistema de antivírus e
> acredita-se estar livre de perigo.

-- 
Esta mensagem foi verificada pelo sistema de antiv�rus e
 acredita-se estar livre de perigo.



[obm-l] Re: [obm-l] Séries

2022-08-25 Por tôpico Esdras Muniz
Me manda.

Em qui, 25 de ago de 2022 17:36, Israel Meireles Chrisostomo <
israelmchrisost...@gmail.com> escreveu:

> Olá pessoal, recentemente eu tive umas ideias sobre séries envolvendo o
> número e (napier), o seno e o cosseno.Alguém por favor poderia me
> corrigir?São ideias originais e séries infinitas nunca antes pensadas.
> Alguém por favor me ajuda a corrigir.Ver se estou viajandoMeu desejo é
> que vcs digam que esteja certo, sejam pacientes por favor.Quem se dispor,
> por favor, chama inbox
>
> -
> Somente a Deus Glória.
> Israel Meireles Chrisostomo
>
> --
> Esta mensagem foi verificada pelo sistema de antivírus e
> acredita-se estar livre de perigo.

-- 
Esta mensagem foi verificada pelo sistema de antiv�rus e
 acredita-se estar livre de perigo.



[obm-l] Re: [obm-l] Re: [obm-l] Fwd: Módulo

2022-08-14 Por tôpico Rubens Vilhena Fonseca
Uma coisa que você deve definir é a paridade de n. Vamos reescrever em
linguagem de congruências :
2^n==1 (mod 3). Sabendo que 2== -1 (mod 3), então  (-1)^n == 1 (mod 3). O
que só será verdade se n for par.
Então, para n = 2k, temos 4^k = 3x +1.  Por experimentação, você pode
concluir alguns pares (k, x) de solução, (0, 0);
(1, 1); (2, 5); (3, 21)...Então, seu trabalho é mostrar que o par (k,
(4^k-1)/3 ) é uma solução.

Em qui., 11 de ago. de 2022 às 17:38, Anderson Torres <
torres.anderson...@gmail.com> escreveu:

>
>
> Em qui, 11 de ago de 2022 16:12, Esaú Gomes 
> escreveu:
>
>> Alguém poderia me falar o que estudar mais especificamente na questão
>> abaixo?
>>
>> Para quais valores naturais  de *n* e *x*, existe solução
>> 2^n = 3x + 1.
>>
>
>   Provas antigas.
>
> Esses problemas são resolvidos geralmente apelando para fatos padrão de
> congruências, em especial potenciação, ordem etc.
>
> E, no geral, a melhor maneira de entender e aplicar estes fatos é mediante
> treino, treino e mais treino.
>
> --
>> Esta mensagem foi verificada pelo sistema de antivírus e
>> acredita-se estar livre de perigo.
>
>
> --
> Esta mensagem foi verificada pelo sistema de antivírus e
> acredita-se estar livre de perigo.

-- 
Esta mensagem foi verificada pelo sistema de antiv�rus e
 acredita-se estar livre de perigo.



[obm-l] Re: [obm-l] Fwd: Módulo

2022-08-11 Por tôpico Anderson Torres
Em qui, 11 de ago de 2022 16:12, Esaú Gomes  escreveu:

> Alguém poderia me falar o que estudar mais especificamente na questão
> abaixo?
>
> Para quais valores naturais  de *n* e *x*, existe solução
> 2^n = 3x + 1.
>

  Provas antigas.

Esses problemas são resolvidos geralmente apelando para fatos padrão de
congruências, em especial potenciação, ordem etc.

E, no geral, a melhor maneira de entender e aplicar estes fatos é mediante
treino, treino e mais treino.

-- 
> Esta mensagem foi verificada pelo sistema de antivírus e
> acredita-se estar livre de perigo.

-- 
Esta mensagem foi verificada pelo sistema de antiv�rus e
 acredita-se estar livre de perigo.



[obm-l] Re: [obm-l] Fwd: Módulo

2022-08-11 Por tôpico Carlos Gomes
Ola amigo. Normalmente essas equações diofantinas nao lineares tem solução
passando por congruência.



Em qui., 11 de ago. de 2022 16:11, Esaú Gomes 
escreveu:

> Alguém poderia me falar o que estudar mais especificamente na questão
> abaixo?
>
> Para quais valores naturais  de *n* e *x*, existe solução
> 2^n = 3x + 1.
>
> --
> Esta mensagem foi verificada pelo sistema de antivírus e
> acredita-se estar livre de perigo.

-- 
Esta mensagem foi verificada pelo sistema de antiv�rus e
 acredita-se estar livre de perigo.



[obm-l] Re: [obm-l] Provar que a inteira f é um polinômio de grau positivo

2022-07-15 Por tôpico Artur Costa Steiner
Obrigado, abordagem bem interessante

Eu dei a seguinte prova:

Para z em C/{0}, seja g(z) = f(1/z), obtendo-se uma função holomorfa tal
que lim z —> 0 g(z) = lim z—> oo f(z) = oo. Assim, g é meromorfa em C,
tendo em 0 seu único polo. Sendo n > 0 a ordem deste polo, g é expandida em
C/{0} por uma série de Laurent em torno de 0, havendo portanto complexos
c(-n), … c(0), c(1) ….tais que

g(z) = Soma (k = -n, oo) c(k) z^k, z em C/{0}

Para z em  C/{0} temos então que

f(z) = g(1/z) =Soma (k = n, -oo c(k) z^(k) (1)

Em (1), temos a série de Laurent que, em C/{0}, expande f e em torno de 0.
Se nesta série houvesse algum coeficiente não nulo associado a potência
negativa de z, f apresentaria uma singularidade em 0. Mas sendo uma função
 inteira, f não apresenta nenhuma singularidade em C, do que deduzimos que,
em (1), todos os coeficientes associados a potências negativas de z são
nulos. Logo, em C/{0} f é o polinômio de grau n dado por

f(z) = c(-n) z^n + ….  c(1) z + c(0) (2)

Como f é contínua, temos que f(0) = lim z —> 0 f(z) = c(0), o que mostra
que (2) vale em todo o C. Logo, f é em C um polinômio de grau positivo.

Abs
Artur

Em qui., 14 de jul. de 2022 às 19:23, Claudio Buffara <
claudio.buff...@gmail.com> escreveu:

> Use o fato de que toda função meromorfica  em C união {inf} é da forma
> f(z)/g(z), onde f, g são polinômios.
> Daí, como a função do enunciado é inteira, g(z) é constante (e não nula).
> E como f(z) rende a inf quando z tende a inf, f é um polinômio não
> constante.
>
> Enviado do meu iPhone
>
> > Em 14 de jul. de 2022, à(s) 16:41, Artur Costa Steiner <
> artur.costa.stei...@gmail.com> escreveu:
> >
> > Oi amigos!
> >
> > Um teorema da Análise Complexa diz que, se f é inteira e lim z —> oo
> f(z) = oo, então f é um polinômio (claramente não constante). Nos
> livros em que estudei isso era dado como exercício, de modo que nunca vi a
> demonstração deste teorema. Eu consegui dar duas demonstrações para
> ele, sendo que uma delas sei que está certa A outra acho que também está
> certa, mas a primeira me parece bem melhor.Â
> >
> > Alguém aqui pode dar uma prova, para comparar com a minha? Se houver
> interesse (Análise Complexa não costuma aparecer aqui) eu dou as minhas.Â
> >
> > Obrigado
> >
> > Artur
> >
> >
> >
> > --
> > Esta mensagem foi verificada pelo sistema de antivírus e
> > acredita-se estar livre de perigo.
>
> --
> Esta mensagem foi verificada pelo sistema de antivírus e
>  acredita-se estar livre de perigo.
>
>
> =
> Instru�ões para entrar na lista, sair da lista e usar a lista em
> http://www.mat.puc-rio.br/~obmlistas/obm-l.html
> =
>

-- 
Esta mensagem foi verificada pelo sistema de antiv�rus e
 acredita-se estar livre de perigo.



[obm-l] Re: [obm-l] Função phi de Euler

2022-07-15 Por tôpico Anderson Torres
Em qui, 14 de jul de 2022 11:52, Rubens Vilhena Fonseca <
rubens.vilhen...@gmail.com> escreveu:

> Saudações a todos da lista.
> É um fato que para primos p ímpares, a função de Euler phi(p)=p-1 é sempre
> um valor par.
> Os primos 7, 13, 19, 31, 37, 67, 73, 79, 97, ... tem valores pares
> múltiplos de 3.
> Existe algum caminho a tomar para determinar quando phi(p) = 3 .(2k)?
>

quem é esse k?

Agradeço qualquer solução ou  informação ou indicação de leituras sobre o
> problema.
> Att
>
>
> --
> Esta mensagem foi verificada pelo sistema de antivírus e
> acredita-se estar livre de perigo.

-- 
Esta mensagem foi verificada pelo sistema de antiv�rus e
 acredita-se estar livre de perigo.



[obm-l] Re: [obm-l] Re: [obm-l] Função phi de Euler

2022-07-15 Por tôpico Anderson Torres
Em qui, 14 de jul de 2022 12:19, Esdras Muniz 
escreveu:

> Quis dizer φ(p)=p-1.
>
> Em qui, 14 de jul de 2022 12:02, Esdras Muniz 
> escreveu:
>
>> Oi(o)=p-1, aí isso só vale se o primo for da firma 6k+1.
>>
>
phi(4+3)=7-1


>> Em qui, 14 de jul de 2022 11:52, Rubens Vilhena Fonseca <
>> rubens.vilhen...@gmail.com> escreveu:
>>
>>> Saudações a todos da lista.
>>> É um fato que para primos p ímpares, a função de Euler phi(p)=p-1 é
>>> sempre um valor par.
>>> Os primos 7, 13, 19, 31, 37, 67, 73, 79, 97, ... tem valores pares
>>> múltiplos de 3.
>>> Existe algum caminho a tomar para determinar quando phi(p) = 3 .(2k)?
>>> Agradeço qualquer solução ou  informação ou indicação de leituras sobre
>>> o  problema.
>>> Att
>>>
>>>
>>> --
>>> Esta mensagem foi verificada pelo sistema de antivírus e
>>> acredita-se estar livre de perigo.
>>
>>
> --
> Esta mensagem foi verificada pelo sistema de antivírus e
> acredita-se estar livre de perigo.

-- 
Esta mensagem foi verificada pelo sistema de antiv�rus e
 acredita-se estar livre de perigo.



Re: [obm-l] Provar que a inteira f é um polinômio de grau positivo

2022-07-14 Por tôpico Claudio Buffara
Use o fato de que toda função meromorfica  em C união {inf} é da forma 
f(z)/g(z), onde f, g são polinômios.
Daí, como a função do enunciado é inteira, g(z) é constante (e não nula).
E como f(z) rende a inf quando z tende a inf, f é um polinômio não constante.

Enviado do meu iPhone

> Em 14 de jul. de 2022, à(s) 16:41, Artur Costa Steiner 
>  escreveu:
> 
> Oi amigos!
> 
> Um teorema da Análise Complexa diz que, se f é inteira e lim z —> oo f(z) 
> = oo, então f é um polinômio (claramente não constante). Nos livros em 
> que estudei isso era dado como exercício, de modo que nunca vi a 
> demonstração deste teorema. Eu consegui dar duas demonstrações para ele, 
> sendo que uma delas sei que está certa A outra acho que também está certa, 
> mas a primeira me parece bem melhor. 
> 
> Alguém aqui pode dar uma prova, para comparar com a minha? Se houver 
> interesse (Análise Complexa não costuma aparecer aqui) eu dou as minhas. 
> 
> Obrigado
> 
> Artur
> 
> 
> 
> -- 
> Esta mensagem foi verificada pelo sistema de antivírus e 
> acredita-se estar livre de perigo.

-- 
Esta mensagem foi verificada pelo sistema de antiv�rus e
 acredita-se estar livre de perigo.


=
Instru��es para entrar na lista, sair da lista e usar a lista em
http://www.mat.puc-rio.br/~obmlistas/obm-l.html
=


[obm-l] Re: [obm-l] Função phi de Euler

2022-07-14 Por tôpico Esdras Muniz
Quis dizer φ(p)=p-1.

Em qui, 14 de jul de 2022 12:02, Esdras Muniz 
escreveu:

> Oi(o)=p-1, aí isso só vale se o primo for da firma 6k+1.
>
> Em qui, 14 de jul de 2022 11:52, Rubens Vilhena Fonseca <
> rubens.vilhen...@gmail.com> escreveu:
>
>> Saudações a todos da lista.
>> É um fato que para primos p ímpares, a função de Euler phi(p)=p-1 é
>> sempre um valor par.
>> Os primos 7, 13, 19, 31, 37, 67, 73, 79, 97, ... tem valores pares
>> múltiplos de 3.
>> Existe algum caminho a tomar para determinar quando phi(p) = 3 .(2k)?
>> Agradeço qualquer solução ou  informação ou indicação de leituras sobre
>> o  problema.
>> Att
>>
>>
>> --
>> Esta mensagem foi verificada pelo sistema de antivírus e
>> acredita-se estar livre de perigo.
>
>

-- 
Esta mensagem foi verificada pelo sistema de antiv�rus e
 acredita-se estar livre de perigo.



[obm-l] Re: [obm-l] Função phi de Euler

2022-07-14 Por tôpico Esdras Muniz
Oi(o)=p-1, aí isso só vale se o primo for da firma 6k+1.

Em qui, 14 de jul de 2022 11:52, Rubens Vilhena Fonseca <
rubens.vilhen...@gmail.com> escreveu:

> Saudações a todos da lista.
> É um fato que para primos p ímpares, a função de Euler phi(p)=p-1 é sempre
> um valor par.
> Os primos 7, 13, 19, 31, 37, 67, 73, 79, 97, ... tem valores pares
> múltiplos de 3.
> Existe algum caminho a tomar para determinar quando phi(p) = 3 .(2k)?
> Agradeço qualquer solução ou  informação ou indicação de leituras sobre o
> problema.
> Att
>
>
> --
> Esta mensagem foi verificada pelo sistema de antivírus e
> acredita-se estar livre de perigo.

-- 
Esta mensagem foi verificada pelo sistema de antiv�rus e
 acredita-se estar livre de perigo.



Re: [obm-l] Ajuda em Repunits

2022-07-10 Por tôpico Rubens Vilhena Fonseca
Muito interessante, não faço a mínima ideia de como fazer, mas como você
disse vou me divertir pesquisando. Não sei se tem alguma coisa a ver mas,
se dividir o período desses exemplos ao "meio"  e somar (1/11 deu essa
ideia) o resultado parecem ser 9's. Outra coisa que percebi é que a ordem
desses denominadores módulo 10 é igual ao tamanho do período ( de novo 1/11
deu essa ideia). E como alguns são raízes primitivas de 10 o período é o
maior possível...
Com certeza se for verdade, são fatos já provados, vou tentar encontrar as
fontes.
Obrigado pela atenção
[[ ]]'s


Em dom., 10 de jul. de 2022 às 16:38, Claudio Buffara <
claudio.buff...@gmail.com> escreveu:

> Se quiser se divertir mais com isso, veja o seguinte:
> 1/7 = 0,142857142857142...
> O período é 142 857 e 1+8 = 4+5 = 2+7 = 9.
>
> 1/11: o período é 09 e 0+9 = 9.
>
> 1/13: o período é 076 923 e 0+9 = 7+2 = 6+3 = 9.
>
> Determine, com demonstração, para quais números N, o período de 1/N tem
> esta propriedade.
>
>
>
>
> On Sun, Jul 10, 2022 at 8:41 AM Rubens Vilhena Fonseca <
> rubens.vilhen...@gmail.com> wrote:
>
>> Muito obrigado ao Ralph Costa Teixeira e ao Claudio Buffara por todos os
>> ótimos esclarecimentos.
>> [[ ]]'s
>>
>> Em dom., 10 de jul. de 2022 às 01:39, Ralph Costa Teixeira <
>> ralp...@gmail.com> escreveu:
>>
>>> Argh, corrigindo um detalhe ali perto do fim:
>>> -- Sabemos que 10^q*B-B=r/10^w, portanto 9*(111...)**x**10^w = r*n.
>>> Novamente, como n é primo com 2, 3 e 5 *e x*, conclui-se que n divide
>>> 111 (com q 1's), e portanto q>=p=k.
>>>
>>> On Sun, Jul 10, 2022 at 1:24 AM Ralph Costa Teixeira 
>>> wrote:
>>>
 A chave: *os "restos parciais" que aparecem são exatamente os restos
 que x, 10x, 100x,  deixam na divisão por n.*
 ---///---

 MAIS SPOILERS ABAIXO


 ...


 


 ...


 

 Acho que facilita bastante pensar no "período" de 1/n de outro jeito:
 ---///---
 LEMA:
 (i) Dado n não divisível por 2 ou 5, existe algum número da forma
 111...111 que é múltiplo de n.
 (ii) Se n não for divisível por 2, 3 ou 5, o *menor* número do tipo
 111...111 que é múltiplo de n tem k dígitos, onde k é exatamente o tamanho
 do período (fundamental) da dízima em 1/n.
 PROVA:

 (i) Olhe os restos de 1, 11, 111, , ... na divisão por n. São n
 possibilidades, de 0 a n-1, então alguma hora algum resto tem que repetir.
 Isto significa que .. (com A dígitos) e 11...111 (com B dígitos,
 B>>> ...1110 (A 1's e B 0's) = 111 * (10^B) é múltiplo de n. Mas
 n não tem fator comum com aquele 10^B (pois não é divisível por 2 nem por
 5), portanto ...111 (com k=A-B dígitos) é divisível por n.

 (ii) Denote por P=111111 (com p dígitos) o menor daqueles caras com
 apenas "1s" que é múltiplo de n, e denote por k o "período fundamental" na
 dízima de 1/n.
 Por um lado, como 9P=999=10^p-1 é múltiplo de n, temos 10^p *
 (1/n) - 1/n inteiro. Mas isso significa que a parte decimal de 1/n "se
 repete" de p em p dígitos, ou seja, que a dízima de 1/n tem período p. Em
 particular, p>=k.
 Por outro lado, sendo k o período fundamental, temos 10^k * (1/n) -
 (1/n) com número finito de casas decimais, ou seja, (10^k-1)/n = m/10^z com
 m inteiro, e z=número de casas decimais que "sobraram". Mas daqui vem
 9*(111...111)*10^z = m*n (com k dígitos 1s). Como n é primo com 2, 3 e 5,
 conclui-se que 111... (k 1's) tem que ser múltiplo de n, e portanto
 k>=p.

 Note um efeito colateral disso tudo: provamos que 10^k*(1/n)- 1/n =
 10^p*(1/n)-1/n = inteiro. Assim aquele z vale 0, ou seja, não tem "casas
 decimais que sobram" -- a dízima periódica do 1/n se inicia logo no
 primeiro dígito!

 ---///---
 Agora fica tudo bem simples:
 a) Na notação acima, provamos que k=p, e n divide 111 com p
 dígitos.
 b) Seja q o período (fundamental) da dízima de B=x/n irredutível.

 Em primeiro lugar, provemos que q=k. Basicamente repetimos o que
 fizemos no lema:
 -- Sabemos que 10^q*B-B=r/10^w, portanto 9*(111...)*10^w = r*n.
 Novamente, como n é primo com 2, 3 e 5, conclui-se que n divide 111
 (com q 1's), e portanto q>=p=k.
 -- Por outro lado, como (10^k-1)/n é inteiro, (10^k-1)*x/n=10^k*B-B
 também é inteiro, ou seja, a dízima de B tem período k (e se inicia no
 primeiro dígito!). Portanto k>=q.

 *Enfim, note que os tais "restos parciais" que aparecem são exatamente
 os restos que x, 10x, 100x, , 10^q.x deixam na divisão por n. *A
 soma desses caras vale (...)*x, que é divisível por n pois temos
 ali q=k=p dígitos 1. Por isso, ao dividir esses restos parciais por n, a
 soma dos novos restos tem que ser múltiplo de n tambem.

 Foi?


 On Sat, Jul 9, 2022 at 7:16 PM Rubens Vilhena 

Re: [obm-l] Ajuda em Repunits

2022-07-10 Por tôpico Claudio Buffara
Se quiser se divertir mais com isso, veja o seguinte:
1/7 = 0,142857142857142...
O período é 142 857 e 1+8 = 4+5 = 2+7 = 9.

1/11: o período é 09 e 0+9 = 9.

1/13: o período é 076 923 e 0+9 = 7+2 = 6+3 = 9.

Determine, com demonstração, para quais números N, o período de 1/N tem
esta propriedade.




On Sun, Jul 10, 2022 at 8:41 AM Rubens Vilhena Fonseca <
rubens.vilhen...@gmail.com> wrote:

> Muito obrigado ao Ralph Costa Teixeira e ao Claudio Buffara por todos os
> ótimos esclarecimentos.
> [[ ]]'s
>
> Em dom., 10 de jul. de 2022 às 01:39, Ralph Costa Teixeira <
> ralp...@gmail.com> escreveu:
>
>> Argh, corrigindo um detalhe ali perto do fim:
>> -- Sabemos que 10^q*B-B=r/10^w, portanto 9*(111...)**x**10^w = r*n.
>> Novamente, como n é primo com 2, 3 e 5 *e x*, conclui-se que n divide
>> 111 (com q 1's), e portanto q>=p=k.
>>
>> On Sun, Jul 10, 2022 at 1:24 AM Ralph Costa Teixeira 
>> wrote:
>>
>>> A chave: *os "restos parciais" que aparecem são exatamente os restos
>>> que x, 10x, 100x,  deixam na divisão por n.*
>>> ---///---
>>>
>>> MAIS SPOILERS ABAIXO
>>>
>>>
>>> ...
>>>
>>>
>>> 
>>>
>>>
>>> ...
>>>
>>>
>>> 
>>>
>>> Acho que facilita bastante pensar no "período" de 1/n de outro jeito:
>>> ---///---
>>> LEMA:
>>> (i) Dado n não divisível por 2 ou 5, existe algum número da forma
>>> 111...111 que é múltiplo de n.
>>> (ii) Se n não for divisível por 2, 3 ou 5, o *menor* número do tipo
>>> 111...111 que é múltiplo de n tem k dígitos, onde k é exatamente o tamanho
>>> do período (fundamental) da dízima em 1/n.
>>> PROVA:
>>>
>>> (i) Olhe os restos de 1, 11, 111, , ... na divisão por n. São n
>>> possibilidades, de 0 a n-1, então alguma hora algum resto tem que repetir.
>>> Isto significa que .. (com A dígitos) e 11...111 (com B dígitos,
>>> B>> ...1110 (A 1's e B 0's) = 111 * (10^B) é múltiplo de n. Mas
>>> n não tem fator comum com aquele 10^B (pois não é divisível por 2 nem por
>>> 5), portanto ...111 (com k=A-B dígitos) é divisível por n.
>>>
>>> (ii) Denote por P=111111 (com p dígitos) o menor daqueles caras com
>>> apenas "1s" que é múltiplo de n, e denote por k o "período fundamental" na
>>> dízima de 1/n.
>>> Por um lado, como 9P=999=10^p-1 é múltiplo de n, temos 10^p *
>>> (1/n) - 1/n inteiro. Mas isso significa que a parte decimal de 1/n "se
>>> repete" de p em p dígitos, ou seja, que a dízima de 1/n tem período p. Em
>>> particular, p>=k.
>>> Por outro lado, sendo k o período fundamental, temos 10^k * (1/n) -
>>> (1/n) com número finito de casas decimais, ou seja, (10^k-1)/n = m/10^z com
>>> m inteiro, e z=número de casas decimais que "sobraram". Mas daqui vem
>>> 9*(111...111)*10^z = m*n (com k dígitos 1s). Como n é primo com 2, 3 e 5,
>>> conclui-se que 111... (k 1's) tem que ser múltiplo de n, e portanto
>>> k>=p.
>>>
>>> Note um efeito colateral disso tudo: provamos que 10^k*(1/n)- 1/n =
>>> 10^p*(1/n)-1/n = inteiro. Assim aquele z vale 0, ou seja, não tem "casas
>>> decimais que sobram" -- a dízima periódica do 1/n se inicia logo no
>>> primeiro dígito!
>>>
>>> ---///---
>>> Agora fica tudo bem simples:
>>> a) Na notação acima, provamos que k=p, e n divide 111 com p
>>> dígitos.
>>> b) Seja q o período (fundamental) da dízima de B=x/n irredutível.
>>>
>>> Em primeiro lugar, provemos que q=k. Basicamente repetimos o que fizemos
>>> no lema:
>>> -- Sabemos que 10^q*B-B=r/10^w, portanto 9*(111...)*10^w = r*n.
>>> Novamente, como n é primo com 2, 3 e 5, conclui-se que n divide 111
>>> (com q 1's), e portanto q>=p=k.
>>> -- Por outro lado, como (10^k-1)/n é inteiro, (10^k-1)*x/n=10^k*B-B
>>> também é inteiro, ou seja, a dízima de B tem período k (e se inicia no
>>> primeiro dígito!). Portanto k>=q.
>>>
>>> *Enfim, note que os tais "restos parciais" que aparecem são exatamente
>>> os restos que x, 10x, 100x, , 10^q.x deixam na divisão por n. *A
>>> soma desses caras vale (...)*x, que é divisível por n pois temos
>>> ali q=k=p dígitos 1. Por isso, ao dividir esses restos parciais por n, a
>>> soma dos novos restos tem que ser múltiplo de n tambem.
>>>
>>> Foi?
>>>
>>>
>>> On Sat, Jul 9, 2022 at 7:16 PM Rubens Vilhena Fonseca <
>>> rubens.vilhen...@gmail.com> wrote:
>>>
 Gostaria de uma demonstração para o seguinte teorema.
 *Teorema*. Seja n um inteiro positivo não divisível por 2, 3 ou 5, e
 suponha que a expansão decimal de l/n tenha período k. Então n é um fator
 do inteiro 111 ... 11 (k 1 's). Além disso, a soma dos restos parciais na
 divisão obtida de cada fração irredutível x/n é um múltiplo de n.
 Comentário:
 Pelo que entendi, se 1/13  tem período k =6. Então 13  divide 11 (
 k=6 1's).
 Essa parte consegui provar.
 Quanto à segunda parte  para 1/13 os resto da divisão sem repetição são
 {10, 9, 12, 3, 4, 1}. Então 10+9+12+3+4+1= 13q . (Não soube provar)
 Não consigo organizar uma sequência  de passos para a demonstração
 dos dois 

Re: [obm-l] Ajuda em Repunits

2022-07-10 Por tôpico Rubens Vilhena Fonseca
Muito obrigado ao Ralph Costa Teixeira e ao Claudio Buffara por todos os
ótimos esclarecimentos.
[[ ]]'s

Em dom., 10 de jul. de 2022 às 01:39, Ralph Costa Teixeira <
ralp...@gmail.com> escreveu:

> Argh, corrigindo um detalhe ali perto do fim:
> -- Sabemos que 10^q*B-B=r/10^w, portanto 9*(111...)**x**10^w = r*n.
> Novamente, como n é primo com 2, 3 e 5 *e x*, conclui-se que n divide
> 111 (com q 1's), e portanto q>=p=k.
>
> On Sun, Jul 10, 2022 at 1:24 AM Ralph Costa Teixeira 
> wrote:
>
>> A chave: *os "restos parciais" que aparecem são exatamente os restos que
>> x, 10x, 100x,  deixam na divisão por n.*
>> ---///---
>>
>> MAIS SPOILERS ABAIXO
>>
>>
>> ...
>>
>>
>> 
>>
>>
>> ...
>>
>>
>> 
>>
>> Acho que facilita bastante pensar no "período" de 1/n de outro jeito:
>> ---///---
>> LEMA:
>> (i) Dado n não divisível por 2 ou 5, existe algum número da forma
>> 111...111 que é múltiplo de n.
>> (ii) Se n não for divisível por 2, 3 ou 5, o *menor* número do tipo
>> 111...111 que é múltiplo de n tem k dígitos, onde k é exatamente o tamanho
>> do período (fundamental) da dízima em 1/n.
>> PROVA:
>>
>> (i) Olhe os restos de 1, 11, 111, , ... na divisão por n. São n
>> possibilidades, de 0 a n-1, então alguma hora algum resto tem que repetir.
>> Isto significa que .. (com A dígitos) e 11...111 (com B dígitos,
>> B> ...1110 (A 1's e B 0's) = 111 * (10^B) é múltiplo de n. Mas
>> n não tem fator comum com aquele 10^B (pois não é divisível por 2 nem por
>> 5), portanto ...111 (com k=A-B dígitos) é divisível por n.
>>
>> (ii) Denote por P=111111 (com p dígitos) o menor daqueles caras com
>> apenas "1s" que é múltiplo de n, e denote por k o "período fundamental" na
>> dízima de 1/n.
>> Por um lado, como 9P=999=10^p-1 é múltiplo de n, temos 10^p *
>> (1/n) - 1/n inteiro. Mas isso significa que a parte decimal de 1/n "se
>> repete" de p em p dígitos, ou seja, que a dízima de 1/n tem período p. Em
>> particular, p>=k.
>> Por outro lado, sendo k o período fundamental, temos 10^k * (1/n) - (1/n)
>> com número finito de casas decimais, ou seja, (10^k-1)/n = m/10^z com m
>> inteiro, e z=número de casas decimais que "sobraram". Mas daqui vem
>> 9*(111...111)*10^z = m*n (com k dígitos 1s). Como n é primo com 2, 3 e 5,
>> conclui-se que 111... (k 1's) tem que ser múltiplo de n, e portanto
>> k>=p.
>>
>> Note um efeito colateral disso tudo: provamos que 10^k*(1/n)- 1/n =
>> 10^p*(1/n)-1/n = inteiro. Assim aquele z vale 0, ou seja, não tem "casas
>> decimais que sobram" -- a dízima periódica do 1/n se inicia logo no
>> primeiro dígito!
>>
>> ---///---
>> Agora fica tudo bem simples:
>> a) Na notação acima, provamos que k=p, e n divide 111 com p
>> dígitos.
>> b) Seja q o período (fundamental) da dízima de B=x/n irredutível.
>>
>> Em primeiro lugar, provemos que q=k. Basicamente repetimos o que fizemos
>> no lema:
>> -- Sabemos que 10^q*B-B=r/10^w, portanto 9*(111...)*10^w = r*n.
>> Novamente, como n é primo com 2, 3 e 5, conclui-se que n divide 111
>> (com q 1's), e portanto q>=p=k.
>> -- Por outro lado, como (10^k-1)/n é inteiro, (10^k-1)*x/n=10^k*B-B
>> também é inteiro, ou seja, a dízima de B tem período k (e se inicia no
>> primeiro dígito!). Portanto k>=q.
>>
>> *Enfim, note que os tais "restos parciais" que aparecem são exatamente os
>> restos que x, 10x, 100x, , 10^q.x deixam na divisão por n. *A soma
>> desses caras vale (...)*x, que é divisível por n pois temos ali
>> q=k=p dígitos 1. Por isso, ao dividir esses restos parciais por n, a soma
>> dos novos restos tem que ser múltiplo de n tambem.
>>
>> Foi?
>>
>>
>> On Sat, Jul 9, 2022 at 7:16 PM Rubens Vilhena Fonseca <
>> rubens.vilhen...@gmail.com> wrote:
>>
>>> Gostaria de uma demonstração para o seguinte teorema.
>>> *Teorema*. Seja n um inteiro positivo não divisível por 2, 3 ou 5, e
>>> suponha que a expansão decimal de l/n tenha período k. Então n é um fator
>>> do inteiro 111 ... 11 (k 1 's). Além disso, a soma dos restos parciais na
>>> divisão obtida de cada fração irredutível x/n é um múltiplo de n.
>>> Comentário:
>>> Pelo que entendi, se 1/13  tem período k =6. Então 13  divide 11 (
>>> k=6 1's).
>>> Essa parte consegui provar.
>>> Quanto à segunda parte  para 1/13 os resto da divisão sem repetição são
>>> {10, 9, 12, 3, 4, 1}. Então 10+9+12+3+4+1= 13q . (Não soube provar)
>>> Não consigo organizar uma sequência  de passos para a demonstração
>>> dos dois fatos.
>>> Agradeço qualquer ajuda.
>>> [[ ]]'s
>>>
>>> --
>>> Esta mensagem foi verificada pelo sistema de antivírus e
>>> acredita-se estar livre de perigo.
>>
>>
> --
> Esta mensagem foi verificada pelo sistema de antivírus e
> acredita-se estar livre de perigo.

-- 
Esta mensagem foi verificada pelo sistema de antiv�rus e
 acredita-se estar livre de perigo.



Re: [obm-l] Ajuda em Repunits

2022-07-09 Por tôpico Claudio Buffara
Se n não é divisível por 2 e nem por 5, então 1/n = 0,a1a2...ak a1a2...ak
a1...  (dízima periódica simples de período k)
Daí (10^k)*n - n = a1a2...ak ==> (99...9)*n é inteiro (onde há k algarismos
9) ==> n é fator de 99...9 = 9*(11...1).
Mas n é primo com 3 ==> n | 11...1

Pra segunda parte, a ideia é tentar ver porque é verdade com exemplos
concretos.
Por exemplo, 1/7:
10*1 = 1*7 + 3
10*3 = 4*7 + 2
10*2 = 2*7 + 6
10*6 = 8*7 + 4
10*4 = 5*7 + 5
10*5 = 7*7 + 1
10*1 = 1*7 + 3  (e as equações se repetem a partir daqui)

1/13:
10*1 = 0*13 + 10
10*10 = 7*13 + 9
10*9 = 6*13 + 12
10*12 = 9*13 + 3
10*3 = 2*13 + 4
10*4 = 3*13 + 1
10*1 = 0*13 + 10 (idem)

Assim, no caso geral, pra calcular a representação de 1/n, as k primeiras
divisões sucessivas resultam em:
10*1 = a1*n + r1
10*r1 = a2*n + r2
10*r2 = a3*n + r3
...
10*r(k-1) = ak*n + rk

Como n é primo com 2 e 5, 1/n será uma dízima periódica simples, digamos de
período k.
Isso significa que rk, o resto da k-ésima divisão, será necessariamente
igual a 1, já que os dividendos (os algarismos aj que formam o período)
irão se repetir a partir da (k+1)-ésima equação.
Ou seja, a(k+1) = a1 e, portanto, r(k+1) = r1.

Somando as k equações, obtemos:
10*(1+r1+r2+ ...r(k-1)) = (a1+a2+a3...+ak)*n + (r1+r2+r3+...+rk).
Como rk = 1, isso fica:
10*(rk+r1+r2+ ...r(k-1)) = (a1+a2+a3...+ak)*n + (r1+r2+r3+...+rk) ==>
9*(rk+r2+...+r(k-1)) = (a1+a2+a3+...+ak)*n
Como n é primo com 3 (e, portanto, com 9), concluímos que n divide
r1+r2+...+rk.




On Sat, Jul 9, 2022 at 7:16 PM Rubens Vilhena Fonseca <
rubens.vilhen...@gmail.com> wrote:

> Gostaria de uma demonstração para o seguinte teorema.
> *Teorema*. Seja n um inteiro positivo não divisível por 2, 3 ou 5, e
> suponha que a expansão decimal de l/n tenha período k. Então n é um fator
> do inteiro 111 ... 11 (k 1 's). Além disso, a soma dos restos parciais na
> divisão obtida de cada fração irredutível x/n é um múltiplo de n.
> Comentário:
> Pelo que entendi, se 1/13  tem período k =6. Então 13  divide 11 ( k=6
> 1's).
> Essa parte consegui provar.
> Quanto à segunda parte  para 1/13 os resto da divisão sem repetição são
> {10, 9, 12, 3, 4, 1}. Então 10+9+12+3+4+1= 13q . (Não soube provar)
> Não consigo organizar uma sequência  de passos para a demonstração
> dos dois fatos.
> Agradeço qualquer ajuda.
> [[ ]]'s
>
> --
> Esta mensagem foi verificada pelo sistema de antivírus e
> acredita-se estar livre de perigo.

-- 
Esta mensagem foi verificada pelo sistema de antiv�rus e
 acredita-se estar livre de perigo.



Re: [obm-l] Ajuda em Repunits

2022-07-09 Por tôpico Ralph Costa Teixeira
Argh, corrigindo um detalhe ali perto do fim:
-- Sabemos que 10^q*B-B=r/10^w, portanto 9*(111...)**x**10^w = r*n.
Novamente, como n é primo com 2, 3 e 5 *e x*, conclui-se que n divide
111 (com q 1's), e portanto q>=p=k.

On Sun, Jul 10, 2022 at 1:24 AM Ralph Costa Teixeira 
wrote:

> A chave: *os "restos parciais" que aparecem são exatamente os restos que
> x, 10x, 100x,  deixam na divisão por n.*
> ---///---
>
> MAIS SPOILERS ABAIXO
>
>
> ...
>
>
> 
>
>
> ...
>
>
> 
>
> Acho que facilita bastante pensar no "período" de 1/n de outro jeito:
> ---///---
> LEMA:
> (i) Dado n não divisível por 2 ou 5, existe algum número da forma
> 111...111 que é múltiplo de n.
> (ii) Se n não for divisível por 2, 3 ou 5, o *menor* número do tipo
> 111...111 que é múltiplo de n tem k dígitos, onde k é exatamente o tamanho
> do período (fundamental) da dízima em 1/n.
> PROVA:
>
> (i) Olhe os restos de 1, 11, 111, , ... na divisão por n. São n
> possibilidades, de 0 a n-1, então alguma hora algum resto tem que repetir.
> Isto significa que .. (com A dígitos) e 11...111 (com B dígitos,
> B ...1110 (A 1's e B 0's) = 111 * (10^B) é múltiplo de n. Mas
> n não tem fator comum com aquele 10^B (pois não é divisível por 2 nem por
> 5), portanto ...111 (com k=A-B dígitos) é divisível por n.
>
> (ii) Denote por P=111111 (com p dígitos) o menor daqueles caras com
> apenas "1s" que é múltiplo de n, e denote por k o "período fundamental" na
> dízima de 1/n.
> Por um lado, como 9P=999=10^p-1 é múltiplo de n, temos 10^p *
> (1/n) - 1/n inteiro. Mas isso significa que a parte decimal de 1/n "se
> repete" de p em p dígitos, ou seja, que a dízima de 1/n tem período p. Em
> particular, p>=k.
> Por outro lado, sendo k o período fundamental, temos 10^k * (1/n) - (1/n)
> com número finito de casas decimais, ou seja, (10^k-1)/n = m/10^z com m
> inteiro, e z=número de casas decimais que "sobraram". Mas daqui vem
> 9*(111...111)*10^z = m*n (com k dígitos 1s). Como n é primo com 2, 3 e 5,
> conclui-se que 111... (k 1's) tem que ser múltiplo de n, e portanto
> k>=p.
>
> Note um efeito colateral disso tudo: provamos que 10^k*(1/n)- 1/n =
> 10^p*(1/n)-1/n = inteiro. Assim aquele z vale 0, ou seja, não tem "casas
> decimais que sobram" -- a dízima periódica do 1/n se inicia logo no
> primeiro dígito!
>
> ---///---
> Agora fica tudo bem simples:
> a) Na notação acima, provamos que k=p, e n divide 111 com p
> dígitos.
> b) Seja q o período (fundamental) da dízima de B=x/n irredutível.
>
> Em primeiro lugar, provemos que q=k. Basicamente repetimos o que fizemos
> no lema:
> -- Sabemos que 10^q*B-B=r/10^w, portanto 9*(111...)*10^w = r*n.
> Novamente, como n é primo com 2, 3 e 5, conclui-se que n divide 111
> (com q 1's), e portanto q>=p=k.
> -- Por outro lado, como (10^k-1)/n é inteiro, (10^k-1)*x/n=10^k*B-B também
> é inteiro, ou seja, a dízima de B tem período k (e se inicia no primeiro
> dígito!). Portanto k>=q.
>
> *Enfim, note que os tais "restos parciais" que aparecem são exatamente os
> restos que x, 10x, 100x, , 10^q.x deixam na divisão por n. *A soma
> desses caras vale (...)*x, que é divisível por n pois temos ali
> q=k=p dígitos 1. Por isso, ao dividir esses restos parciais por n, a soma
> dos novos restos tem que ser múltiplo de n tambem.
>
> Foi?
>
>
> On Sat, Jul 9, 2022 at 7:16 PM Rubens Vilhena Fonseca <
> rubens.vilhen...@gmail.com> wrote:
>
>> Gostaria de uma demonstração para o seguinte teorema.
>> *Teorema*. Seja n um inteiro positivo não divisível por 2, 3 ou 5, e
>> suponha que a expansão decimal de l/n tenha período k. Então n é um fator
>> do inteiro 111 ... 11 (k 1 's). Além disso, a soma dos restos parciais na
>> divisão obtida de cada fração irredutível x/n é um múltiplo de n.
>> Comentário:
>> Pelo que entendi, se 1/13  tem período k =6. Então 13  divide 11 (
>> k=6 1's).
>> Essa parte consegui provar.
>> Quanto à segunda parte  para 1/13 os resto da divisão sem repetição são
>> {10, 9, 12, 3, 4, 1}. Então 10+9+12+3+4+1= 13q . (Não soube provar)
>> Não consigo organizar uma sequência  de passos para a demonstração
>> dos dois fatos.
>> Agradeço qualquer ajuda.
>> [[ ]]'s
>>
>> --
>> Esta mensagem foi verificada pelo sistema de antivírus e
>> acredita-se estar livre de perigo.
>
>

-- 
Esta mensagem foi verificada pelo sistema de antiv�rus e
 acredita-se estar livre de perigo.



Re: [obm-l] Ajuda em Repunits

2022-07-09 Por tôpico Ralph Costa Teixeira
A chave: *os "restos parciais" que aparecem são exatamente os restos que x,
10x, 100x,  deixam na divisão por n.*
---///---

MAIS SPOILERS ABAIXO


...





...




Acho que facilita bastante pensar no "período" de 1/n de outro jeito:
---///---
LEMA:
(i) Dado n não divisível por 2 ou 5, existe algum número da forma 111...111
que é múltiplo de n.
(ii) Se n não for divisível por 2, 3 ou 5, o *menor* número do tipo
111...111 que é múltiplo de n tem k dígitos, onde k é exatamente o tamanho
do período (fundamental) da dízima em 1/n.
PROVA:

(i) Olhe os restos de 1, 11, 111, , ... na divisão por n. São n
possibilidades, de 0 a n-1, então alguma hora algum resto tem que repetir.
Isto significa que .. (com A dígitos) e 11...111 (com B dígitos,
B=k.
Por outro lado, sendo k o período fundamental, temos 10^k * (1/n) - (1/n)
com número finito de casas decimais, ou seja, (10^k-1)/n = m/10^z com m
inteiro, e z=número de casas decimais que "sobraram". Mas daqui vem
9*(111...111)*10^z = m*n (com k dígitos 1s). Como n é primo com 2, 3 e 5,
conclui-se que 111... (k 1's) tem que ser múltiplo de n, e portanto
k>=p.

Note um efeito colateral disso tudo: provamos que 10^k*(1/n)- 1/n =
10^p*(1/n)-1/n = inteiro. Assim aquele z vale 0, ou seja, não tem "casas
decimais que sobram" -- a dízima periódica do 1/n se inicia logo no
primeiro dígito!

---///---
Agora fica tudo bem simples:
a) Na notação acima, provamos que k=p, e n divide 111 com p dígitos.
b) Seja q o período (fundamental) da dízima de B=x/n irredutível.

Em primeiro lugar, provemos que q=k. Basicamente repetimos o que fizemos no
lema:
-- Sabemos que 10^q*B-B=r/10^w, portanto 9*(111...)*10^w = r*n.
Novamente, como n é primo com 2, 3 e 5, conclui-se que n divide 111
(com q 1's), e portanto q>=p=k.
-- Por outro lado, como (10^k-1)/n é inteiro, (10^k-1)*x/n=10^k*B-B também
é inteiro, ou seja, a dízima de B tem período k (e se inicia no primeiro
dígito!). Portanto k>=q.

*Enfim, note que os tais "restos parciais" que aparecem são exatamente os
restos que x, 10x, 100x, , 10^q.x deixam na divisão por n. *A soma
desses caras vale (...)*x, que é divisível por n pois temos ali
q=k=p dígitos 1. Por isso, ao dividir esses restos parciais por n, a soma
dos novos restos tem que ser múltiplo de n tambem.

Foi?


On Sat, Jul 9, 2022 at 7:16 PM Rubens Vilhena Fonseca <
rubens.vilhen...@gmail.com> wrote:

> Gostaria de uma demonstração para o seguinte teorema.
> *Teorema*. Seja n um inteiro positivo não divisível por 2, 3 ou 5, e
> suponha que a expansão decimal de l/n tenha período k. Então n é um fator
> do inteiro 111 ... 11 (k 1 's). Além disso, a soma dos restos parciais na
> divisão obtida de cada fração irredutível x/n é um múltiplo de n.
> Comentário:
> Pelo que entendi, se 1/13  tem período k =6. Então 13  divide 11 ( k=6
> 1's).
> Essa parte consegui provar.
> Quanto à segunda parte  para 1/13 os resto da divisão sem repetição são
> {10, 9, 12, 3, 4, 1}. Então 10+9+12+3+4+1= 13q . (Não soube provar)
> Não consigo organizar uma sequência  de passos para a demonstração
> dos dois fatos.
> Agradeço qualquer ajuda.
> [[ ]]'s
>
> --
> Esta mensagem foi verificada pelo sistema de antivírus e
> acredita-se estar livre de perigo.

-- 
Esta mensagem foi verificada pelo sistema de antiv�rus e
 acredita-se estar livre de perigo.



[obm-l] Re: [obm-l] Questão de probabilidade

2022-06-29 Por tôpico Rogerio Ponce
Ola' Vanderlei e pessoal da lista!
Pediram-me para resolver o problema por inteiro.
Ok, vamos la'!


Em um pet shop ha' 3 gatos e 5 caes. Sabemos que 3 desses animais sao
pretos, 4 sao brancos e 1 e' malhado. Alem disso, pelo menos 1
cachorro e' preto. Assinale o que for correto.
01) A probabilidade de haver exatamente 1 cachorro preto e' de 1/6.
02) A probabilidade de haver pelo menos 1 gato branco e pelo menos 2
cachorros brancos e' de 2/3.
04) A probabilidade de haver um cachorro malhado e' maior do que a
probabilidade de haver um gato malhado.
08) Se um animal for escolhido ao acaso, a probabilidade de ele ser um
cachorro preto e' de 1/8.
16) Se um animal for escolhido ao acaso, a probabilidade de ele ser um
gato malhado e' de 1/16.



Afirmacao 01) A probabilidade de haver exatamente 1 cachorro preto e' de 1/6.

-

O cachorro que ja' e' preto, deixaremos de lado, de modo que temos
3Gatos,4Caes para pintar com 2Preto,4Branco,1Malhado.
Bem, dizer que existe exatamente 1 cachorro preto, significa dizer que
as outras 2 pinturas pretas foram para os gatos, ou que existem
exatamente 2 gatos pretos.

Entao, vamos comecar a pintar com a tinta preta, e verificar de
quantas formas podemos fazer isso.

Repare que para ponderar corretamente as ocorrencias que interessam,
eu considero todas as permutacoes possiveis, ou das tintas, ou dos
animais. Assim, posso, por exemplo, fixar a ordem das tintas, e
aplicar isso em todas as permutacoes possiveis entre os animais.
A razao entre os casos que interessam e os casos possiveis e'
justamente a probabilidade de ocorrencia de um caso que interessa.

Como sao 2 pinturas pretas, temos um total de 7 escolhas de animal
para a primeira pintura, e 6 escolhas para a segunda, com um total de
42 escolhas para a primeira e segunda pinturas pretas.

Deste total, temos 3 gatos para a primeira escolha e 2 gatos para a
segunda, com um total de 6 escolhas para termos 2 gatos pretos.
Deste total, vamos escolher 1 entre 3 gatos para a primeira pintura, e
1 entre 2 gatos para a segunda pintura, com um total de 6 escolhas
para pintarmos 2 gatos de preto.

Assim, a probabilidade de termos exatamente 2 gatos pretos vale 6/42 = 1/7
Ou seja, a probabilidade de termos exatamente um cachorro preto vale
1/7, de modo que a afirmacao 01 esta' errada.

OBS: note que, entre os gatos g1, g2 e g3 (por exemplo), mesmo que
pintemos g1 e g2 de preto, a ordem e' importante, pois ordens
diferentes correspondem a permutacoes diferentes. Pintar de preto o
gato g1, e depois o gato g2, e' uma permutacao diferente daquela em
que pintamos de preto primeiramente o gato g2, e depois o gato g1,
embora o resultado final seja obtermos g1 e g2 pretos.



Afirmacao 02) A probabilidade de haver pelo menos 1 gato branco e pelo
menos 2 cachorros brancos e' de 2/3.

-

O cachorro que ja' e' preto pode ser deixado de lado.

Seguindo agora com as 4 pinturas brancas, temos um total de 7 escolhas
para o primeiro animal, 6 para o segundo, 5 para o terceiro, e 4 para
o quarto, de modo que existem 7*6*5*4 escolhas diferentes para as
pinturas brancas.

Entre os animais que receberao as pinturas brancas, existem os seguintes casos:

1o caso: 4 caes brancos
O numero de combinacoes de 4 caes entre 4 caes e' C(4,4)=1.
Assim, considerando-se as permutacoes entre estes 4 animais
brancos, obtemos um total de permutacoes diferentes igual a
C(4,4)*4! = 1*4! = 24

2o caso: 3 caes e 1 gato brancos
Como sao 3 caes de um total de 4 caes possiveis, e 1 gato de
um total de 3 gatos possiveis, o numero total de permutacoes vale
C(4,3) * C(3,1) * 4! = 4*3*4! = 288

3o caso: 2 caes e 2 gatos brancos
o numero total de permutacoes vale
C(4,2) * C(3,2) * 4! = 6*3*4! = 432

4o caso: 1 cao e 3 gatos brancos
o numero total de permutacoes vale
C(4,1) * C(3,3) * 4! = 4*1*4! = 96


Contabilizando-se o segundo e o terceiro casos (que sao os que nos
interessam), temos um total de (288+432)=720 permutacoes, de um
universo de 840 permutacoes, de modo que a probabilidade pedida vale
720/840 = 6/7

Portanto, a afirmacao 02 esta' errada.



Afirmacao 04) A probabilidade de haver um cachorro malhado e' maior do
que a probabilidade de haver um gato malhado.

-

Deixando de lado o cachorro que ja' e' preto, e seguindo agora com a
pintura malhada, temos 7 escolhas de animais, das quais 4 sao de
cachorros, e 3 sao de gatos.

Assim, a probabilidade de haver um cachorro malhado e' 4/7 , e a
probabilidade de haver um gato malhado e' 3/7.

Portanto, a afirmacao 04 esta' correta.



Afirmacao 08) Se um animal for escolhido ao acaso, a probabilidade de
ele ser um cachorro preto e' de 1/8.

-

Se houvesse apenas um cachorro preto - aquele que ja' seria mesmo
preto, por definicao - a probabilidade de escolhermos um 

[obm-l] Re: [obm-l] Re: [obm-l] Re: [obm-l] Re: [obm-l] Re: [obm-l] Questão de probabilidade

2022-06-24 Por tôpico Rogerio Ponce
Otima explicacao!
Obrigado, Ralph!

PS: e sim, a provocacao foi pra voce mesmo!
:)

[]'s
Rogerio Ponce


On Wed, Jun 22, 2022 at 1:00 PM Ralph Costa Teixeira  wrote:
>
> Ponce está provocando a gente... senti que esta flecha tinha um bocado a 
> minha direção...  :D :D :D
>
> Olha, tem duas "visões" sobre o que "probabilidade" significa.
>
> A primeira vai na linha de que só podemos falar de probabilidade sobre coisas 
> que ainda não aconteceram. Vai nessa linha: se os eventos estão no passado, 
> então já aconteceram, já estão definidos, e não faz sentido dizer que tinha x 
> de chance de ser assim ou y de ser assado. Se você já jogou a moeda justa, 
> não é mais 50/50 --  é 100% de ser cara, ou 100% de ser coroa, dependendo do 
> que ocorreu. Quem pensa assim vai dizer que dado um certo evento (sempre no 
> futuro), ele tem uma probabilidade dada; se duas pessoas diferentes derem 
> duas probabilidades diferentes para o mesmo evento, uma delas errou.
>
> Outra linha diz que podemos falar de probabilidade sempre que houver 
> incerteza; não interessa o que aconteceu ou o que vai acontecer, o que 
> interessa é o que você SABE sobre o acontecimento. Se você jogou a moeda 
> justa mas eu não sei nada mais sobre o lançamento, continua sendo 50/50 
> **PARA MIM**. Probabilidade passa a ser um conceito sobre INFORMAÇÃO, não 
> sobre os fatos em si (a probabilidade não está na moeda, está no que você 
> sabe sobre a moeda). Quem pensa assim vai dizer que a probabilidade do evento 
> depende não apenas do evento em si, mas da informação que se tem em mãos. 
> Quem pensa assim admite que duas pessoas diferentes podem dar probabilidades 
> diferentes ao mesmo evento SE SOUBEREM FATOS DIFERENTES a respeito do evento, 
> ou seja, probabilidade passa a ser bastante "subjetivo".
>
> Eu talvez tenha descrito mal a primeira interpretação, pois sou ferrenho 
> defensor da segunda. Ela simplesmente engloba a primeira, porque você pode 
> ter informação parcial sobre fatos que ocorrem no futuro. E falar de 
> probabilidade para descrever incerteza presente ou passada é MUITO útil! Eu 
> quero poder expressar incerteza sobre fatos passados com frases do tipo 
> "fulano tem x% de probabilidade de ter cometido tal crime", ou "tem y% de 
> probabilidade de ter petróleo nesse poço", ou "tem z% de chance de eu ter 
> COVID"... Se você tem uma reação negativa a essas frases, lembre o que elas 
> realmente significam (na segunda interpretação): claro que ou o cara cometeu 
> o crime ou não, não faz sentido dizer que ele cometeu o crime x% das vezes em 
> que fizermos um experimento de ele cometer o crime... mas o que aquilo 
> significa é "com a informação que eu tenho, numa escala de 0 a 1, eu tenho 
> x/100 de certeza que fulano cometeu o crime". E "certeza baseada em informaç�!
 �es" é sim quantificável -- e satisfaz exatamente as leis das probabilidades 
com as quais concordamos. "Subjetivo" não significa "posso falar qualquer 
coisa", significa apenas que a conta pode variar de pessoa para pessoa... mas, 
de novo, SE ESSAS PESSOAS TIVEREM INFORMAÇÕES DIFERENTES sobre o evento.
>
> Abraço, Ralph.
>
> On Wed, Jun 22, 2022 at 12:09 PM Rogerio Ponce  wrote:
>>
>> Olá Pedro e pessoal da lista!
>>
>> Segundo a opinião do Pedro, nao faz sentido perguntar qual a probabilidade 
>> de Jose ter conseguido um 6 ao jogar o dado ontem, pois isso ja' aconteceu, 
>> e, portanto, ja' esta' definido.
>>
>> Sera' que e' isso mesmo?
>>
>> []'s
>> Rogerio Ponce
>>
>>
>> On Mon, Jun 20, 2022 at 9:45 PM Pedro José  wrote:
>>>
>>> Eu na minha humilde opinião creio que a probabilidade exista quando pode 
>>> ser uma coisa ou outra. No caso já é definido o que os animais são. Então 
>>> já está tudo errado. A questão seria viável se dessem esses limitantes para 
>>> uma criança que pintaria os desenhos dos animais. Aí sim há probabilidade.
>>>
>>> Em sáb., 18 de jun. de 2022 03:33, Rogerio Ponce  
>>> escreveu:

 Ola' Vanderlei e pessoal da lista!

 Sem perda de generalidade, podemos imaginar que vamos fazer o seguinte:

 - uma pintura preta em um dos caes, escolhido aleatoriamente

 - uma pintura "malhada" em um dos animais, escolhido aleatoriamente entre 
 os 7 animais nao pintados

 - duas pintura pretas, em dois animais, escolhidos aleatoriamente entre os 
 6 animais restantes,

 - quatro pinturas brancas nos 4 animais restantes


 Analisando a afirmacao 04, por exemplo, verificamos que, no segundo passo 
 (pintura malhada) existem 4 opcoes de cachorro e 3 opcoes de gato.

 Assim, a probabilidade de haver um cachorro malhado (4/7) e' maior que a 
 probabilidade de haver um gato malhado (3/7).
 Portanto, a afirmacao 04 esta' correta.
 (e o gabarito esta' errado).

 []'s
 Rogerio Ponce



 On Wed, Mar 16, 2022 at 8:08 AM Professor Vanderlei Nemitz 
  wrote:
>
> Bom dia!
> Na questão a seguir, do vestibular da UEM, 

[obm-l] Re: [obm-l] Re: [obm-l] Re: [obm-l] Re: [obm-l] Questão de probabilidade

2022-06-22 Por tôpico Ralph Costa Teixeira
Ponce está provocando a gente... senti que esta flecha tinha um bocado a
minha direção...  :D :D :D

Olha, tem duas "visões" sobre o que "probabilidade" significa.

A primeira vai na linha de que só podemos falar de probabilidade sobre
coisas que ainda não aconteceram. Vai nessa linha: se os eventos estão no
passado, então já aconteceram, já estão definidos, e não faz sentido dizer
que tinha x de chance de ser assim ou y de ser assado. Se você já jogou a
moeda justa, não é mais 50/50 --  é 100% de ser cara, ou 100% de ser coroa,
dependendo do que ocorreu. Quem pensa assim vai dizer que dado um certo
evento (sempre no futuro), ele tem uma probabilidade dada; se duas pessoas
diferentes derem duas probabilidades diferentes para o mesmo evento, uma
delas errou.

Outra linha diz que podemos falar de probabilidade sempre que houver
incerteza; não interessa o que aconteceu ou o que vai acontecer, o que
interessa é o que você SABE sobre o acontecimento. Se você jogou a moeda
justa mas eu não sei nada mais sobre o lançamento, continua sendo 50/50
**PARA MIM**. Probabilidade passa a ser um conceito sobre INFORMAÇÃO, não
sobre os fatos em si (a probabilidade não está na moeda, está no que você
sabe sobre a moeda). Quem pensa assim vai dizer que a probabilidade do
evento depende não apenas do evento em si, mas da informação que se tem em
mãos. Quem pensa assim admite que duas pessoas diferentes podem dar
probabilidades diferentes ao mesmo evento SE SOUBEREM FATOS DIFERENTES a
respeito do evento, ou seja, probabilidade passa a ser bastante "subjetivo".

Eu talvez tenha descrito mal a primeira interpretação, pois sou ferrenho
defensor da segunda. Ela simplesmente engloba a primeira, porque você pode
ter informação parcial sobre fatos que ocorrem no futuro. E falar de
probabilidade para descrever incerteza presente ou passada é MUITO útil! Eu
quero poder expressar incerteza sobre fatos passados com frases do tipo
"fulano tem x% de probabilidade de ter cometido tal crime", ou "tem y% de
probabilidade de ter petróleo nesse poço", ou "tem z% de chance de eu ter
COVID"... Se você tem uma reação negativa a essas frases, lembre o que elas
realmente significam (na segunda interpretação): claro que ou o cara
cometeu o crime ou não, não faz sentido dizer que ele cometeu o crime x%
das vezes em que fizermos um experimento de ele cometer o crime... mas o
que aquilo significa é "com a informação que eu tenho, numa escala de 0 a
1, eu tenho x/100 de certeza que fulano cometeu o crime". E "certeza
baseada em informações" é sim quantificável -- e satisfaz exatamente as
leis das probabilidades com as quais concordamos. "Subjetivo" não significa
"posso falar qualquer coisa", significa apenas que a conta pode variar de
pessoa para pessoa... mas, de novo, SE ESSAS PESSOAS TIVEREM INFORMAÇÕES
DIFERENTES sobre o evento.

Abraço, Ralph.

On Wed, Jun 22, 2022 at 12:09 PM Rogerio Ponce  wrote:

> Olá Pedro e pessoal da lista!
>
> Segundo a opinião do Pedro, nao faz sentido perguntar qual a probabilidade
> de Jose ter conseguido um 6 ao jogar o dado ontem, pois isso ja' aconteceu,
> e, portanto, ja' esta' definido.
>
> Sera' que e' isso mesmo?
>
> []'s
> Rogerio Ponce
>
>
> On Mon, Jun 20, 2022 at 9:45 PM Pedro José  wrote:
>
>> Eu na minha humilde opinião creio que a probabilidade exista quando pode
>> ser uma coisa ou outra. No caso já é definido o que os animais são. Então
>> já está tudo errado. A questão seria viável se dessem esses limitantes para
>> uma criança que pintaria os desenhos dos animais. Aí sim há probabilidade.
>>
>> Em sáb., 18 de jun. de 2022 03:33, Rogerio Ponce da Silva <
>> abrlw...@gmail.com> escreveu:
>>
>>> Ola' Vanderlei e pessoal da lista!
>>>
>>> Sem perda de generalidade, podemos imaginar que vamos fazer o seguinte:
>>>
>>> - uma pintura preta em um dos caes, escolhido aleatoriamente
>>>
>>> - uma pintura "malhada" em um dos animais, escolhido aleatoriamente
>>> entre os 7 animais nao pintados
>>>
>>> - duas pintura pretas, em dois animais, escolhidos aleatoriamente entre
>>> os 6 animais restantes,
>>>
>>> - quatro pinturas brancas nos 4 animais restantes
>>>
>>>
>>> Analisando a afirmacao 04, por exemplo, verificamos que, no segundo
>>> passo (pintura malhada) existem 4 opcoes de cachorro e 3 opcoes de gato.
>>>
>>> Assim, a probabilidade de haver um cachorro malhado (4/7) e' maior que a
>>> probabilidade de haver um gato malhado (3/7).
>>> Portanto, a afirmacao 04 esta' correta.
>>> (e o gabarito esta' errado).
>>>
>>> []'s
>>> Rogerio Ponce
>>>
>>>
>>>
>>> On Wed, Mar 16, 2022 at 8:08 AM Professor Vanderlei Nemitz <
>>> vanderma...@gmail.com> wrote:
>>>
 Bom dia!
 Na questão a seguir, do vestibular da UEM, penso que o espaço amostral
 tem 105 elementos, pois um cachorro é preto (desconsideramos esse). Porém,
 com esse pensamento, não consigo obter o gabarito, que diz que 02 e 16 são
 corretas.
 Alguém poderia ajudar?
 Muito obrigado!

 *Em um pet shop há 3 gatos e 5 cães. 

[obm-l] Re: [obm-l] Re: [obm-l] Re: [obm-l] Questão de probabilidade

2022-06-22 Por tôpico Rogerio Ponce
Olá Pedro e pessoal da lista!

Segundo a opinião do Pedro, nao faz sentido perguntar qual a probabilidade
de Jose ter conseguido um 6 ao jogar o dado ontem, pois isso ja' aconteceu,
e, portanto, ja' esta' definido.

Sera' que e' isso mesmo?

[]'s
Rogerio Ponce


On Mon, Jun 20, 2022 at 9:45 PM Pedro José  wrote:

> Eu na minha humilde opinião creio que a probabilidade exista quando pode
> ser uma coisa ou outra. No caso já é definido o que os animais são. Então
> já está tudo errado. A questão seria viável se dessem esses limitantes para
> uma criança que pintaria os desenhos dos animais. Aí sim há probabilidade.
>
> Em sáb., 18 de jun. de 2022 03:33, Rogerio Ponce da Silva <
> abrlw...@gmail.com> escreveu:
>
>> Ola' Vanderlei e pessoal da lista!
>>
>> Sem perda de generalidade, podemos imaginar que vamos fazer o seguinte:
>>
>> - uma pintura preta em um dos caes, escolhido aleatoriamente
>>
>> - uma pintura "malhada" em um dos animais, escolhido aleatoriamente entre
>> os 7 animais nao pintados
>>
>> - duas pintura pretas, em dois animais, escolhidos aleatoriamente entre
>> os 6 animais restantes,
>>
>> - quatro pinturas brancas nos 4 animais restantes
>>
>>
>> Analisando a afirmacao 04, por exemplo, verificamos que, no segundo passo
>> (pintura malhada) existem 4 opcoes de cachorro e 3 opcoes de gato.
>>
>> Assim, a probabilidade de haver um cachorro malhado (4/7) e' maior que a
>> probabilidade de haver um gato malhado (3/7).
>> Portanto, a afirmacao 04 esta' correta.
>> (e o gabarito esta' errado).
>>
>> []'s
>> Rogerio Ponce
>>
>>
>>
>> On Wed, Mar 16, 2022 at 8:08 AM Professor Vanderlei Nemitz <
>> vanderma...@gmail.com> wrote:
>>
>>> Bom dia!
>>> Na questão a seguir, do vestibular da UEM, penso que o espaço amostral
>>> tem 105 elementos, pois um cachorro é preto (desconsideramos esse). Porém,
>>> com esse pensamento, não consigo obter o gabarito, que diz que 02 e 16 são
>>> corretas.
>>> Alguém poderia ajudar?
>>> Muito obrigado!
>>>
>>> *Em um pet shop há 3 gatos e 5 cães. Sabemos que 3 desses animais são
>>> pretos, 4 são brancos e 1 é malhado. Além disso, pelo menos 1 cachorro é
>>> preto. Assinale o que for correto. *
>>> *01) A probabilidade de haver exatamente 1 cachorro preto é de 1/6. *
>>> *02) A probabilidade de haver pelo menos 1 gato branco e pelo menos 2
>>> cachorros brancos é de 2/3.*
>>> *04) A probabilidade de haver um cachorro malhado é maior do que a
>>> probabilidade de haver um gato malhado. *
>>> *08) Se um animal for escolhido ao acaso, a probabilidade de ele ser um
>>> cachorro preto é de 1/8. *
>>> *16) Se um animal for escolhido ao acaso, a probabilidade de ele ser um
>>> gato malhado é de 1/16.   *
>>>
>>> --
>>> Esta mensagem foi verificada pelo sistema de antivírus e
>>> acredita-se estar livre de perigo.
>>
>>
>> --
>> Esta mensagem foi verificada pelo sistema de antivírus e
>> acredita-se estar livre de perigo.
>
>
> --
> Esta mensagem foi verificada pelo sistema de antivírus e
> acredita-se estar livre de perigo.

-- 
Esta mensagem foi verificada pelo sistema de antiv�rus e
 acredita-se estar livre de perigo.



[obm-l] Re: [obm-l] Re: [obm-l] Questão de probabilidade

2022-06-20 Por tôpico Pedro José
Eu na minha humilde opinião creio que a probabilidade exista quando pode
ser uma coisa ou outra. No caso já é definido o que os animais são. Então
já está tudo errado. A questão seria viável se dessem esses limitantes para
uma criança que pintaria os desenhos dos animais. Aí sim há probabilidade.

Em sáb., 18 de jun. de 2022 03:33, Rogerio Ponce da Silva <
abrlw...@gmail.com> escreveu:

> Ola' Vanderlei e pessoal da lista!
>
> Sem perda de generalidade, podemos imaginar que vamos fazer o seguinte:
>
> - uma pintura preta em um dos caes, escolhido aleatoriamente
>
> - uma pintura "malhada" em um dos animais, escolhido aleatoriamente entre
> os 7 animais nao pintados
>
> - duas pintura pretas, em dois animais, escolhidos aleatoriamente entre os
> 6 animais restantes,
>
> - quatro pinturas brancas nos 4 animais restantes
>
>
> Analisando a afirmacao 04, por exemplo, verificamos que, no segundo passo
> (pintura malhada) existem 4 opcoes de cachorro e 3 opcoes de gato.
>
> Assim, a probabilidade de haver um cachorro malhado (4/7) e' maior que a
> probabilidade de haver um gato malhado (3/7).
> Portanto, a afirmacao 04 esta' correta.
> (e o gabarito esta' errado).
>
> []'s
> Rogerio Ponce
>
>
>
> On Wed, Mar 16, 2022 at 8:08 AM Professor Vanderlei Nemitz <
> vanderma...@gmail.com> wrote:
>
>> Bom dia!
>> Na questão a seguir, do vestibular da UEM, penso que o espaço amostral
>> tem 105 elementos, pois um cachorro é preto (desconsideramos esse). Porém,
>> com esse pensamento, não consigo obter o gabarito, que diz que 02 e 16 são
>> corretas.
>> Alguém poderia ajudar?
>> Muito obrigado!
>>
>> *Em um pet shop há 3 gatos e 5 cães. Sabemos que 3 desses animais são
>> pretos, 4 são brancos e 1 é malhado. Além disso, pelo menos 1 cachorro é
>> preto. Assinale o que for correto. *
>> *01) A probabilidade de haver exatamente 1 cachorro preto é de 1/6. *
>> *02) A probabilidade de haver pelo menos 1 gato branco e pelo menos 2
>> cachorros brancos é de 2/3.*
>> *04) A probabilidade de haver um cachorro malhado é maior do que a
>> probabilidade de haver um gato malhado. *
>> *08) Se um animal for escolhido ao acaso, a probabilidade de ele ser um
>> cachorro preto é de 1/8. *
>> *16) Se um animal for escolhido ao acaso, a probabilidade de ele ser um
>> gato malhado é de 1/16.   *
>>
>> --
>> Esta mensagem foi verificada pelo sistema de antivírus e
>> acredita-se estar livre de perigo.
>
>
> --
> Esta mensagem foi verificada pelo sistema de antivírus e
> acredita-se estar livre de perigo.

-- 
Esta mensagem foi verificada pelo sistema de antiv�rus e
 acredita-se estar livre de perigo.



[obm-l] Re: [obm-l] Questão de probabilidade

2022-06-18 Por tôpico Rogerio Ponce da Silva
Ola' Vanderlei e pessoal da lista!

Sem perda de generalidade, podemos imaginar que vamos fazer o seguinte:

- uma pintura preta em um dos caes, escolhido aleatoriamente

- uma pintura "malhada" em um dos animais, escolhido aleatoriamente entre
os 7 animais nao pintados

- duas pintura pretas, em dois animais, escolhidos aleatoriamente entre os
6 animais restantes,

- quatro pinturas brancas nos 4 animais restantes


Analisando a afirmacao 04, por exemplo, verificamos que, no segundo passo
(pintura malhada) existem 4 opcoes de cachorro e 3 opcoes de gato.

Assim, a probabilidade de haver um cachorro malhado (4/7) e' maior que a
probabilidade de haver um gato malhado (3/7).
Portanto, a afirmacao 04 esta' correta.
(e o gabarito esta' errado).

[]'s
Rogerio Ponce



On Wed, Mar 16, 2022 at 8:08 AM Professor Vanderlei Nemitz <
vanderma...@gmail.com> wrote:

> Bom dia!
> Na questão a seguir, do vestibular da UEM, penso que o espaço amostral tem
> 105 elementos, pois um cachorro é preto (desconsideramos esse). Porém, com
> esse pensamento, não consigo obter o gabarito, que diz que 02 e 16 são
> corretas.
> Alguém poderia ajudar?
> Muito obrigado!
>
> *Em um pet shop há 3 gatos e 5 cães. Sabemos que 3 desses animais são
> pretos, 4 são brancos e 1 é malhado. Além disso, pelo menos 1 cachorro é
> preto. Assinale o que for correto. *
> *01) A probabilidade de haver exatamente 1 cachorro preto é de 1/6. *
> *02) A probabilidade de haver pelo menos 1 gato branco e pelo menos 2
> cachorros brancos é de 2/3.*
> *04) A probabilidade de haver um cachorro malhado é maior do que a
> probabilidade de haver um gato malhado. *
> *08) Se um animal for escolhido ao acaso, a probabilidade de ele ser um
> cachorro preto é de 1/8. *
> *16) Se um animal for escolhido ao acaso, a probabilidade de ele ser um
> gato malhado é de 1/16.   *
>
> --
> Esta mensagem foi verificada pelo sistema de antivírus e
> acredita-se estar livre de perigo.

-- 
Esta mensagem foi verificada pelo sistema de antiv�rus e
 acredita-se estar livre de perigo.



Re: [obm-l] Retirar cadastro e recebimento de e-mails

2022-06-14 Por tôpico Ralph Costa Teixeira
Oi, pessoal.

Nao sei se isso ajuda, mas deixa eu tentar: na mensagem que recebi quando
me inscrevi, tem o seguinte:

"Como sair da lista: basta enviar novo e-mail para majord...@mat.puc-rio.br
com o texto:
unsubscribe obm-l
end"

Não sei se estas instruções ainda valem (faz muito tempo!), mas vale a pena
tentar, talvez.

-- Ralph

On Wed, Jun 15, 2022 at 12:33 AM Vitor Sousa 
wrote:

> Por favour, também retire meu email do cadastro de recebimento de emails
>
>
>
> Sent from Mail <https://go.microsoft.com/fwlink/?LinkId=550986> for
> Windows
>
>
> --
> *From:* owner-ob...@mat.puc-rio.br  on behalf
> of luizbga18 
> *Sent:* Tuesday, March 10, 2020 11:05:07 PM
> *To:* obm-l@mat.puc-rio.br 
> *Subject:* Re: [obm-l] Retirar cadastro e recebimento de e-mails
>
> O meu também, por favor.
>
>
>
> Enviado do meu smartphone Samsung Galaxy.
>
>  Mensagem original 
> De: Francisco Nazário 
> Data: 10/03/20 19:18 (GMT-03:00)
> Para: obm-l@mat.puc-rio.br
> Assunto: Re: [obm-l] Retirar cadastro e recebimento de e-mails
>
> Eu também! Por favor!
>
> Em ter., 10 de mar. de 2020 às 01:21, Larissa Fernandes <
> larissafernande2010...@gmail.com> escreveu:
>
> Boa tarde,
> solicito que meu e-mail seja retirado do cadastro de recebimento de
> e-mails.
>
> --
> Esta mensagem foi verificada pelo sistema de antivírus e
> acredita-se estar livre de perigo.
>
>
> --
> Esta mensagem foi verificada pelo sistema de antiv�rus e
> acredita-se estar livre de perigo.
> --
> Esta mensagem foi verificada pelo sistema de antiv�rus e
> acredita-se estar livre de perigo.
>
> --
> Esta mensagem foi verificada pelo sistema de antivírus e
> acredita-se estar livre de perigo.
>

-- 
Esta mensagem foi verificada pelo sistema de antiv�rus e
 acredita-se estar livre de perigo.



RE: [obm-l] Retirar cadastro e recebimento de e-mails

2022-06-14 Por tôpico Vitor Sousa
Por favour, também retire meu email do cadastro de recebimento de emails

Sent from Mail<https://go.microsoft.com/fwlink/?LinkId=550986> for Windows


From: owner-ob...@mat.puc-rio.br  on behalf of 
luizbga18 
Sent: Tuesday, March 10, 2020 11:05:07 PM
To: obm-l@mat.puc-rio.br 
Subject: Re: [obm-l] Retirar cadastro e recebimento de e-mails

O meu também, por favor.



Enviado do meu smartphone Samsung Galaxy.

 Mensagem original 
De: Francisco Nazário 
Data: 10/03/20 19:18 (GMT-03:00)
Para: obm-l@mat.puc-rio.br
Assunto: Re: [obm-l] Retirar cadastro e recebimento de e-mails

Eu também! Por favor!

Em ter., 10 de mar. de 2020 às 01:21, Larissa Fernandes 
mailto:larissafernande2010...@gmail.com>> 
escreveu:
Boa tarde,
solicito que meu e-mail seja retirado do cadastro de recebimento de e-mails.

--
Esta mensagem foi verificada pelo sistema de antivírus e
acredita-se estar livre de perigo.

--
Esta mensagem foi verificada pelo sistema de antiv�rus e
acredita-se estar livre de perigo.
--
Esta mensagem foi verificada pelo sistema de antiv�rus e
acredita-se estar livre de perigo.

-- 
Esta mensagem foi verificada pelo sistema de antiv�rus e
 acredita-se estar livre de perigo.



[obm-l] Re: [obm-l] Série

2022-05-08 Por tôpico Anderson Torres
Em sex., 29 de abr. de 2022 às 23:09, Israel Meireles Chrisostomo
 escreveu:
>
> Alguém aí consegue calcular o limite contida no arquivo desse link logo 
> abaixo?
> https://www.overleaf.com/project/624ee701e9cd2d14986e6f48
>

Link indisponível.

obrigado...

> --
> Israel Meireles Chrisostomo
>
> --
> Esta mensagem foi verificada pelo sistema de antivírus e
> acredita-se estar livre de perigo.

-- 
Esta mensagem foi verificada pelo sistema de antiv�rus e
 acredita-se estar livre de perigo.


=
Instru��es para entrar na lista, sair da lista e usar a lista em
http://www.mat.puc-rio.br/~obmlistas/obm-l.html
=


[obm-l] Re: [obm-l] Sistema de equações lineares

2022-04-18 Por tôpico Anderson Torres
Para de spammar

Em dom., 17 de abr. de 2022 às 01:16, Felippe Coulbert Balbi
 escreveu:
>
> Eu tenho um sistema de equações lineares com 12 variaveis: x1, x2,...,x12. 
> Essas variaveis assumem valor somente no conjunto {0, 1, 1/2, 1/3}.
>
> Eu tenho 8 equações
>
> 4 equações é um sistema linear que pode ser escrito como:
>
> Ax= b
>
> A é uma matriz de 4 linhas e 12 colunas, b é uma matriz de 4 linhas e 1 coluna
>
> As outras 4 equações são:
>
> x1+x2+x3 = 1
>
> x4+x5+x6 = 1
>
> x7+x8+x9 = 1
>
> x10+x11+x12 = 1
>
> Para quais valores de A e b, esse sistema tem solucao? Quando a solucao desse 
> sistema é unica?
>
> Grato,
> Felippe
>
> --
> Esta mensagem foi verificada pelo sistema de antivírus e
> acredita-se estar livre de perigo.

-- 
Esta mensagem foi verificada pelo sistema de antiv�rus e
 acredita-se estar livre de perigo.


=
Instru��es para entrar na lista, sair da lista e usar a lista em
http://www.mat.puc-rio.br/~obmlistas/obm-l.html
=


[obm-l] Re: [obm-l] Dúvida e ajuda.

2022-04-10 Por tôpico Anderson Torres
Em sex., 8 de abr. de 2022 às 11:17, Pedro José  escreveu:
>
> Bom dia!
> Posso concluir que um número representado por uma infinidade de algarismos 
> decimais é racional se e somente se tem um período de repetições desses 
> algarismos?
> A ida é fácil se tiver o período é racional.
> Já a volta não sei se é verdade e se for há como provar?

Acho que dá para fazer isso mais algoritmicamente.

Um número da forma

0,(A)

onde A é um período de k dígitos (por óbvio, zeros à esquerda são
permitidos, como em 0,010101010101...) é essencialmente um racional da
forma A/..9 com k noves - ou melhor escrevendo, (A/(10^k-1)).

Já números da forma 0,B(A) onde B tem m dígitos são a mesma coisa que
10^(-m)*(B+A/(10^k-1)), o que, após simplificar, dá (maçaroca
qualquer)/(10^m*(10^k-1)).

Qualquer racional por definição é da forma p/q com q natural. Bastaria
demonstrar que todo natural q tem um múltiplo da forma
(10^m*(10^k-1)), o que sai de uma aplicação de Euler-Fermat ou mesmo
de casa do pombo.

(Aliás, quem foi o BR que traduziu "princípio do escaninho" para
"princípio de casa de pombo"?)

>
> Meu objetivo primário é saber se:
> 0,123456789112233445566778899111222333444555666777888999... é racional. As 
> reticências se referem ao aumento de mais um algarismo repetido a cada 
> sequência, ou seja a primeira aparição de 1 será 1, a 2a 11 a 3a 111 e assim 
> sucessivamente, o mesmo vale para os demais algarismos.
>
> Alguém poderia me ajudar?
> Grato,
> PJMS
>
> --
> Esta mensagem foi verificada pelo sistema de antivírus e
> acredita-se estar livre de perigo.

-- 
Esta mensagem foi verificada pelo sistema de antiv�rus e
 acredita-se estar livre de perigo.


=
Instru��es para entrar na lista, sair da lista e usar a lista em
http://www.mat.puc-rio.br/~obmlistas/obm-l.html
=


[obm-l] Re: [obm-l] Re: [obm-l] Dúvida e ajuda.

2022-04-08 Por tôpico Pedro José
Grato a todos!
Já, já tenho de voltar ao trabalho.
Depois dou uma olhada.
Mas achei a demonstração usando casa de pombos, simples e prática.
Já que tem de haver um p/q com pp temos w=x+p/q,
onde x é a parte inteira de w/q, então pq e os restos só podem q-1, uma hora tem de
repetir e aí volta a sequência.
Mas saindo do trabalho dou uma olhada.
Mais uma vez, minha gratidão.

Cordialmente,
PJMS



Em sex., 8 de abr. de 2022 às 13:02, Claudio Buffara <
claudio.buff...@gmail.com> escreveu:

> A volta é fácil também: ao calcular a representação decimal de a/b (a e b
> naturais), nas divisões sucessivas por b só existem b-1 restos possíveis
> (resto = 0 em alguma etapa implica numa decimal finita) e, portanto, após
> não mais do que b-1 divisões, um resto vai se repetir, marcando o início de
> um novo período na representação decimal.
>
> Agora, suponha que  X =
> 0,123456789112233445566778899111222333444555666777888999... seja racional.
> Então existirão n e p naturais tais que, a partir da n-ésima casa decimal
> (1/10^n), os algarismos de X vão se repetir numa sequência com período p.
>
> Mas, pela lei de formação de X, vai existir uma sequência de n+p+1
> algarismos iguais a 1, e esta sequência vai começar após a n-ésima casa
> decimal.
> Ou seja, a sequência vai estar incluída na parte periódica da
> representação decimal de X.
> Mas como o período é p, isso implica que a parte periódica teria que
> ser 111..11 (p algarismos 1) ==> contradição à lei de formação de X.
>
> []s,
> Claudio.
>
>
> On Fri, Apr 8, 2022 at 11:17 AM Pedro José  wrote:
>
>> Bom dia!
>> Posso concluir que um número representado por uma infinidade de
>> algarismos decimais é racional se e somente se tem um período de repetições
>> desses algarismos?
>> A ida é fácil se tiver o período é racional.
>> Já a volta não sei se é verdade e se for há como provar?
>>
>> Meu objetivo primário é saber se:
>> 0,123456789112233445566778899111222333444555666777888999... é racional.
>> As reticências se referem ao aumento de mais um algarismo repetido a cada
>> sequência, ou seja a primeira aparição de 1 será 1, a 2a 11 a 3a 111 e
>> assim sucessivamente, o mesmo vale para os demais algarismos.
>>
>> Alguém poderia me ajudar?
>> Grato,
>> PJMS
>>
>> --
>> Esta mensagem foi verificada pelo sistema de antivírus e
>> acredita-se estar livre de perigo.
>
>
> --
> Esta mensagem foi verificada pelo sistema de antivírus e
> acredita-se estar livre de perigo.

-- 
Esta mensagem foi verificada pelo sistema de antiv�rus e
 acredita-se estar livre de perigo.



[obm-l] Re: [obm-l] Dúvida e ajuda.

2022-04-08 Por tôpico Claudio Buffara
A volta é fácil também: ao calcular a representação decimal de a/b (a e b
naturais), nas divisões sucessivas por b só existem b-1 restos possíveis
(resto = 0 em alguma etapa implica numa decimal finita) e, portanto, após
não mais do que b-1 divisões, um resto vai se repetir, marcando o início de
um novo período na representação decimal.

Agora, suponha que  X =
0,123456789112233445566778899111222333444555666777888999... seja racional.
Então existirão n e p naturais tais que, a partir da n-ésima casa decimal
(1/10^n), os algarismos de X vão se repetir numa sequência com período p.

Mas, pela lei de formação de X, vai existir uma sequência de n+p+1
algarismos iguais a 1, e esta sequência vai começar após a n-ésima casa
decimal.
Ou seja, a sequência vai estar incluída na parte periódica da representação
decimal de X.
Mas como o período é p, isso implica que a parte periódica teria que
ser 111..11 (p algarismos 1) ==> contradição à lei de formação de X.

[]s,
Claudio.


On Fri, Apr 8, 2022 at 11:17 AM Pedro José  wrote:

> Bom dia!
> Posso concluir que um número representado por uma infinidade de algarismos
> decimais é racional se e somente se tem um período de repetições desses
> algarismos?
> A ida é fácil se tiver o período é racional.
> Já a volta não sei se é verdade e se for há como provar?
>
> Meu objetivo primário é saber se:
> 0,123456789112233445566778899111222333444555666777888999... é racional. As
> reticências se referem ao aumento de mais um algarismo repetido a cada
> sequência, ou seja a primeira aparição de 1 será 1, a 2a 11 a 3a 111 e
> assim sucessivamente, o mesmo vale para os demais algarismos.
>
> Alguém poderia me ajudar?
> Grato,
> PJMS
>
> --
> Esta mensagem foi verificada pelo sistema de antivírus e
> acredita-se estar livre de perigo.

-- 
Esta mensagem foi verificada pelo sistema de antiv�rus e
 acredita-se estar livre de perigo.



[obm-l] Re: [obm-l] Dúvida e ajuda.

2022-04-08 Por tôpico Caio Costa
Para a volta considere a repetição dividida por 9...9 onde há o mesmo
número de algarismos na repetição e no denominador, incluindo possíveis
zeros à esquerda.

Exemplo

0.3520012001200120012...

= 0.352 + (0012/)/1000

Em sex., 8 de abr. de 2022 11:17, Pedro José  escreveu:

> Bom dia!
> Posso concluir que um número representado por uma infinidade de algarismos
> decimais é racional se e somente se tem um período de repetições desses
> algarismos?
> A ida é fácil se tiver o período é racional.
> Já a volta não sei se é verdade e se for há como provar?
>
> Meu objetivo primário é saber se:
> 0,123456789112233445566778899111222333444555666777888999... é racional. As
> reticências se referem ao aumento de mais um algarismo repetido a cada
> sequência, ou seja a primeira aparição de 1 será 1, a 2a 11 a 3a 111 e
> assim sucessivamente, o mesmo vale para os demais algarismos.
>
> Alguém poderia me ajudar?
> Grato,
> PJMS
>
> --
> Esta mensagem foi verificada pelo sistema de antivírus e
> acredita-se estar livre de perigo.

-- 
Esta mensagem foi verificada pelo sistema de antiv�rus e
 acredita-se estar livre de perigo.



Re: [obm-l] Universidade

2022-03-02 Por tôpico Esaú Gomes
Sim.

Para maiores informações, você pode entrar no site da USP e verificar o
edital de chamada para ver o que mais é necessário além da graduação, e
como é o mecanismo de seleção. Desejo boa sorte e sucesso nessa lida.

On Wed, Mar 2, 2022 at 3:41 PM Israel Meireles Chrisostomo <
israelmchrisost...@gmail.com> wrote:

> Olá pessoal é possível fazer mestrado na usp mesmo eu fazendo graduação em
> Universidade particular?
>
> --
> Esta mensagem foi verificada pelo sistema de antivírus e
> acredita-se estar livre de perigo.

-- 
Esta mensagem foi verificada pelo sistema de antiv�rus e
 acredita-se estar livre de perigo.



Re: [obm-l] Arimetica Diofanto

2022-02-02 Por tôpico Anderson Torres
Em qua., 2 de fev. de 2022 às 00:39, Carlos Gustavo Tamm de Araujo
Moreira  escreveu:
>
> Vou enviar uma solução resumida:
> Se 3^x-5^y=2, vamos testar os menores valores de y: se y=0 então 3^x=3 e x=1. 
> Se y=1 então 3^x=7, o que não dá solução inteira.
> Se y=2 então 3^x=27 e x=3. Vamos então supor y=2+b>2, o que dá 3^x>27, logo 
> x=3+a>3, e podemos escrever a equação como
> 3^3(3^a-1)=5^2(5^b-1). Como a ordem de 3 módulo 5^2 é 20, segue que 20|a, 
> logo 3^a-1 é múltiplo de 3^20-1, e portanto é múltiplo de 61.
> Como a ordem de 5 módulo 61 é 30, segue que 30|b, logo 5^b-1 é múltiplo de 
> 5^30-1, e portanto é múltiplo de 31. Como a ordem de 3
> módulo 31 é 30, segue que 30|a, logo 3^a-1 é múltiplo de 3^30-1, e portanto é 
> múltiplo de 271. Como a ordem de 5 módulo 271 é 27,
> segue que 27|b, e como 30|b segue que 54|b, logo 5^b-1 é múltiplo de 5^54-1, 
> e portanto é múltiplo de 81, absurdo, pois, como 3^a-1 não
> é múltiplo de 3,  3^3(3^a-1) não é múltiplo de 81.

Eu tinha pensado nisso, mas não sabia qual primo usar! Inicialmente
pensei em usar a ideia de ordem para descobrir esse 5^30-1 e
fatorá-lo. Mas acertar esse pingue-pongue de ordens é complicadinho...


> Abraços,
>  Gugu
> é múltiplo de 81
>
> On Fri, Jan 28, 2022 at 5:28 PM Carlos Gustavo Tamm de Araujo Moreira 
>  wrote:
>>
>> Mas acho que lá uma solução está incompleta e as outras duas erradas...
>>
>> On Fri, Jan 28, 2022 at 5:11 PM Gabriel Torkomian  wrote:
>>>
>>> https://artofproblemsolving.com/community/q1h2640462p22841017
>>> Tem no aops
>>>
>>> Em sex., 28 de jan. de 2022 10:32, Israel Meireles Chrisostomo 
>>>  escreveu:

 3^x-5^y=2

 Em sex., 28 de jan. de 2022 09:53, Esaú Gomes  
 escreveu:
>
> E qual a equação?
>
> On Wed, Jan 26, 2022 at 3:33 PM Israel Meireles Chrisostomo 
>  wrote:
>>
>> Olá pessoal, recentimente estava estudando e me deparei com uma equação 
>> diofantina.eu tentei resolve-la mas ñ sei se está correta a solução ou 
>> incompleta, vcs poderiam por favor me ajudar a fechar o argumento?ñ 
>> quero outra solução só quero fazer da minha solução uma solução 
>> top.Tenho a impressão que falta alguma coisa.
>>
>> --
>> Esta mensagem foi verificada pelo sistema de antivírus e
>> acredita-se estar livre de perigo.
>
>
> --
> Esta mensagem foi verificada pelo sistema de antivírus e
> acredita-se estar livre de perigo.


 --
 Esta mensagem foi verificada pelo sistema de antivírus e
 acredita-se estar livre de perigo.
>>>
>>>
>>> --
>>> Esta mensagem foi verificada pelo sistema de antivírus e
>>> acredita-se estar livre de perigo.
>
>
> --
> Esta mensagem foi verificada pelo sistema de antivírus e
> acredita-se estar livre de perigo.

-- 
Esta mensagem foi verificada pelo sistema de antiv�rus e
 acredita-se estar livre de perigo.


=
Instru��es para entrar na lista, sair da lista e usar a lista em
http://www.mat.puc-rio.br/~obmlistas/obm-l.html
=


Re: [obm-l] Arimetica Diofanto

2022-02-01 Por tôpico Carlos Gustavo Tamm de Araujo Moreira
Vou enviar uma solução resumida:
Se 3^x-5^y=2, vamos testar os menores valores de y: se y=0 então 3^x=3 e
x=1. Se y=1 então 3^x=7, o que não dá solução inteira.
Se y=2 então 3^x=27 e x=3. Vamos então supor y=2+b>2, o que dá 3^x>27, logo
x=3+a>3, e podemos escrever a equação como
3^3(3^a-1)=5^2(5^b-1). Como a ordem de 3 módulo 5^2 é 20, segue que 20|a,
logo 3^a-1 é múltiplo de 3^20-1, e portanto é múltiplo de 61.
Como a ordem de 5 módulo 61 é 30, segue que 30|b, logo 5^b-1 é múltiplo de
5^30-1, e portanto é múltiplo de 31. Como a ordem de 3
módulo 31 é 30, segue que 30|a, logo 3^a-1 é múltiplo de 3^30-1, e portanto
é múltiplo de 271. Como a ordem de 5 módulo 271 é 27,
segue que 27|b, e como 30|b segue que 54|b, logo 5^b-1 é múltiplo de
5^54-1, e portanto é múltiplo de 81, absurdo, pois, como 3^a-1 não
é múltiplo de 3,  3^3(3^a-1) não é múltiplo de 81.
Abraços,
 Gugu
é múltiplo de 81

On Fri, Jan 28, 2022 at 5:28 PM Carlos Gustavo Tamm de Araujo Moreira <
g...@impa.br> wrote:

> Mas acho que lá uma solução está incompleta e as outras duas erradas...
>
> On Fri, Jan 28, 2022 at 5:11 PM Gabriel Torkomian 
> wrote:
>
>> https://artofproblemsolving.com/community/q1h2640462p22841017
>> Tem no aops
>>
>> Em sex., 28 de jan. de 2022 10:32, Israel Meireles Chrisostomo <
>> israelmchrisost...@gmail.com> escreveu:
>>
>>> 3^x-5^y=2
>>>
>>> Em sex., 28 de jan. de 2022 09:53, Esaú Gomes 
>>> escreveu:
>>>
 E qual a equação?

 On Wed, Jan 26, 2022 at 3:33 PM Israel Meireles Chrisostomo <
 israelmchrisost...@gmail.com> wrote:

> Olá pessoal, recentimente estava estudando e me deparei com uma
> equação diofantina.eu tentei resolve-la mas ñ sei se está correta a
> solução ou incompleta, vcs poderiam por favor me ajudar a fechar o
> argumento?ñ quero outra solução só quero fazer da minha solução uma 
> solução
> top.Tenho a impressão que falta alguma coisa.
>
> --
> Esta mensagem foi verificada pelo sistema de antivírus e
> acredita-se estar livre de perigo.


 --
 Esta mensagem foi verificada pelo sistema de antivírus e
 acredita-se estar livre de perigo.
>>>
>>>
>>> --
>>> Esta mensagem foi verificada pelo sistema de antivírus e
>>> acredita-se estar livre de perigo.
>>
>>
>> --
>> Esta mensagem foi verificada pelo sistema de antivírus e
>> acredita-se estar livre de perigo.
>
>

-- 
Esta mensagem foi verificada pelo sistema de antiv�rus e
 acredita-se estar livre de perigo.



Re: [obm-l] Re: Polinomio

2022-01-29 Por tôpico Israel Meireles Chrisostomo
Muito obrigado pessoal

Em sáb., 29 de jan. de 2022 19:14, Artur Costa Steiner <
artur.costa.stei...@gmail.com> escreveu:

> Creio que vc se refere a polinômios reais.
>
> Se P tiver grau par positivo  então:
>   Se o coeficiente líder for positivo, P tem  um mínimo global. Se for
> negativo,  P tem um máximo global.
>
> Se P tiver  grau ímpar, P não tem mínimo nem máximo globais.
>
> Limitado inferior e superiormente, só se P for constante
>
> Artur
>
>
>
> Em sáb., 29 de jan. de 2022 às 18:41, Esdras Muniz <
> esdrasmunizm...@gmail.com> escreveu:
>
>> O único polinômio limitado é o constante.
>>
>> Em sáb, 29 de jan de 2022 14:03, Carlos Juarez <
>> carlosjuarezmart...@gmail.com> escreveu:
>>
>>> k=p(c)+1 não vale sempre?
>>>
>>> Em sáb, 29 de jan de 2022 09:27, Israel Meireles Chrisostomo <
>>> israelmchrisost...@gmail.com> escreveu:
>>>
 Desculpe me o que eu quis dizer é que dado um c real existe um k
 positivo tal que p(c)>>>
 Em sáb., 29 de jan. de 2022 09:12, Israel Meireles Chrisostomo <
 israelmchrisost...@gmail.com> escreveu:

> Olá pessoal.Eu gostaria de saber se um polinomio é limitado, isto é,
> dado P(x) existe um k positivo tal que P(x)

 --
 Esta mensagem foi verificada pelo sistema de antivírus e
 acredita-se estar livre de perigo.
>>>
>>>
>>> --
>>> Esta mensagem foi verificada pelo sistema de antivírus e
>>> acredita-se estar livre de perigo.
>>
>>
>> --
>> Esta mensagem foi verificada pelo sistema de antivírus e
>> acredita-se estar livre de perigo.
>
>
> --
> Esta mensagem foi verificada pelo sistema de antivírus e
> acredita-se estar livre de perigo.

-- 
Esta mensagem foi verificada pelo sistema de antiv�rus e
 acredita-se estar livre de perigo.



Re: [obm-l] Re: Polinomio

2022-01-29 Por tôpico Artur Costa Steiner
Creio que vc se refere a polinômios reais.

Se P tiver grau par positivo  então:
  Se o coeficiente líder for positivo, P tem  um mínimo global. Se for
negativo,  P tem um máximo global.

Se P tiver  grau ímpar, P não tem mínimo nem máximo globais.

Limitado inferior e superiormente, só se P for constante

Artur



Em sáb., 29 de jan. de 2022 às 18:41, Esdras Muniz <
esdrasmunizm...@gmail.com> escreveu:

> O único polinômio limitado é o constante.
>
> Em sáb, 29 de jan de 2022 14:03, Carlos Juarez <
> carlosjuarezmart...@gmail.com> escreveu:
>
>> k=p(c)+1 não vale sempre?
>>
>> Em sáb, 29 de jan de 2022 09:27, Israel Meireles Chrisostomo <
>> israelmchrisost...@gmail.com> escreveu:
>>
>>> Desculpe me o que eu quis dizer é que dado um c real existe um k
>>> positivo tal que p(c)>>
>>> Em sáb., 29 de jan. de 2022 09:12, Israel Meireles Chrisostomo <
>>> israelmchrisost...@gmail.com> escreveu:
>>>
 Olá pessoal.Eu gostaria de saber se um polinomio é limitado, isto é,
 dado P(x) existe um k positivo tal que P(x)>>>
>>>
>>> --
>>> Esta mensagem foi verificada pelo sistema de antivírus e
>>> acredita-se estar livre de perigo.
>>
>>
>> --
>> Esta mensagem foi verificada pelo sistema de antivírus e
>> acredita-se estar livre de perigo.
>
>
> --
> Esta mensagem foi verificada pelo sistema de antivírus e
> acredita-se estar livre de perigo.

-- 
Esta mensagem foi verificada pelo sistema de antiv�rus e
 acredita-se estar livre de perigo.



Re: [obm-l] Re: Polinomio

2022-01-29 Por tôpico Esdras Muniz
O único polinômio limitado é o constante.

Em sáb, 29 de jan de 2022 14:03, Carlos Juarez <
carlosjuarezmart...@gmail.com> escreveu:

> k=p(c)+1 não vale sempre?
>
> Em sáb, 29 de jan de 2022 09:27, Israel Meireles Chrisostomo <
> israelmchrisost...@gmail.com> escreveu:
>
>> Desculpe me o que eu quis dizer é que dado um c real existe um k positivo
>> tal que p(c)>
>> Em sáb., 29 de jan. de 2022 09:12, Israel Meireles Chrisostomo <
>> israelmchrisost...@gmail.com> escreveu:
>>
>>> Olá pessoal.Eu gostaria de saber se um polinomio é limitado, isto é,
>>> dado P(x) existe um k positivo tal que P(x)>>
>>
>> --
>> Esta mensagem foi verificada pelo sistema de antivírus e
>> acredita-se estar livre de perigo.
>
>
> --
> Esta mensagem foi verificada pelo sistema de antivírus e
> acredita-se estar livre de perigo.

-- 
Esta mensagem foi verificada pelo sistema de antiv�rus e
 acredita-se estar livre de perigo.



Re: [obm-l] Re: Polinomio

2022-01-29 Por tôpico Carlos Juarez
k=p(c)+1 não vale sempre?

Em sáb, 29 de jan de 2022 09:27, Israel Meireles Chrisostomo <
israelmchrisost...@gmail.com> escreveu:

> Desculpe me o que eu quis dizer é que dado um c real existe um k positivo
> tal que p(c)
> Em sáb., 29 de jan. de 2022 09:12, Israel Meireles Chrisostomo <
> israelmchrisost...@gmail.com> escreveu:
>
>> Olá pessoal.Eu gostaria de saber se um polinomio é limitado, isto é, dado
>> P(x) existe um k positivo tal que P(x)>
>
> --
> Esta mensagem foi verificada pelo sistema de antivírus e
> acredita-se estar livre de perigo.

-- 
Esta mensagem foi verificada pelo sistema de antiv�rus e
 acredita-se estar livre de perigo.



Re: [obm-l] Arimetica Diofanto

2022-01-28 Por tôpico Anderson Torres
Em qua., 26 de jan. de 2022 às 15:33, Israel Meireles Chrisostomo
 escreveu:
>
> Olá pessoal, recentimente estava estudando e me deparei com uma equação 
> diofantina.eu tentei resolve-la mas ñ sei se está correta a solução ou 
> incompleta, vcs poderiam por favor me ajudar a fechar o argumento?ñ quero 
> outra solução só quero fazer da minha solução uma solução top.Tenho a 
> impressão que falta alguma coisa.
>

Que equação?

> --
> Esta mensagem foi verificada pelo sistema de antivírus e
> acredita-se estar livre de perigo.

-- 
Esta mensagem foi verificada pelo sistema de antiv�rus e
 acredita-se estar livre de perigo.


=
Instru��es para entrar na lista, sair da lista e usar a lista em
http://www.mat.puc-rio.br/~obmlistas/obm-l.html
=


Re: [obm-l] Arimetica Diofanto

2022-01-28 Por tôpico Carlos Gustavo Tamm de Araujo Moreira
Mas acho que lá uma solução está incompleta e as outras duas erradas...

On Fri, Jan 28, 2022 at 5:11 PM Gabriel Torkomian  wrote:

> https://artofproblemsolving.com/community/q1h2640462p22841017
> Tem no aops
>
> Em sex., 28 de jan. de 2022 10:32, Israel Meireles Chrisostomo <
> israelmchrisost...@gmail.com> escreveu:
>
>> 3^x-5^y=2
>>
>> Em sex., 28 de jan. de 2022 09:53, Esaú Gomes 
>> escreveu:
>>
>>> E qual a equação?
>>>
>>> On Wed, Jan 26, 2022 at 3:33 PM Israel Meireles Chrisostomo <
>>> israelmchrisost...@gmail.com> wrote:
>>>
 Olá pessoal, recentimente estava estudando e me deparei com uma equação
 diofantina.eu tentei resolve-la mas ñ sei se está correta a solução ou
 incompleta, vcs poderiam por favor me ajudar a fechar o argumento?ñ quero
 outra solução só quero fazer da minha solução uma solução top.Tenho a
 impressão que falta alguma coisa.

 --
 Esta mensagem foi verificada pelo sistema de antivírus e
 acredita-se estar livre de perigo.
>>>
>>>
>>> --
>>> Esta mensagem foi verificada pelo sistema de antivírus e
>>> acredita-se estar livre de perigo.
>>
>>
>> --
>> Esta mensagem foi verificada pelo sistema de antivírus e
>> acredita-se estar livre de perigo.
>
>
> --
> Esta mensagem foi verificada pelo sistema de antivírus e
> acredita-se estar livre de perigo.

-- 
Esta mensagem foi verificada pelo sistema de antiv�rus e
 acredita-se estar livre de perigo.



Re: [obm-l] Arimetica Diofanto

2022-01-28 Por tôpico Gabriel Torkomian
https://artofproblemsolving.com/community/q1h2640462p22841017
Tem no aops

Em sex., 28 de jan. de 2022 10:32, Israel Meireles Chrisostomo <
israelmchrisost...@gmail.com> escreveu:

> 3^x-5^y=2
>
> Em sex., 28 de jan. de 2022 09:53, Esaú Gomes 
> escreveu:
>
>> E qual a equação?
>>
>> On Wed, Jan 26, 2022 at 3:33 PM Israel Meireles Chrisostomo <
>> israelmchrisost...@gmail.com> wrote:
>>
>>> Olá pessoal, recentimente estava estudando e me deparei com uma equação
>>> diofantina.eu tentei resolve-la mas ñ sei se está correta a solução ou
>>> incompleta, vcs poderiam por favor me ajudar a fechar o argumento?ñ quero
>>> outra solução só quero fazer da minha solução uma solução top.Tenho a
>>> impressão que falta alguma coisa.
>>>
>>> --
>>> Esta mensagem foi verificada pelo sistema de antivírus e
>>> acredita-se estar livre de perigo.
>>
>>
>> --
>> Esta mensagem foi verificada pelo sistema de antivírus e
>> acredita-se estar livre de perigo.
>
>
> --
> Esta mensagem foi verificada pelo sistema de antivírus e
> acredita-se estar livre de perigo.

-- 
Esta mensagem foi verificada pelo sistema de antiv�rus e
 acredita-se estar livre de perigo.



Re: [obm-l] Arimetica Diofanto

2022-01-28 Por tôpico Israel Meireles Chrisostomo
3^x-5^y=2

Em sex., 28 de jan. de 2022 09:53, Esaú Gomes 
escreveu:

> E qual a equação?
>
> On Wed, Jan 26, 2022 at 3:33 PM Israel Meireles Chrisostomo <
> israelmchrisost...@gmail.com> wrote:
>
>> Olá pessoal, recentimente estava estudando e me deparei com uma equação
>> diofantina.eu tentei resolve-la mas ñ sei se está correta a solução ou
>> incompleta, vcs poderiam por favor me ajudar a fechar o argumento?ñ quero
>> outra solução só quero fazer da minha solução uma solução top.Tenho a
>> impressão que falta alguma coisa.
>>
>> --
>> Esta mensagem foi verificada pelo sistema de antivírus e
>> acredita-se estar livre de perigo.
>
>
> --
> Esta mensagem foi verificada pelo sistema de antivírus e
> acredita-se estar livre de perigo.

-- 
Esta mensagem foi verificada pelo sistema de antiv�rus e
 acredita-se estar livre de perigo.



Re: [obm-l] Arimetica Diofanto

2022-01-28 Por tôpico Esaú Gomes
E qual a equação?

On Wed, Jan 26, 2022 at 3:33 PM Israel Meireles Chrisostomo <
israelmchrisost...@gmail.com> wrote:

> Olá pessoal, recentimente estava estudando e me deparei com uma equação
> diofantina.eu tentei resolve-la mas ñ sei se está correta a solução ou
> incompleta, vcs poderiam por favor me ajudar a fechar o argumento?ñ quero
> outra solução só quero fazer da minha solução uma solução top.Tenho a
> impressão que falta alguma coisa.
>
> --
> Esta mensagem foi verificada pelo sistema de antivírus e
> acredita-se estar livre de perigo.

-- 
Esta mensagem foi verificada pelo sistema de antiv�rus e
 acredita-se estar livre de perigo.



[obm-l] Re: [obm-l] Re: [obm-l] Experiência mental

2022-01-26 Por tôpico Israel Meireles Chrisostomo
Muito obrigado, bem que eu achei meio estranho ninguém ter percebido kkk

Em qua., 26 de jan. de 2022 10:40, Fernando Villar 
escreveu:

>
> Olá Israel. A primeira vez que vi também tive essa impressão, mas ao ler o
> livro descobri que os seres de Planolandia identificam uns aos outros por
> meio do tato, identificando os ângulos. A referência ao formato é para
> estabelecer uma correspondência com o que conhecemos.
> O livro é muito bom, propõe uma discussão da hierarquia social vigente no
> século XIX.
>
> Abraços e uma excelente semana para você.
>
> Fernando Villar
>
>
> Em qua., 26 de jan. de 2022 às 09:20, Israel Meireles Chrisostomo <
> israelmchrisost...@gmail.com> escreveu:
>
>> olá pessoal, eu estava no youtube assistindo a um vídeo de Carl Sagan
>> falando sobre a planolandia.Para quem ñ sabe, a planolandia é uma
>> experiencia mental que considera seres em universos planos.Sem delongas, eu
>> refuto a ideia de que os habitantes de tal universo enxerguem figuras
>> geométricas planas, como o triângulo, quadrado, retângulo, circulo...O
>> argumento é bem simples: só é possível ver figuras planas fora do plano,
>> mas quem está no plano só consegue ver linhas retas.
>>
>> --
>> Esta mensagem foi verificada pelo sistema de antivírus e
>> acredita-se estar livre de perigo.
>
> --
> Fernando Villar
>
>
> --
> Esta mensagem foi verificada pelo sistema de antivírus e
> acredita-se estar livre de perigo.

-- 
Esta mensagem foi verificada pelo sistema de antiv�rus e
 acredita-se estar livre de perigo.



[obm-l] Re: [obm-l] Experiência mental

2022-01-26 Por tôpico Fernando Villar
Olá Israel. A primeira vez que vi também tive essa impressão, mas ao ler o
livro descobri que os seres de Planolandia identificam uns aos outros por
meio do tato, identificando os ângulos. A referência ao formato é para
estabelecer uma correspondência com o que conhecemos.
O livro é muito bom, propõe uma discussão da hierarquia social vigente no
século XIX.

Abraços e uma excelente semana para você.

Fernando Villar


Em qua., 26 de jan. de 2022 às 09:20, Israel Meireles Chrisostomo <
israelmchrisost...@gmail.com> escreveu:

> olá pessoal, eu estava no youtube assistindo a um vídeo de Carl Sagan
> falando sobre a planolandia.Para quem ñ sabe, a planolandia é uma
> experiencia mental que considera seres em universos planos.Sem delongas, eu
> refuto a ideia de que os habitantes de tal universo enxerguem figuras
> geométricas planas, como o triângulo, quadrado, retângulo, circulo...O
> argumento é bem simples: só é possível ver figuras planas fora do plano,
> mas quem está no plano só consegue ver linhas retas.
>
> --
> Esta mensagem foi verificada pelo sistema de antivírus e
> acredita-se estar livre de perigo.

-- 
Fernando Villar

-- 
Esta mensagem foi verificada pelo sistema de antiv�rus e
 acredita-se estar livre de perigo.



Re: [obm-l]

2022-01-13 Por tôpico Israel Meireles Chrisostomo
Muito obrigado pela gentileza em me responder!

Em qui, 13 de jan de 2022 10:51, Marcos Grilo  escreveu:

> Sim. O livro Curso de Análise Vol.1 do prof. Elon contém uma demonstração
> mais geral: dado um intervalo aberto (a,b), existem um número racional e um
> número irracional em (a,b). Procure no capítulo sobre Números Reais o
> Teorema em que se demonstra que o conjunto Q e IR-Q são ambos densos em IR.
> O prof. Elon parte do axioma "Existe um corpo ordenado completo, chamado de
> números reais".
>
> Marcos Grilo
> Professor Adjunto | DEXA | UEFS
> http://lattes.cnpq.br/2105015661240571
>
>
>
> 
>  Livre
> de vírus. www.avg.com
> .
>
> <#m_5944008406174362581_m_188824641277804530_DAB4FAD8-2DD7-40BB-A1B8-4E2AA1F9FDF2>
>
> Em ter., 11 de jan. de 2022 às 20:55, Israel Meireles Chrisostomo <
> israelmchrisost...@gmail.com> escreveu:
>
>> É possível provar que entre 2 IRRACIONAIS há sempre um racional?o
>> contrário eu sei como fazer
>>
>> --
>> Esta mensagem foi verificada pelo sistema de antivírus e
>> acredita-se estar livre de perigo.
>
>
> --
> Esta mensagem foi verificada pelo sistema de antivírus e
> acredita-se estar livre de perigo.

-- 
Esta mensagem foi verificada pelo sistema de antiv�rus e
 acredita-se estar livre de perigo.



Re: [obm-l]

2022-01-13 Por tôpico Marcos Grilo
Sim. O livro Curso de Análise Vol.1 do prof. Elon contém uma demonstração
mais geral: dado um intervalo aberto (a,b), existem um número racional e um
número irracional em (a,b). Procure no capítulo sobre Números Reais o
Teorema em que se demonstra que o conjunto Q e IR-Q são ambos densos em IR.
O prof. Elon parte do axioma "Existe um corpo ordenado completo, chamado de
números reais".

Marcos Grilo
Professor Adjunto | DEXA | UEFS
http://lattes.cnpq.br/2105015661240571



Livre
de vírus. www.avg.com
.
<#m_188824641277804530_DAB4FAD8-2DD7-40BB-A1B8-4E2AA1F9FDF2>

Em ter., 11 de jan. de 2022 às 20:55, Israel Meireles Chrisostomo <
israelmchrisost...@gmail.com> escreveu:

> É possível provar que entre 2 IRRACIONAIS há sempre um racional?o
> contrário eu sei como fazer
>
> --
> Esta mensagem foi verificada pelo sistema de antivírus e
> acredita-se estar livre de perigo.

-- 
Esta mensagem foi verificada pelo sistema de antiv�rus e
 acredita-se estar livre de perigo.



Re: [obm-l]

2022-01-12 Por tôpico Israel Meireles Chrisostomo
Muito obrigado!

Em qua, 12 de jan de 2022 01:07, Daniel Jelin 
escreveu:

> Tem uma prova famosa, q vale pra dois reais quaisquer a, b, a assim. Existe n natural tal q n*(b-a)>1. Assim, nb-na>1. Logo, existe algum
> inteiro m entre nb e na, de modo que na a
> Em ter., 11 de jan. de 2022 20:46, Israel Meireles Chrisostomo <
> israelmchrisost...@gmail.com> escreveu:
>
>> É possível provar que entre 2 IRRACIONAIS há sempre um racional?o
>> contrário eu sei como fazer
>>
>> --
>> Esta mensagem foi verificada pelo sistema de antivírus e
>> acredita-se estar livre de perigo.
>
>
> --
> Esta mensagem foi verificada pelo sistema de antivírus e
> acredita-se estar livre de perigo.

-- 
Esta mensagem foi verificada pelo sistema de antiv�rus e
 acredita-se estar livre de perigo.



Re: [obm-l]

2022-01-11 Por tôpico Daniel Jelin
Tem uma prova famosa, q vale pra dois reais quaisquer a, b, a1. Assim, nb-na>1. Logo, existe algum
inteiro m entre nb e na, de modo que na escreveu:

> É possível provar que entre 2 IRRACIONAIS há sempre um racional?o
> contrário eu sei como fazer
>
> --
> Esta mensagem foi verificada pelo sistema de antivírus e
> acredita-se estar livre de perigo.

-- 
Esta mensagem foi verificada pelo sistema de antiv�rus e
 acredita-se estar livre de perigo.



[obm-l] Re: [obm-l] Quebra do RSA por solução do problema de fatoração - Eric Campos Bastos Guedes

2022-01-11 Por tôpico Eduardo Guimarães
Chave pública:
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

Em ter., 11 de jan. de 2022 às 11:53, Eduardo Guimarães <
eduardoestudo...@gmail.com> escreveu:

>
> 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
> Desencripta isso.
>
> Em ter., 11 de jan. de 2022 às 11:19, Claudio Buffara <
> claudio.buff...@gmail.com> escreveu:
>
>> Eu diria que a melhor forma de avaliar seu trabalho é testando.
>> Apesar do “desafio RSA” já ter encerrado, os números ainda estão
>> disponíveis.
>> Da uma olhada no verbete “RSA numbers” na Wikipédia.
>>
>> Enviado do meu iPhone
>>
>> > Em 11 de jan. de 2022, à(s) 15:03, Eric Campos Bastos Guedes <
>> ebastosgue...@gmail.com> escreveu:
>> >
>> > 
>> > Proponho um algoritmo para quebrar o RSA. O algoritmo que eu propus
>> antes trabalhava com números muito grandes e por isso podia não funcionar
>> direito. Esse trabalha com números bem menores porque usa módulo N numa
>> etapa. O algoritmo e sua explicação estão no YouTube com o mesmo título
>> desse e-mail. São dois vídeos, o que conta é o mais recente deste ano de
>> 2022.
>> >
>> > QUEBRA DO RSA - ALGORITMO N.2
>> >
>> > PASSO 1: a=3
>> >
>> > inicializando o valor de a
>> >
>> > PASSO 2: N é o inteiro a ser fatoradoÂ
>> >
>> > N é o número usado no RSA. N é o produto de dois números primos
>> grandes não muito próximos.Â
>> >
>> > PASSO 3: M=N^512 (N elevado a 512)
>> >
>> > M é um número grande mas não muito grande. O valor de P não vai
>> ultrapassar muito o valor de M. P é uma variável inteira que acumula
>> fatores primos. Aí você faz MDC(P, N) para tentar fatorar N.
>> >
>> > PASSO 4: a=a+1
>> >
>> > O valor de a é atualizado para a+1, isto é,  é  acrescentado 1 ao
>> valor de a
>> >
>> > PASSO 5: P=aÂ
>> >
>> > O valor de P é inicializadoÂ
>> >
>> > PASSO 6: b = número aleatório entre 0 e 1
>> > PASSO 7: Se b > 1/2 faça c=1 senão faça c=-1
>> >
>> > O objetivo dos passos 6 e 7 é atribuir à variável c um valor que
>> pode ser 1 ou -1. Isso nem precisa ser feito de modo aleatório, mas acho
>> que vai funcionar melhor se for aleatório.Â
>> >
>> > PASSO 8: P=P(P+c)
>> >
>> > É uma atribuição de valor. O novo valor de P passa a ser P(P+c).
>> Note que P+c é relativamente primo com P. Na prática são acrescentados
>> novos fatores primos a P que vai acumular fatores primos.Â
>> >
>> > PASSO 9: Se P < M vá para o PASSO 6
>> >
>> > Esse passo determina um looping para acumular fatores em P.
>> >
>> > PASSO 10: Se MDC(P, N) for diferente de 1 vá para o PASSO 14
>> >
>> > Se MDC(P, N) for diferente de 1 ele pode ser um fator primo de N. Resta
>> verificar se ele não é o próprio N. Isso vai ser feito no PASSO 14.
>> >
>> > PASSO 11: P = Resto da divisão de P por N
>> >
>> > Esse passo é para trabalharmos com números menores.Â
>> >
>> > PASSO 12: Se P < 4 faça P=4
>> >
>> > Talvez esse passo possa ser omitido
>> >
>> > PASSO 13: vá para o PASSO 6
>> >
>> > PASSO 14: Se MDC(P, N)=N vá para o PASSO 4
>> >
>> > Se MDC(P, N) = N não foram encontrados fatores primos e algoritmo
>> recomeça do ponto apropriado.Â
>> >
>> > PASSO 15: MDC(P, N) é fator (primo) de N
>> >
>> > FIM
>> >
>> > Eu fui menção honrosa na Olimpíada Ibero-americana de Matemática
>> Universitária em 2006. Acho que este meu trabalho merece ser avaliado.
>> >
>> >
>> >
>> >
>> > --
>> > Esta mensagem foi verificada pelo sistema de antivírus e
>> > acredita-se estar livre de perigo.
>>
>> --
>> Esta mensagem foi verificada pelo sistema de antivírus e
>>  acredita-se estar livre de perigo.
>>
>>
>> =
>> Instru�ões para entrar na 

[obm-l] Re: [obm-l] Quebra do RSA por solução do problema de fatoração - Eric Campos Bastos Guedes

2022-01-11 Por tôpico Eduardo Guimarães
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
Desencripta isso.

Em ter., 11 de jan. de 2022 às 11:19, Claudio Buffara <
claudio.buff...@gmail.com> escreveu:

> Eu diria que a melhor forma de avaliar seu trabalho é testando.
> Apesar do “desafio RSA” já ter encerrado, os números ainda estão
> disponíveis.
> Da uma olhada no verbete “RSA numbers” na Wikipédia.
>
> Enviado do meu iPhone
>
> > Em 11 de jan. de 2022, à(s) 15:03, Eric Campos Bastos Guedes <
> ebastosgue...@gmail.com> escreveu:
> >
> > 
> > Proponho um algoritmo para quebrar o RSA. O algoritmo que eu propus
> antes trabalhava com números muito grandes e por isso podia não funcionar
> direito. Esse trabalha com números bem menores porque usa módulo N numa
> etapa. O algoritmo e sua explicação estão no YouTube com o mesmo título
> desse e-mail. São dois vídeos, o que conta é o mais recente deste ano de
> 2022.
> >
> > QUEBRA DO RSA - ALGORITMO N.2
> >
> > PASSO 1: a=3
> >
> > inicializando o valor de a
> >
> > PASSO 2: N é o inteiro a ser fatoradoÂ
> >
> > N é o número usado no RSA. N é o produto de dois números primos
> grandes não muito próximos.Â
> >
> > PASSO 3: M=N^512 (N elevado a 512)
> >
> > M é um número grande mas não muito grande. O valor de P não vai
> ultrapassar muito o valor de M. P é uma variável inteira que acumula
> fatores primos. Aí você faz MDC(P, N) para tentar fatorar N.
> >
> > PASSO 4: a=a+1
> >
> > O valor de a é atualizado para a+1, isto é,  é  acrescentado 1 ao
> valor de a
> >
> > PASSO 5: P=aÂ
> >
> > O valor de P é inicializadoÂ
> >
> > PASSO 6: b = número aleatório entre 0 e 1
> > PASSO 7: Se b > 1/2 faça c=1 senão faça c=-1
> >
> > O objetivo dos passos 6 e 7 é atribuir à variável c um valor que pode
> ser 1 ou -1. Isso nem precisa ser feito de modo aleatório, mas acho que
> vai funcionar melhor se for aleatório.Â
> >
> > PASSO 8: P=P(P+c)
> >
> > É uma atribuição de valor. O novo valor de P passa a ser P(P+c). Note
> que P+c é relativamente primo com P. Na prática são acrescentados novos
> fatores primos a P que vai acumular fatores primos.Â
> >
> > PASSO 9: Se P < M vá para o PASSO 6
> >
> > Esse passo determina um looping para acumular fatores em P.
> >
> > PASSO 10: Se MDC(P, N) for diferente de 1 vá para o PASSO 14
> >
> > Se MDC(P, N) for diferente de 1 ele pode ser um fator primo de N. Resta
> verificar se ele não é o próprio N. Isso vai ser feito no PASSO 14.
> >
> > PASSO 11: P = Resto da divisão de P por N
> >
> > Esse passo é para trabalharmos com números menores.Â
> >
> > PASSO 12: Se P < 4 faça P=4
> >
> > Talvez esse passo possa ser omitido
> >
> > PASSO 13: vá para o PASSO 6
> >
> > PASSO 14: Se MDC(P, N)=N vá para o PASSO 4
> >
> > Se MDC(P, N) = N não foram encontrados fatores primos e algoritmo
> recomeça do ponto apropriado.Â
> >
> > PASSO 15: MDC(P, N) é fator (primo) de N
> >
> > FIM
> >
> > Eu fui menção honrosa na Olimpíada Ibero-americana de Matemática
> Universitária em 2006. Acho que este meu trabalho merece ser avaliado.
> >
> >
> >
> >
> > --
> > Esta mensagem foi verificada pelo sistema de antivírus e
> > acredita-se estar livre de perigo.
>
> --
> Esta mensagem foi verificada pelo sistema de antivírus e
>  acredita-se estar livre de perigo.
>
>
> =
> Instru�ões para entrar na lista, sair da lista e usar a lista em
> http://www.mat.puc-rio.br/~obmlistas/obm-l.html
> =
>

-- 
Esta mensagem foi verificada pelo sistema de antiv�rus e
 acredita-se estar livre de perigo.



Re: [obm-l] Quebra do RSA por solução do problema de fatoração - Eric Campos Bastos Guedes

2022-01-11 Por tôpico Claudio Buffara
Eu diria que a melhor forma de avaliar seu trabalho é testando.
Apesar do “desafio RSA” já ter encerrado, os números ainda estão disponíveis.
Da uma olhada no verbete “RSA numbers” na Wikipédia.

Enviado do meu iPhone

> Em 11 de jan. de 2022, à(s) 15:03, Eric Campos Bastos Guedes 
>  escreveu:
> 
> 
> Proponho um algoritmo para quebrar o RSA. O algoritmo que eu propus antes 
> trabalhava com números muito grandes e por isso podia não funcionar 
> direito. Esse trabalha com números bem menores porque usa módulo N numa 
> etapa. O algoritmo e sua explicação estão no YouTube com o mesmo título 
> desse e-mail. São dois vídeos, o que conta é o mais recente deste ano de 
> 2022.
> 
> QUEBRA DO RSA - ALGORITMO N.2
> 
> PASSO 1: a=3
> 
> inicializando o valor de a
> 
> PASSO 2: N é o inteiro a ser fatorado 
> 
> N é o número usado no RSA. N é o produto de dois números primos grandes 
> não muito próximos. 
> 
> PASSO 3: M=N^512 (N elevado a 512)
> 
> M é um número grande mas não muito grande. O valor de P não vai 
> ultrapassar muito o valor de M. P é uma variável inteira que acumula 
> fatores primos. Aí você faz MDC(P, N) para tentar fatorar N.
> 
> PASSO 4: a=a+1
> 
> O valor de a é atualizado para a+1, isto é,  é  acrescentado 1 ao valor 
> de a
> 
> PASSO 5: P=a 
> 
> O valor de P é inicializado 
> 
> PASSO 6: b = número aleatório entre 0 e 1
> PASSO 7: Se b > 1/2 faça c=1 senão faça c=-1
> 
> O objetivo dos passos 6 e 7 é atribuir à variável c um valor que pode ser 
> 1 ou -1. Isso nem precisa ser feito de modo aleatório, mas acho que vai 
> funcionar melhor se for aleatório. 
> 
> PASSO 8: P=P(P+c)
> 
> É uma atribuição de valor. O novo valor de P passa a ser P(P+c). Note que 
> P+c é relativamente primo com P. Na prática são acrescentados novos 
> fatores primos a P que vai acumular fatores primos. 
> 
> PASSO 9: Se P < M vá para o PASSO 6
> 
> Esse passo determina um looping para acumular fatores em P.
> 
> PASSO 10: Se MDC(P, N) for diferente de 1 vá para o PASSO 14
> 
> Se MDC(P, N) for diferente de 1 ele pode ser um fator primo de N. Resta 
> verificar se ele não é o próprio N. Isso vai ser feito no PASSO 14.
> 
> PASSO 11: P = Resto da divisão de P por N
> 
> Esse passo é para trabalharmos com números menores. 
> 
> PASSO 12: Se P < 4 faça P=4
> 
> Talvez esse passo possa ser omitido
> 
> PASSO 13: vá para o PASSO 6
> 
> PASSO 14: Se MDC(P, N)=N vá para o PASSO 4
> 
> Se MDC(P, N) = N não foram encontrados fatores primos e algoritmo recomeça 
> do ponto apropriado. 
> 
> PASSO 15: MDC(P, N) é fator (primo) de N
> 
> FIM
> 
> Eu fui menção honrosa na Olimpíada Ibero-americana de Matemática 
> Universitária em 2006. Acho que este meu trabalho merece ser avaliado.
> 
> 
> 
> 
> -- 
> Esta mensagem foi verificada pelo sistema de antivírus e 
> acredita-se estar livre de perigo.

-- 
Esta mensagem foi verificada pelo sistema de antiv�rus e
 acredita-se estar livre de perigo.


=
Instru��es para entrar na lista, sair da lista e usar a lista em
http://www.mat.puc-rio.br/~obmlistas/obm-l.html
=


[obm-l] Re: [obm-l] Quebra do RSA por solução do problema de fatoração - Eric Campos Bastos Guedes

2022-01-11 Por tôpico Esdras Muniz
Bem, eu não sou especialista no assunto, mas uma observação óbvia é que
para tentar na força bruta fatorar N, vc vai usar no máximo 2√N/ln(N)
divisões (pelo teorema dos números primos). Uma coisa bastante interessante
seria vc mostrar que seu algoritmo faz menos interação que isso, ou ainda
que na maioria dos casos ele é melhor.

Em ter, 11 de jan de 2022 10:03, Eric Campos Bastos Guedes <
ebastosgue...@gmail.com> escreveu:

> Proponho um algoritmo para quebrar o RSA. O algoritmo que eu propus antes
> trabalhava com números muito grandes e por isso podia não funcionar
> direito. Esse trabalha com números bem menores porque usa módulo N numa
> etapa. O algoritmo e sua explicação estão no YouTube com o mesmo título
> desse e-mail. São dois vídeos, o que conta é o mais recente deste ano de
> 2022.
>
> QUEBRA DO RSA - ALGORITMO N.2
>
> PASSO 1: a=3
>
> inicializando o valor de a
>
> PASSO 2: N é o inteiro a ser fatorado
>
> N é o número usado no RSA. N é o produto de dois números primos grandes
> não muito próximos.
>
> PASSO 3: M=N^512 (N elevado a 512)
>
> M é um número grande mas não muito grande. O valor de P não vai
> ultrapassar muito o valor de M. P é uma variável inteira que acumula
> fatores primos. Aí você faz MDC(P, N) para tentar fatorar N.
>
> PASSO 4: a=a+1
>
> O valor de a é atualizado para a+1, isto é,  é  acrescentado 1 ao valor de
> a
>
> PASSO 5: P=a
>
> O valor de P é inicializado
>
> PASSO 6: b = número aleatório entre 0 e 1
> PASSO 7: Se b > 1/2 faça c=1 senão faça c=-1
>
> O objetivo dos passos 6 e 7 é atribuir à variável c um valor que pode ser
> 1 ou -1. Isso nem precisa ser feito de modo aleatório, mas acho que vai
> funcionar melhor se for aleatório.
>
> PASSO 8: P=P(P+c)
>
> É uma atribuição de valor. O novo valor de P passa a ser P(P+c). Note que
> P+c é relativamente primo com P. Na prática são acrescentados novos fatores
> primos a P que vai acumular fatores primos.
>
> PASSO 9: Se P < M vá para o PASSO 6
>
> Esse passo determina um looping para acumular fatores em P.
>
> PASSO 10: Se MDC(P, N) for diferente de 1 vá para o PASSO 14
>
> Se MDC(P, N) for diferente de 1 ele pode ser um fator primo de N. Resta
> verificar se ele não é o próprio N. Isso vai ser feito no PASSO 14.
>
> PASSO 11: P = Resto da divisão de P por N
>
> Esse passo é para trabalharmos com números menores.
>
> PASSO 12: Se P < 4 faça P=4
>
> Talvez esse passo possa ser omitido
>
> PASSO 13: vá para o PASSO 6
>
> PASSO 14: Se MDC(P, N)=N vá para o PASSO 4
>
> Se MDC(P, N) = N não foram encontrados fatores primos e algoritmo recomeça
> do ponto apropriado.
>
> PASSO 15: MDC(P, N) é fator (primo) de N
>
> FIM
>
> Eu fui menção honrosa na Olimpíada Ibero-americana de Matemática
> Universitária em 2006. Acho que este meu trabalho merece ser avaliado.
>
>
>
>
> --
> Esta mensagem foi verificada pelo sistema de antivírus e
> acredita-se estar livre de perigo.

-- 
Esta mensagem foi verificada pelo sistema de antiv�rus e
 acredita-se estar livre de perigo.



Re: [obm-l] Irracional

2022-01-09 Por tôpico Israel Meireles Chrisostomo
Obrigado pela correção.o que eu quis dizer é que entre dois racionais
existem duas sequências de números irracionais, uma decrescente e outra
crescente, com quantos termos se desejar.

Em dom, 9 de jan de 2022 13:03, Anderson Torres <
torres.anderson...@gmail.com> escreveu:

> Em sex., 7 de jan. de 2022 às 09:21, Israel Meireles Chrisostomo
>  escreveu:
> >
> > Olá acho que consigo provar o seguinte teorema: entre dois racionais
> existe uma sequência de números irracionais, decrescente e crescente, com
> quantos termos se desejar.Alguém aí se interessa por esse problema?
>
> se é crescente E descrescente ao mesmo tempo, então é uma sequência
> constante.
>
> >
> > --
> > Esta mensagem foi verificada pelo sistema de antivírus e
> > acredita-se estar livre de perigo.
>
> --
> Esta mensagem foi verificada pelo sistema de antivírus e
>  acredita-se estar livre de perigo.
>
>
> =
> Instru�ões para entrar na lista, sair da lista e usar a lista em
> http://www.mat.puc-rio.br/~obmlistas/obm-l.html
> =
>

-- 
Esta mensagem foi verificada pelo sistema de antiv�rus e
 acredita-se estar livre de perigo.



Re: [obm-l] Irracional

2022-01-09 Por tôpico Anderson Torres
Em sex., 7 de jan. de 2022 às 09:21, Israel Meireles Chrisostomo
 escreveu:
>
> Olá acho que consigo provar o seguinte teorema: entre dois racionais existe 
> uma sequência de números irracionais, decrescente e crescente, com quantos 
> termos se desejar.Alguém aí se interessa por esse problema?

se é crescente E descrescente ao mesmo tempo, então é uma sequência constante.

>
> --
> Esta mensagem foi verificada pelo sistema de antivírus e
> acredita-se estar livre de perigo.

-- 
Esta mensagem foi verificada pelo sistema de antiv�rus e
 acredita-se estar livre de perigo.


=
Instru��es para entrar na lista, sair da lista e usar a lista em
http://www.mat.puc-rio.br/~obmlistas/obm-l.html
=


[obm-l] Re: [obm-l] Número de matrizes 0-1

2022-01-05 Por tôpico Anderson Torres
Em seg., 20 de dez. de 2021 às 18:58, Claudio Buffara
 escreveu:
>
> Num outro grupo, propuseram o problema de achar o número de matrizes 4x4 com 
> entradas em {0,1} e cujo determinante seja ímpar.
> Olhando mod 2, isso é equivalente a achar o número de matrizes 4x4 
> invertíveis com entradas em Z2 (o corpo com 2 elementos).
> Este é um resultado conhecido: o número de tais matrizes é 
> (2^4-1)(2^4-2)(2^4-2^2)(2^4-2^3) = 15*14*12*8 = 20.160.
> Provar isso pode ser um bom problema pra quem não conhece o "truque" (que 
> nada mais é do que usar uma caracterização alternativa de "matriz 
> invertível").
>
> Daí surgiram duas dúvidas:
> 1) Quais os valores possíveis do determinante desta matriz?
> 2) Quantas matrizes existem com cada valor possível do determinante?

Eu desconfio fortemente que não existe forma mais fácil do que fazer na raça.

>
> Não é difícil fazer um programa de computador pra calcular isso (afinal, 
> existem apenas 2^16 = 65.536 matrizes 4x4 com entradas em {0,1}).
> Mas será que há uma forma "esperta" de calcular isso?
> E que seja generalizável pra matrizes nxn?
>
> []s,
> Claudio.
>
> --
> Esta mensagem foi verificada pelo sistema de antivírus e
> acredita-se estar livre de perigo.

-- 
Esta mensagem foi verificada pelo sistema de antiv�rus e
 acredita-se estar livre de perigo.


=
Instru��es para entrar na lista, sair da lista e usar a lista em
http://www.mat.puc-rio.br/~obmlistas/obm-l.html
=


[obm-l] Re: [obm-l] como saber se uma moeda é viciada

2022-01-05 Por tôpico Anderson Torres
Em ter., 21 de dez. de 2021 às 09:16, jamil dasilva
 escreveu:
>
> Se em cem lançamentos de uma moeda a probabilidade de sair qualquer um dos 
> 2^100 resultados é a mesma,
> seria correto dizer que a moeda seria viciada se o resultado fosse CARA em 
> todas as cem vezes ?

Isso me parece bastante estranho.

Isso seria equivalente a dizer "a moeda é viciada se o resultado é
idêntico aos cem primeiros dígitos após a vírgula da expansão binária
de pi^2/6".

Eu suspeito que, se existe uma teoria sobre vício em dispositivos de
geração de número aleatório, o seu método seria bem pouco recomendado.
Algo mais certeiro seria calcular uma medida de concentração e/ou dispersão.

> --- Como explicar esse PARADOXO ? Como decidir com base em experimentos de 
> observação frequencial se uma moeda é, ou não, VICIADA ? Você acreditaria que 
> uma moeda é HONESTA se desse CARA em cem lançamentos consecutivos ?
>
> --
> Esta mensagem foi verificada pelo sistema de antivírus e
> acredita-se estar livre de perigo.

-- 
Esta mensagem foi verificada pelo sistema de antiv�rus e
 acredita-se estar livre de perigo.


=
Instru��es para entrar na lista, sair da lista e usar a lista em
http://www.mat.puc-rio.br/~obmlistas/obm-l.html
=


Re: [obm-l] CARA ou COROA com dez moedas

2022-01-04 Por tôpico Anderson Torres
Em qua., 22 de dez. de 2021 às 12:00, jamil dasilva
 escreveu:
>
> Duas pessoas disputam um CARA e COROA, jogando uma moeda honesta CEM VEZES.Um 
> deles aposta  que em todos os lançamentos ocorrerá  CARA e o outro, por sua 
> vez, aposta que ocorrerá CARA apenas  nos primeiros cinquenta lançamentos e, 
> consequentemente, cinquenta coroas nos cinquenta últimos.Qual deles tem maior 
> probabilidade de ganhar nesse CARA e COROA ?
>

Nem um nem outro.

Ambos estão apostando uma mesma coisa: se a sequência que vai sair é



ou

1100

as quais são equiprováveis.

>
>
>
> --
> Esta mensagem foi verificada pelo sistema de antivírus e
> acredita-se estar livre de perigo.

-- 
Esta mensagem foi verificada pelo sistema de antiv�rus e
 acredita-se estar livre de perigo.


=
Instru��es para entrar na lista, sair da lista e usar a lista em
http://www.mat.puc-rio.br/~obmlistas/obm-l.html
=


Re: [obm-l] Irracionalidade

2021-12-28 Por tôpico Israel Meireles Chrisostomo
Muito obrigado Ralph, era isso sim!!!

Em seg, 27 de dez de 2021 14:56, Ralph Costa Teixeira 
escreveu:

> Um segmento de reta de comprimento x sempre pode ser preenchido com n
> segmentos de reta iguais de comprimento x/n (sem superposição), mesmo que x
> seja irracional.
>
> Agora: se o segmento "maior" tiver comprimento x irracional e o segmento
> "menor" tiver comprimento y RACIONAL, não podemos preencher o maior com n
> cópias do menor, sem superposição.
>
> Afinal, se pudéssemos, teríamos x=ny; mas como y=p/q com p e q inteiros,
> viria que x=(np)/q, onde np e q são inteiros. Ou seja, x seria racional.
>
> Era isso?
>
> On Mon, Dec 27, 2021 at 2:01 PM Armando Staib 
> wrote:
>
>> Acredito que sim , porque se pudéssemos dividir por n seria um número
>> racional.  Concorda?
>> São segmentos incomensuráveis.
>>
>> Se eu estiver errado DESCULPE-ME
>>
>> Em dom., 26 de dez. de 2021 às 16:14, Israel Meireles Chrisostomo <
>> israelmchrisost...@gmail.com> escreveu:
>>
>>> Uma dada reta tem comprimento irracional então é impossível preenche-la
>>> com n segmentos de retas iguais?
>>>
>>> --
>>> Esta mensagem foi verificada pelo sistema de antivírus e
>>> acredita-se estar livre de perigo.
>>
>>
>> --
>> Esta mensagem foi verificada pelo sistema de antivírus e
>> acredita-se estar livre de perigo.
>
>
> --
> Esta mensagem foi verificada pelo sistema de antivírus e
> acredita-se estar livre de perigo.

-- 
Esta mensagem foi verificada pelo sistema de antiv�rus e
 acredita-se estar livre de perigo.



Re: [obm-l] Irracionalidade

2021-12-27 Por tôpico Ralph Costa Teixeira
Um segmento de reta de comprimento x sempre pode ser preenchido com n
segmentos de reta iguais de comprimento x/n (sem superposição), mesmo que x
seja irracional.

Agora: se o segmento "maior" tiver comprimento x irracional e o segmento
"menor" tiver comprimento y RACIONAL, não podemos preencher o maior com n
cópias do menor, sem superposição.

Afinal, se pudéssemos, teríamos x=ny; mas como y=p/q com p e q inteiros,
viria que x=(np)/q, onde np e q são inteiros. Ou seja, x seria racional.

Era isso?

On Mon, Dec 27, 2021 at 2:01 PM Armando Staib 
wrote:

> Acredito que sim , porque se pudéssemos dividir por n seria um número
> racional.  Concorda?
> São segmentos incomensuráveis.
>
> Se eu estiver errado DESCULPE-ME
>
> Em dom., 26 de dez. de 2021 às 16:14, Israel Meireles Chrisostomo <
> israelmchrisost...@gmail.com> escreveu:
>
>> Uma dada reta tem comprimento irracional então é impossível preenche-la
>> com n segmentos de retas iguais?
>>
>> --
>> Esta mensagem foi verificada pelo sistema de antivírus e
>> acredita-se estar livre de perigo.
>
>
> --
> Esta mensagem foi verificada pelo sistema de antivírus e
> acredita-se estar livre de perigo.

-- 
Esta mensagem foi verificada pelo sistema de antiv�rus e
 acredita-se estar livre de perigo.



Re: [obm-l] Irracionalidade

2021-12-27 Por tôpico Armando Staib
Acredito que sim , porque se pudéssemos dividir por n seria um número
racional.  Concorda?
São segmentos incomensuráveis.

Se eu estiver errado DESCULPE-ME

Em dom., 26 de dez. de 2021 às 16:14, Israel Meireles Chrisostomo <
israelmchrisost...@gmail.com> escreveu:

> Uma dada reta tem comprimento irracional então é impossível preenche-la
> com n segmentos de retas iguais?
>
> --
> Esta mensagem foi verificada pelo sistema de antivírus e
> acredita-se estar livre de perigo.

-- 
Esta mensagem foi verificada pelo sistema de antiv�rus e
 acredita-se estar livre de perigo.



Re: [obm-l] CARA ou COROA com dez moedas

2021-12-22 Por tôpico Claudio Buffara
Uma pergunta mais interessante é: Qual o número esperado de lançamentos da
moeda até que um deles vença?

On Wed, Dec 22, 2021 at 12:00 PM jamil dasilva 
wrote:

> Duas pessoas disputam um CARA e COROA, jogando uma moeda honesta *CEM* 
> VEZES.Um
> deles aposta  que em todos os lançamentos ocorrerá  CARA e o outro, por sua
> vez, aposta que ocorrerá CARA *apenas*  nos *primeiros
> cinquenta lançamentos* e, consequentemente, cinquenta coroas nos
> cinquenta últimos.Qual deles tem maior probabilidade de ganhar nesse CARA e
> COROA ?
>
>
>
>
> --
> Esta mensagem foi verificada pelo sistema de antivírus e
> acredita-se estar livre de perigo.

-- 
Esta mensagem foi verificada pelo sistema de antiv�rus e
 acredita-se estar livre de perigo.



Re: [obm-l] CARA ou COROA com dez moedas

2021-12-22 Por tôpico Caio Costa
nenhum deles, cada um tem uma chance de 1/2^100 de ganhar, e os dois não
ganham ao mesmo tempo.

Em qua., 22 de dez. de 2021 às 12:00, jamil dasilva 
escreveu:

> Duas pessoas disputam um CARA e COROA, jogando uma moeda honesta *CEM* 
> VEZES.Um
> deles aposta  que em todos os lançamentos ocorrerá  CARA e o outro, por sua
> vez, aposta que ocorrerá CARA *apenas*  nos *primeiros
> cinquenta lançamentos* e, consequentemente, cinquenta coroas nos
> cinquenta últimos.Qual deles tem maior probabilidade de ganhar nesse CARA e
> COROA ?
>
>
>
>
> --
> Esta mensagem foi verificada pelo sistema de antivírus e
> acredita-se estar livre de perigo.

-- 
Esta mensagem foi verificada pelo sistema de antiv�rus e
 acredita-se estar livre de perigo.



[obm-l] Re: [obm-l] Números de tentativas

2021-12-14 Por tôpico Ralph Costa Teixeira
Hm, primeiro precisamos deixar o enunciado mais preciso:

i) Eu preciso apenas DESCOBRIR a senha, ou preciso INSERI-LA no dispositivo?
ii) O dispositivo avisa quando a gente acerta a senha totalmente (acho que
o usual seria "sim")? Ou apenas diz "não"/"quase"?
iii) "Coincidente" significa digito correto na posição correta, ou apenas
"aparece em algum lugar da senha"?
iv) A priori, a senha pode ter dígitos repetidos (acho que o usual seria
"sim")?
v) A senha seria um CONJUNTO de 3 dígitos, ou a ordem importa (acho que o
usual seria "ordem importa")?

Para uma cota inferior (usualmente bem ruinzinha), tem uma ideia que
funciona em vários problemas deste tipo: qualquer algoritmo vai pegar uma
sequência de respostas do dispositivo (digamos, Q="quase", N="nao" e
A="acertou!") e traduzir isso numa possivel senha. Em outras palavras, por
mais complexo que seja o algoritmo, no final das contas ele "gera" uma
grande tabela, algo assim:

Se as respostas forem QQNQNQA, a senha vai ser 127;
Se as respostas forem NNQQNA, a senha vai ser 889;
...
e assim por diante. Por isso, se o número de sequências de letras for MENOR
que o número possivel de senhas, não tem como o algoritmo funcionar
GARANTIDAMENTE -- haverá senhas fora da tabela (ou sequências que levam a
mais de uma senha, evidenciando a falha do algoritmo nesses casos)!

Para ser um pouco mais concreto, vou supor 10^3 possíveis senhas (dígitos
ordenados, com repetição). Vou provar que, neste caso, um algoritmo com 9
tentativas NUNCA descobre a senha -- tem que ser pelo menos 10.

Duas outras observações interessantes:
a) Obviamente, se em algum momento seu algoritmo chega em A, PARE, você
achou a senha correta e **nenhuma das tentativas seguintes te
providencia nenhuma informação adicional**. Se você inventar um algoritmo
doido que continua tentando coisas depois do A, eu posso fazê-lo ficar MAIS
EFICIENTE retirando os passos adicionais; ou seja, fazendo todas as
sequências com terminarem nesse A;
b) Por outro lado, vou supor que você TEM QUE INSERIR a senha correta; ou
seja todas as sequências da sua "tabela" terminam em "A".

Assim, o número MÁXIMO de sequências de letras na sua tabela seria:
Comprimento 1: 1 sequência (a saber, "A")
Comprimento 2: 2 sequências (NA e QA)
Comprimento 3: 4 sequências (NNA, NQA, QNA, QQA)
...
Comprimento 9: 2^8=256 sequências
Total: 511 sequências ("máximo" pois, dependendo do algoritmo, talvez
algumas nunca ocorram). Como são 1000 possíveis senhas, é impossível seu
algoritmo distingui-las todas!



On Mon, Dec 13, 2021 at 10:00 AM Jeferson Almir 
wrote:

> Amigos peço ajuda nessa questão.
>
> Tem uma senha de 3 digitos
> (Qualquer digito  de 0 a 9)
> E nos temos um dispositivo
> Que compara a senha
> Com um número que escolhemos
> E retorna não se tem todos os digitos diferentes da senha
> E retorna quase se tem pelo menos 1 digito coincidente com a senha
> Qual é o menor numero de tentativas que precisamos usar esse dispositivo
> tal que podemos descobrir a senha com certeza, independente de qual ela
> seja?
>
> --
> Esta mensagem foi verificada pelo sistema de antivírus e
> acredita-se estar livre de perigo.

-- 
Esta mensagem foi verificada pelo sistema de antiv�rus e
 acredita-se estar livre de perigo.



[obm-l] Re: [obm-l] Números de tentativas

2021-12-13 Por tôpico Anderson Torres
Em seg., 13 de dez. de 2021 às 10:00, Jeferson Almir
 escreveu:
>
> Amigos peço ajuda nessa questão.
>
> Tem uma senha de 3 digitos
> (Qualquer digito  de 0 a 9)
> E nos temos um dispositivo
> Que compara a senha
> Com um número que escolhemos
> E retorna não se tem todos os digitos diferentes da senha
> E retorna quase se tem pelo menos 1 digito coincidente com a senha
> Qual é o menor numero de tentativas que precisamos usar esse dispositivo tal 
> que podemos descobrir a senha com certeza, independente de qual ela seja?

Por ora eu vou fazer uma tentativa.

Se fosse uma senha de um dígito, temos 10 tentativas.

Se fossem dois, bem, vamos pensar um pouco. Inicialmente não sei o que
fazer, vou simplesmente chutar AB

1. A máquina diz "acertou 2". 1 tentativa
2. A máquina diz "acertou 1". Aqui reduzimos o conjunto de tentativas
em 18 (A?- 9 tentativas; B? -  9 tentativas)
3. A máquina diz "acertou 0". Aqui piora: 9*9=81 tentativas sobrando.

No caso mais desfavorável, em 5 tentativas dá para limpar o conjunto.

Claro, isso não prova nada ainda.

>
> --
> Esta mensagem foi verificada pelo sistema de antivírus e
> acredita-se estar livre de perigo.

-- 
Esta mensagem foi verificada pelo sistema de antiv�rus e
 acredita-se estar livre de perigo.


=
Instru��es para entrar na lista, sair da lista e usar a lista em
http://www.mat.puc-rio.br/~obmlistas/obm-l.html
=


[obm-l] Re: [obm-l] Re: [obm-l] Invertíveis e Divisores de Zero

2021-11-30 Por tôpico Pedro Júnior
Sim...

Em ter., 30 de nov. de 2021 às 15:21, Claudio Buffara <
claudio.buff...@gmail.com> escreveu:

> Z_4 x Z_5 é isomorfo a Z_20.
> Talvez isso ajude.
>
> On Tue, Nov 30, 2021 at 2:33 PM Pedro Júnior 
> wrote:
>
>> Quem puder ajudar...
>> Encontre todos os invertíveis e divisores de zero em Z_4 x Z_5.
>>
>>
>>
>> --
>> Esta mensagem foi verificada pelo sistema de antivírus e
>> acredita-se estar livre de perigo.
>
>
> --
> Esta mensagem foi verificada pelo sistema de antivírus e
> acredita-se estar livre de perigo.



-- 

Pedro Jerônimo S. de O. Júnior

Professor de Matemática

Geo João Pessoa – PB

-- 
Esta mensagem foi verificada pelo sistema de antiv�rus e
 acredita-se estar livre de perigo.



[obm-l] Re: [obm-l] Invertíveis e Divisores de Zero

2021-11-30 Por tôpico Claudio Buffara
Z_4 x Z_5 é isomorfo a Z_20.
Talvez isso ajude.

On Tue, Nov 30, 2021 at 2:33 PM Pedro Júnior 
wrote:

> Quem puder ajudar...
> Encontre todos os invertíveis e divisores de zero em Z_4 x Z_5.
>
>
>
> --
> Esta mensagem foi verificada pelo sistema de antivírus e
> acredita-se estar livre de perigo.

-- 
Esta mensagem foi verificada pelo sistema de antiv�rus e
 acredita-se estar livre de perigo.



[obm-l] Re: [obm-l] Re: [obm-l] Valor máximo

2021-11-29 Por tôpico Artur Costa Steiner
>
> Se a, b e c são positivos e a^2+b^2+c^2 = 1, qual o valor máximo de
> (1-a)(1-b)(1-c)?
>
>> Desde já agradeço
>>
>
Podemos usar multiplicadores de Lagrange. Seja

f(a,b,c,L) = (1-a)(1-b)(1-c) -L(a^2 + b^2 + c^2 - 1)

Tomando as derivadas parciais de f com relação a a, b, c e L e igualando a
0,  obtemos

2a = L(1- b)(1 - c)
2b= L(1- a)(1 - c)
2c = L(1- a)(1 -c)
a^2+b^2+c^2 = 1

Se L <> 0 e se  nenhuma variável for 1, obtemos

a/(1 - b) = b/(1 - a), sendo as outras 2 equações permutações circulares da
1a. Segue-se que

a - a^2 = b - b^2
a - b = (a - b)(a + b) ==> a + b = 1. Considerando as outras 2 equações
chegamos a

a = b = c = raiz(3)/3. Isto leva a que a que (1 - a)(1-b)(1- c) = (1 -
raiz(3)/3)^3

Se L = 0 as equações conduzem às ternas (1,0,0), (0,1,0) e a (0, 0, 1) para
as quais (1 - a)(1-b)(1- c) = 0 < (1 - raiz(3)/3)^3

Como se trata de função contínua em conjunto compacto, as ternas acima dão
o mínimo absoluto e (raiz(3)/3,raiz(3)/3), raiz(3)/3) dá o máximo absoluto
no valor já citado

Nesse problemas geralmente há tambén uma solução baseada em desugualdades
como MA, MG, etc

Artur




















Km

-- 
Esta mensagem foi verificada pelo sistema de antiv�rus e
 acredita-se estar livre de perigo.



[obm-l] Re: [obm-l] Valor máximo

2021-11-29 Por tôpico Ian Barquette
Essa equação é a de uma esfera (x-x0)²+(y-y0)²+(z-z0)²=r², no caso da sua
ela estaria com centro em (0, 0, 0), e raio 1.

Espero que ajude

Em ter., 23 de nov. de 2021 21:54, marcone augusto araújo borges <
marconeborge...@hotmail.com> escreveu:

> Se a, b e c são positivos e a^2+b^2+c^2 = 1, qual o valor máximo de
> (1-a)(1-b)(1-c)?
> Desde já agradeço
>
> --
> Esta mensagem foi verificada pelo sistema de antivírus e
> acredita-se estar livre de perigo.
>

-- 
Esta mensagem foi verificada pelo sistema de antiv�rus e
 acredita-se estar livre de perigo.



[obm-l] Re: [obm-l] Valor máximo

2021-11-29 Por tôpico Anderson Torres
Em ter., 23 de nov. de 2021 às 21:54, marcone augusto araújo borges
 escreveu:
>
> Se a, b e c são positivos e a^2+b^2+c^2 = 1, qual o valor máximo de 
> (1-a)(1-b)(1-c)?

Acho, só acho, que dá para simplesmente fazer assim:

Se fixarmos c, temos que determinar o máximo de (1-a)(1-b) dado que
a^2+b^2=Z^2(=1-c^2) Minha suspeita levemente mal fundada é que isso é
máximo quando a e b são iguais. Com isso bastaria maximizar uma certa
função em Z.

Outra forma seria escrever a = cosF, b = sinF sinG, c=sinF cosG e usar
um pouco de análise de uma variável, por exemplo fixando G e
verificando F.

> Desde já agradeço
>
> --
> Esta mensagem foi verificada pelo sistema de antivírus e
> acredita-se estar livre de perigo.

-- 
Esta mensagem foi verificada pelo sistema de antiv�rus e
 acredita-se estar livre de perigo.


=
Instru��es para entrar na lista, sair da lista e usar a lista em
http://www.mat.puc-rio.br/~obmlistas/obm-l.html
=


[obm-l] Re: [obm-l] Re: [obm-l] Solução do problema de fatoração (quebra do RSA)

2021-11-25 Por tôpico Rodrigo Ângelo
Sobre o passo 6, você quis dizer aleatório com distribuição uniforme?

On Thu, Nov 25, 2021, 09:59 Eric Campos Bastos Guedes <
ebastosgue...@gmail.com> wrote:

>
> Estou trabalhando num algoritmo melhor, mas preciso de acesso a um
> computador com o software de computação algébrica  Maple que é o que eu sei
> usar. Espero ter o retorno de pessoas que sabem mais do que eu.
>
> Em dom., 14 de nov. de 2021 12:58, Claudio Buffara <
> claudio.buff...@gmail.com> escreveu:
>
>> Por que vc não testa?
>>
>> On Sun, Nov 14, 2021 at 9:53 AM Eric Campos Bastos Guedes <
>> ebastosgue...@gmail.com> wrote:
>>
>>> Eu preciso de um retorno sobre o meu algoritmo que quebra o RSA
>>> resolvendo o problema de fatoracao.
>>>
>>> O Passo 8 talvez possa ser substituido por: PASSO 8': P = P(P+C)
>>>
>>> Em seg., 6 de set. de 2021 07:47, Eric Campos Bastos Guedes <
>>> ebastosgue...@gmail.com> escreveu:
>>>
 Aparentemente minha conexão está raqueada por gente do Bolsonaro e eu
 não estou recebendo respostas para minha postagem e também não estou
 conseguindo acessar os sites de discussão sobre o RSA. Há pessoas se
 fazendo passar por mim também.

 -- Forwarded message -
 De: Eric Campos Bastos Guedes 
 Date: sáb., 4 de set. de 2021 00:33
 Subject: [obm-l]
 To: 


 Olá a todos. Gostaria de pedir licença para que vocês avaliem um
 algoritmo que eu fiz para fatorar números grandes com fatores primos também
 grandes. Eu acredito que esse algoritmo quebre o RSA

 O algoritmo é o seguinte:

 PASSO 1: faça A=3
 PASSO 2: N é o inteiro a ser fatorado
 PASSO 3: M = N**16 (potência)
 PASSO 4: faça A=A+1
 PASSO 5: faça P=A
 PASSO 6: faça B=número aleatório entre 0 e 1
 PASSO 7: se B eh maior que 0.5 faça C=1 senão faça C = -1
 PASSO 8: faça P=(PP+CP)/2=P(P+C)/2
 PASSO 9: se P eh menor que M  vá para o PASSO 6
 PASSO 10: se mdc(P, N) = 1 faça M=MM e vá para o PASSO 4
 PASSO 11: se mdc(P, N) = N faça M = raiz quadrada de M e vá para o
 PASSO 4
 PASSO 12: mdc(P, N) é fator de N
 FIM

 AUTOR: ERIC CAMPOS BASTOS GUEDES  (DIA 4 DE SETEMBRO)

 Creio ter resolvido o problema de fatoração. Alguém pode verificar isso
 para mim.

 --
 Esta mensagem foi verificada pelo sistema de antivírus e
 acredita-se estar livre de perigo.

>>>
>>> --
>>> Esta mensagem foi verificada pelo sistema de antivírus e
>>> acredita-se estar livre de perigo.
>>
>>
>> --
>> Esta mensagem foi verificada pelo sistema de antivírus e
>> acredita-se estar livre de perigo.
>
>
> --
> Esta mensagem foi verificada pelo sistema de antivírus e
> acredita-se estar livre de perigo.

-- 
Esta mensagem foi verificada pelo sistema de antiv�rus e
 acredita-se estar livre de perigo.



[obm-l] Re: [obm-l] Solução do problema de fatoração (quebra do RSA)

2021-11-25 Por tôpico Eric Campos Bastos Guedes
Estou trabalhando num algoritmo melhor, mas preciso de acesso a um
computador com o software de computação algébrica  Maple que é o que eu sei
usar. Espero ter o retorno de pessoas que sabem mais do que eu.

Em dom., 14 de nov. de 2021 12:58, Claudio Buffara <
claudio.buff...@gmail.com> escreveu:

> Por que vc não testa?
>
> On Sun, Nov 14, 2021 at 9:53 AM Eric Campos Bastos Guedes <
> ebastosgue...@gmail.com> wrote:
>
>> Eu preciso de um retorno sobre o meu algoritmo que quebra o RSA
>> resolvendo o problema de fatoracao.
>>
>> O Passo 8 talvez possa ser substituido por: PASSO 8': P = P(P+C)
>>
>> Em seg., 6 de set. de 2021 07:47, Eric Campos Bastos Guedes <
>> ebastosgue...@gmail.com> escreveu:
>>
>>> Aparentemente minha conexão está raqueada por gente do Bolsonaro e eu
>>> não estou recebendo respostas para minha postagem e também não estou
>>> conseguindo acessar os sites de discussão sobre o RSA. Há pessoas se
>>> fazendo passar por mim também.
>>>
>>> -- Forwarded message -
>>> De: Eric Campos Bastos Guedes 
>>> Date: sáb., 4 de set. de 2021 00:33
>>> Subject: [obm-l]
>>> To: 
>>>
>>>
>>> Olá a todos. Gostaria de pedir licença para que vocês avaliem um
>>> algoritmo que eu fiz para fatorar números grandes com fatores primos também
>>> grandes. Eu acredito que esse algoritmo quebre o RSA
>>>
>>> O algoritmo é o seguinte:
>>>
>>> PASSO 1: faça A=3
>>> PASSO 2: N é o inteiro a ser fatorado
>>> PASSO 3: M = N**16 (potência)
>>> PASSO 4: faça A=A+1
>>> PASSO 5: faça P=A
>>> PASSO 6: faça B=número aleatório entre 0 e 1
>>> PASSO 7: se B eh maior que 0.5 faça C=1 senão faça C = -1
>>> PASSO 8: faça P=(PP+CP)/2=P(P+C)/2
>>> PASSO 9: se P eh menor que M  vá para o PASSO 6
>>> PASSO 10: se mdc(P, N) = 1 faça M=MM e vá para o PASSO 4
>>> PASSO 11: se mdc(P, N) = N faça M = raiz quadrada de M e vá para o PASSO
>>> 4
>>> PASSO 12: mdc(P, N) é fator de N
>>> FIM
>>>
>>> AUTOR: ERIC CAMPOS BASTOS GUEDES  (DIA 4 DE SETEMBRO)
>>>
>>> Creio ter resolvido o problema de fatoração. Alguém pode verificar isso
>>> para mim.
>>>
>>> --
>>> Esta mensagem foi verificada pelo sistema de antivírus e
>>> acredita-se estar livre de perigo.
>>>
>>
>> --
>> Esta mensagem foi verificada pelo sistema de antivírus e
>> acredita-se estar livre de perigo.
>
>
> --
> Esta mensagem foi verificada pelo sistema de antivírus e
> acredita-se estar livre de perigo.

-- 
Esta mensagem foi verificada pelo sistema de antiv�rus e
 acredita-se estar livre de perigo.



[obm-l] Re: [obm-l] Solução do problema de fatoração (quebra do RSA)

2021-11-24 Por tôpico Eric Campos Bastos Guedes
Estou trabalhando num algoritmo melhor, mas preciso de acesso a um
computador com o software de computação algébrica  Maple que é o que eu sei
usar. Espero ter o retorno de pessoas que sabem mais do que eu.

Em dom., 14 de nov. de 2021 12:58, Claudio Buffara <
claudio.buff...@gmail.com> escreveu:

> Por que vc não testa?
>
> On Sun, Nov 14, 2021 at 9:53 AM Eric Campos Bastos Guedes <
> ebastosgue...@gmail.com> wrote:
>
>> Eu preciso de um retorno sobre o meu algoritmo que quebra o RSA
>> resolvendo o problema de fatoracao.
>>
>> O Passo 8 talvez possa ser substituido por: PASSO 8': P = P(P+C)
>>
>> Em seg., 6 de set. de 2021 07:47, Eric Campos Bastos Guedes <
>> ebastosgue...@gmail.com> escreveu:
>>
>>> Aparentemente minha conexão está raqueada por gente do Bolsonaro e eu
>>> não estou recebendo respostas para minha postagem e também não estou
>>> conseguindo acessar os sites de discussão sobre o RSA. Há pessoas se
>>> fazendo passar por mim também.
>>>
>>> -- Forwarded message -
>>> De: Eric Campos Bastos Guedes 
>>> Date: sáb., 4 de set. de 2021 00:33
>>> Subject: [obm-l]
>>> To: 
>>>
>>>
>>> Olá a todos. Gostaria de pedir licença para que vocês avaliem um
>>> algoritmo que eu fiz para fatorar números grandes com fatores primos também
>>> grandes. Eu acredito que esse algoritmo quebre o RSA
>>>
>>> O algoritmo é o seguinte:
>>>
>>> PASSO 1: faça A=3
>>> PASSO 2: N é o inteiro a ser fatorado
>>> PASSO 3: M = N**16 (potência)
>>> PASSO 4: faça A=A+1
>>> PASSO 5: faça P=A
>>> PASSO 6: faça B=número aleatório entre 0 e 1
>>> PASSO 7: se B eh maior que 0.5 faça C=1 senão faça C = -1
>>> PASSO 8: faça P=(PP+CP)/2=P(P+C)/2
>>> PASSO 9: se P eh menor que M  vá para o PASSO 6
>>> PASSO 10: se mdc(P, N) = 1 faça M=MM e vá para o PASSO 4
>>> PASSO 11: se mdc(P, N) = N faça M = raiz quadrada de M e vá para o PASSO
>>> 4
>>> PASSO 12: mdc(P, N) é fator de N
>>> FIM
>>>
>>> AUTOR: ERIC CAMPOS BASTOS GUEDES  (DIA 4 DE SETEMBRO)
>>>
>>> Creio ter resolvido o problema de fatoração. Alguém pode verificar isso
>>> para mim.
>>>
>>> --
>>> Esta mensagem foi verificada pelo sistema de antivírus e
>>> acredita-se estar livre de perigo.
>>>
>>
>> --
>> Esta mensagem foi verificada pelo sistema de antivírus e
>> acredita-se estar livre de perigo.
>
>
> --
> Esta mensagem foi verificada pelo sistema de antivírus e
> acredita-se estar livre de perigo.

-- 
Esta mensagem foi verificada pelo sistema de antiv�rus e
 acredita-se estar livre de perigo.



<    1   2   3   4   5   6   7   8   9   10   >